Sie sind auf Seite 1von 86

Copyright 2010.

Physician Assistant Education Association


1
ACKNOWLEDGEMENT

The time and effort provided by the following individuals who served as members of this committee are
greatly appreciated:

Kimberly Cavanagh, DHSc, PA-C
Kathleen Ehrhardt, MMS, PA-C
Denise Dillingham, MPAS, PA-C
Rex Hobbs, MPAS, PA-C
Annette Larson, MSPAP, PA-C
Antoinette Polito, MHS, PA-C
J udy Ortiz, PA-C, MHS
Daniel McDermott, MS, PA-C
Robert Baye, MPAS, PA-C

Subcommittee Members:
Ryan J unsay, PA-C, MSPA
Alan Gindoff, DHSc, MPAS, PA-C
Susan King-Barry, BSN, MPAS, PA-C
Mindy Milton, MPA, PA-C





DEDICATION

This examination would not have been possible without the years of commitment of the MR. TIB
Development Committee. Numerous PA educators from across the nation provided their experience and
insight as questions for MR. TIB. It has been this data bank that served as the building blocks for
PACKRAT.

PAEA is proud to be able to continue in the tradition of quality fostered by the forerunners of the self-
assessment examination for physical assistants. It is our honor to dedicate PACKRAT to:

J esse C. Edwards, MS
Claire S. Parker, PhD
University of Nebraska, Physician Assistant Program



















Copyright 2010. Physician Assistant Education Association
2




PHYSICIAN ASSISTANT EDUCATION ASSOCIATION

Physician Assistant Clinical Knowledge Rating and Assessment Tool (PACKRAT)

Form 15

Directions and Explanations


TABLE OF CONTENTS




I. Introduction 1

II. Explanation of the Score Report 4

Page 1: Scores Your total Score and Group Comparisons 4
Page 2: Your STRENGTHS, Weaknesses, and Quality of Responses 4
Page 3: Your Individual and Correct Responses 4
Page 4: Your Responses by Task and Category 4
Page 5-6: Your Profile Comparison: Demographic Profile 4

III. Recommendations for Using the Feedback Package 5

IV. Study Resources 5

V. Answer Key 7

VI. Examination Explanations 8

VII. Comment Form 86
















Copyright 2010. Physician Assistant Education Association. All rights reserved. No part of this publication may be reproduced or
transmitted in any form or by any means, electronic or mechanical, including photocopy or recording, or any information and
retrieval system, without permission in writing from the Physician Assistant Education Association.
Copyright 2010. Physician Assistant Education Association
3




PHYSICIAN ASSISTANT CLINICAL KNOWLEDGE RATING
AND ASSESSMENT TOOL (PACKRAT)



I. Introduction

The Physician Assistant Clinical Knowledge Rating and Assessment Tool (PACKRAT) was developed by
a volunteer committee of experts and is based on the content outline of a nationally recognized
competency examination. The following is a description of the content of PACKRAT:






PACKRAT EXAMINATION MATRIX

CONTENT AREA NUMBER OF ITEMS

1. History & Physical 26
2. Diagnostic Studies 36
3. Diagnosis 45
4. Health Maintenance 22
5. Clinical Intervention 32
6. Clinical Therapeutics 48
7. Scientific Concepts 16

TOTALS 225


Additionally, questions also apply to the following clinical specialties:

A. Cardiology I. Neurology
B. Dermatology J . Obstetrics/Gynecology
C. Endocrinology K. Orthopedics/Rheumatology
D. ENT L. Pediatrics
E. Ophthalmology M. Psychiatry/Behavioral Medicine
F. Gastrointestinal/Nutritional N. Pulmonology
G. Geriatrics O. Surgery
H. Hematology P. Urology/Renal




The task and specialty categories for each item are listed in the answer key on page 5; your feedback
package contains a breakdown of responses by the task and clinical specialty category. Pay particular
attention to the questions you answered incorrectly and determine the specialty for that question and use
this information to identify weaknesses.

The PACKRAT provides a detailed feedback report of performance and it is available to anyone at any
time. Explanations were developed for all the questions to provide a rationale for correct, as well as
incorrect, answers. This information will help determine strengths and weaknesses with respect to the

PACKRAT EXAMINATION MATRIX

CONTENT AREA NUMBER OF ITEMS

1. History & Physical 36
2. Diagnostic Studies 32
3. Diagnosis 41
4. Health Maintenance 22
5. Clinical Intervention 31
6. Clinical Therapeutics 41
7. Scientific Concepts 22

TOTALS 225


Additionally, questions also apply to the following clinical specialties:

A. Cardiology J . Obstetrics/Gynecology
B. Dermatology K. Orthopedics/Rheumatology
C. Endocrinology M. Psychiatry/Behavioral Medicine
D. ENT/Ophthalmology N. Pulmonology
F. Gastrointestinal/Nutritional P. Urology/Renal
H. Hematology Q. Infectious Diseases
I. Neurology
Copyright 2010. Physician Assistant Education Association
4
PACKRAT content outline. If you have weaknesses in specific areas, you may need to obtain additional
clinical experience in those areas.



This booklet is designed to explain and interpret the information contained in the accompanying
computerized score report. You can use the report package to learn more about your abilities.

II. Explanation of the Score Report

This section provides an interpretation of each
page of the computerized score report you
received. You should have your computer
score report in front of you. Begin on page 1
of the report and read the following
information.

Page 1: Scores Your Total Score and
Group Comparisons

Page 1 is an overview of the PACKRAT
feedback report. Toward the bottom of the
page is your examination score. This score
shows the number of questions you answered
correctly out of a possible 225. The average
score for all first-year and second-year
candidates who have taken the PACKRAT to
date is also given.

Page 2: Your Strengths, Weaknesses, and
Quality of Responses

Page 2 of the score report gives an overview
of the content area in which your performance
is categorized as Strong, Satisfactory, or
Needing Improvement. These areas are
based on the examination matrix on page 1.

In each content area, your answers have
been classified as correct, acceptable,
unsatisfactory, or harmful. A definition of
these classifications is also provided on this
page. Pay particular attention to the areas
under Needing Improvement, as these areas
should be noted for further study. Also check
the answer key for the specialty area of these
items. If you selected a harmful answer in any
content area, it will be automatically placed in
the Needing Improvement category,
regardless of the number of correct answers
selected. Carefully review these questions
and their explanations and specialty
classifications in Section VI to help you
understand why your answers were correct.


Page 3: Your Individual and Correct
Responses

Page 3 lists your answers to all questions.
When your answer differs from the correct
one, the proper response appears in
parentheses. Use Section VI with this page to
review the rationale for each option that is
provided in the explanations, which are
referenced to the study resources. The
explanations may help you understand why
one answer is more appropriate than another,
or not the best answer, and why some of your
answers may have been incorrect. If the
option you chose was judged potentially
harmful to the patient or others, an asterisk (*)
appears before your answer. Options
classified as potentially harmful may identify
serious weaknesses. Go over these questions
carefully and read the explanations for the
correct answers. You may be able to identify
areas where you need further study.

Page 4: Your Responses by Task and
Specialty Category

Page 4 lists your responses by both specialty
and task category. You will be able to identify
the areas of the content outline where you
may have difficulty. The numbers reflect how
many items you answered correctly out of the
total possible correct within each task and
specialty area. Categories 1 through 7 identify
the task areas and A-P the clinical specialty
areas. If you missed a significant number of
items in an area, check the key and go over
the explanations for the items in these areas.

Page 5-6: Your Profile Comparison:
Demographic Profile

Page 5 is the beginning of the Demographic
Profile Comparison for programs. This profile
shows the reported demographic information
and compares your students information to
the entire group of individuals who have taken
the PACKRAT to date. The demographic data
are based on the information your students
provided when answering the demographics
questions. The summary demographic
information shown reflects all the data
compiled for either first or second-year
students who taken the PACKRAT.
Copyright 2010. Physician Assistant Education Association
5

RESPONDENT DEMOGRAPHIC
INFORMATION

C. Number of months of clinical rotations
completed at the time of this exam?

Your response: All Second-year
Respondents:

( ) 1. None ( 0% )
( ) 2. Less than 3 months ( 5% )
( ) 3. 3 to 6 months ( 15%)
( ) 4. 7 to 9 months ( 20%)
( ) 5. 10 to 12 months (45%)
( ) 6. Greater than 12 months (15%)


Programs can utilize this information in order
to compare the characteristics of their first
year students and second year students to the
national demographics of first year and
second year students who have taken the
PACKRAT test.

III. Recommendations for using The
Feedback Package

As a current physician assistant student,
PACKRAT can be a useful self-evaluation
tool. Through careful review of question
explanations, noting specific tasks and
content areas, you will be able to assess your
current strengths and weaknesses. You will
be able to identify particular areas in which to
concentrate more effort as you continue your
studies. By concentrating your effort on the
areas in which you did not do well, you may
improve your performance, and you may have
a better chance of passing the proctored
examination. However, PAEA cannot
guarantee that this will occur, since the
conditions under which you attempted the
PACKRAT may have been different from
those in a standardized administration of a
proctored examination.
Use the explanations in Section VI to analyze
why you chose various options. Again, pay
particular attention to the options that were
judged potentially harmful or unsatisfactory.
Look at the question and the four options
again to see why the answer you chose was
incorrect. If there appears to be a deficit in
your exposure to a particular clinical specialty,
perhaps further study would make you more
familiar with these situations.
Once you have completely reviewed your
score report and this booklet, PAEA hopes
you will use this information to improve your
overall performance, either on the job or on
future certification examinations. Should you
wish to provide suggestions about PACKRAT
to PAEA, you will find a comment form on the
last page of this booklet.

IV. Study Resources

A variety of textbooks are currently available
to assist candidates in preparing for the
certification examination. For additional
information, you may contact a faculty
member at an educational program or an
experienced colleague if you need help
determining which references to review in a
specific content area. A short list of general
textbooks is below. All examination questions
are related to material found in these
resources.

Please note that the books on this list are not
available from PAEA. This is not intended as
an all-inclusive list, and the materials listed
below are suggested study materials only.

1. Andreoli TE, et al (eds). Cecils
Essentials of Medicine. 7th ed.
Philadelphia, PA: WB Saunders,
an Elsevier company, 2007.
2. Ballweg R et al. Physician
Assistant: A Guide to Clinical
Practice. 4
th
ed., WB Saunders,
an Elsevier company, 2008.
3. Bickley, LS. Bates Guide to
Physical Examination and History
Taking. 10th ed. Philadelphia,
PA: J B Lippincott Co., 2009.
4. Beckmann CR, et al. Obstetrics &
Gynecology. 6th ed. Philadelphia,
PA: Lippincott Williams & Wilkins,
2009.
5. Behrman RE, et al. Nelsons
Textbook of Pediatrics. 18th ed.
Philadelphia, PA: WB Saunders,
an Elsevier company, 2007.
6. Berkowitz, C. Pediatrics: A
Primary Care Approach, 3
rd
ed.
Philadelphia, PA: WB Saunders,
an Elsevier company 2008.
7. Fauci AS, et al (eds). Harrisons
Principles of Internal Medicine.
17th ed. New York, NY: McGraw-
Hill, Inc., 2008.
Copyright 2010. Physician Assistant Education Association
6
8. DeCherney AH & Pernoll ML
(eds.) Current Obstetric &
Gynecological Diagnosis &
Treatment, 10
th
ed., Norwalk, CT:
Appleton & Lange, 2006
9. Ellsworth AJ , et al. (eds). Mosbys
Medical Drug Reference.
Philadelphia, PA: Mosby, 2007.
10. Wolf K and J ohnson RA.
Fitzpatricks Color Atlas and
Synopsis of Clinical Dermatology.
6th ed. New York, NY: McGraw-
Hill, Inc., 2009.
11. Goldman J and Bennet J C. Cecil
Textbook of Medicine. 23
rd
ed.
Philadelphia, PA: WB Saunders.
2008.
12. Goroll, AH, Mulley, AG, & May,
LA. Primary Care Medicine: Office
Evaluation and Management of
the Adult Patient. 6
th
ed.,
Lippincott Williams &Wilkins, 2009
(To be released).
13. Hay WW, et al. Current Pediatric
Diagnosis and Treatment. 19
th

ed. Norwalk, CT: McGraw Hill,
2008.
14. Kaplan HI and Sadock BJ (eds).
Synopsis of Psychiatry. 10
th
ed.,
Philadelphia, PA: Williams &
Wilkins, 2007.
15. Katzung BG. Basic and Clinical
Pharmacology. 11th ed.
Stamford, CT: Appleton & Lange,
2009.
16. Mandel GL, Bennett J E, and Dolin
R. Principles and Practice of
Infectious Disease. 7th ed.,
Philadelphia, PA: Churchill
Livingston, an Elsevier company,
2009. ISBN#0443066434
17. Marx, J ., Hockberger, RS, and
Walls, RM. Rosens Emergency
Medicine: Concepts and Clinical
Practice. 7
th
ed., Philadelphia, PA:
Mosby, 2009.
18. McPhee SJ , et al.
Pathophysiology of Disease. 6
th

ed., McGraw Hill, 2009.
19. Mercier LR, et al. Practical
Orthopedics. 6th ed. Philadelphia,
PA: Mosby, an Elsevier company,
2008. ISBN#9780323036184
20. Mettler FA, et al. Primary Care
Radiology. Philadelphia, PA: WB
Saunders, Co., 2000. (In PAEA
library 2/07)
21. Howland RD nad Mycek MJ .
Lippincotts Illustrated Reviews:
Pharmacology. 4
th
ed.
Baltimore, MD: Williams & Wilkins,
2008.
22. Noble J , et al. Textbook of
Primary Care Medicine. 3rd ed.
Philadephia PA: Mosby, 2001. (In
PAEA library 2/07)
23. Riordan-Eva, P. and Whitcher, J P.
Vaughn & Asburys General
Ophthalmology. 17
th
ed., McGraw
Hill, 2007.
24. Sacher RA and McPherson RA.
Widmann's Clinical Interpretation
of Laboratory Tests. 11th ed. FA
Davis Co., 2000. (In PAEA library
2/07)
25. Brunicardi, et al. Principles of
Surgery. 9th ed. New York, NY:
McGraw-Hill, Inc., 2009.
26. Skinner HB (ed.) Current
Diagnosis & Treatment in
Orthopedics. 4
th
ed.,
Norwalk,CT:Appleton & Lange,
2008.
27. Steinberg GG. Orthopedics in
Primary Care. 3
rd
ed.
Philadelphia, PA: Lippincott
Williams & Wilkins, 3
rd
ed, 1999.
28. Tierney LM, et al. Current Medical
Diagnosis and Treatment. 49
th

ed. Stamford, CT: Appleton &
Lange, 2010.
29. Tintinalli J E, Kelen GD, and
Stapezynski J S. Emergency
Medicine: A Comprehensive
Study Guide. 6th ed. New York,
NY: McGraw-Hill, Inc., 2004.
30. Townsend CM. Sabistons
Textbook of Surgery. The
Biological Basis of Modern
Surgical Practice. 18 ed.
Philadelphia, PA: WB Saunders,
an Elsevier company, 2008.
31. Doherty GM. Current Surgical
Diagnosis and Treatment. 12
th

ed., McGraw Hill, 2005.
32. Wilson WR. Current Diagnosis
and Treatment in Infectious
Disease. 2
nd
ed., Norwalk, CT:
Appleton & Lange, 2009.





Copyright 2010. Physician Assistant Education Association
7




















THIS PAGE INTENTIONALLY
LEFT BLANK FOR ANSWER
KEY



























Copyright 2010. Physician Assistant Education Association

8
PHYSICIAN ASSISTANT EDUCATION ASSOCIATION

Physician Assistant Clinical Knowledge Rating and Assessment Tool

(PACKRAT) Form 15


EXPLANATIONS




1. History & Physical/Cardiology
A 23 year-old male with recent upper respiratory symptoms presents complaining of chest pain. His pain is worse
lying down and better sitting up and leaning forward. Electrocardiogram shows widespread ST segment elevation.
Which of the following is the most likely physical examination finding in this patient?
A. Elevated blood pressure
B. Subungual hematoma
C. Diastolic murmur
D. Pericardial friction rub
Explanations
(u) A. Acute pericarditis is usually not associated with elevated blood pressure. One would expect to see
hypertensive pressures in the setting of an aortic dissection.
(u) B. Subungual hematomas are usually seen in endocarditis not pericarditis.
(u) C. A diastolic murmur in a patient with chest pain would likely be associated with acute aortic regurgitation in the
setting of an aortic dissection.
(c) D. This patient has symptoms consistent with acute pericarditis and would most likely have a pericardial friction
rub on examination.
Ref: (28)



2. Diagnostic Studies/ENT/Ophthalmology
A 75 year-old woman presents to the office with complaint of vision loss. Examination reveals a palpable cord in the
temporal region. Which of the following is the most helpful initial test to order on this patient?
A. Carotid ultrasound
B. Chest x-ray
C. Complete blood count
D. Erythrocyte sedimentation rate
Explanations
(u) A. See D for explanation.
(u) B. See D for explanation.
(u) C. See D for explanation.
(c) D. The patient is suspected of having temporal arteritis. This disease is most commonly noted in patients over
age 50 and should be suspected in patients with sudden vision loss and a palpable cord in the temporal region.
Erythrocyte sedimentation rate is almost always increased in this disease.
Ref: (7)






Copyright 2010. Physician Assistant Education Association

9
3. Scientific Concepts/Endocrinology
What is the recommended target LDL to reduce the risk of coronary artery disease in a diabetic patient?
A. 200 mg/dL
B. 160 mg/dL
C. 130 mg/dL
D. 100 mg/dL
Explanations
(u) A. See D for explanation.
(u) B. See D for explanation.
(u) C. See D for explanation.
(c) D. The National Cholesterol Education Program clinical practice guidelines have designated diabetes as a
coronary risk equivalent and have recommended that patients with diabetes should have an LDL cholesterol
goal of 100 mg/dL.
Ref: (28)



4. Clinical Therapeutics/Obstetrics/Gynecology
Which of the following best describes the purpose of intravenous magnesium sulfate in patients with preeclampsia?
A. Prevention of convulsions
B. Prevention of HELLP syndrome
C. Lowering of blood pressure
D. Reversal of proteinuria
Explanations
(c) A. Magnesium sulfate is used to prevent and treat eclamptic seizures. Magnesium sulfate is not sufficient to treat
hypertension, therefore antihypertensives must be added. Magnesium sulfate is excreted solely from the
kidneys and urine output must be preserved to prevent accumulation of the drug. Magnesium sulfate does
nothing to prevent HELLP syndrome.
(u) B. See A for explanation.
(u) C. See A for explanation.
(u) D. See A for explanation.
Ref: (4)


5. Diagnostic Studies/Cardiology
A 53 year-old male with history of hypertension presents complaining of recent 4/10 left-sided chest pain with exertion
that is relieved with rest. He states the pain usually lasts approximately 4 minutes and is relieved with rest. Heart
examination reveals regular rate and rhythm with no S3, S4, or murmur. Lungs are clear to auscultation bilaterally.
Electrocardiogram reveals no acute changes. Which of the following is the most appropriate initial step in the
evaluation of this patient?
A. Cardiac catheterization
B. CT Angiogram of the chest
C. Echocardiogram
D. Nuclear stress test
Explanations
(u) A. This patient has signs and symptoms consistent with stable angina. Noninvasive diagnostic testing is
preferred in this patient.
(u) B. CT angiogram may be useful for the evaluation of chest pain, however its role in routine practice has not
been established.
(u) C. This patient has signs and symptoms of stable angina. There are no signs of valvular heart disease on
examination. While an echocardiogram may be performed at some point, it is not the best initial diagnostic
step to determine the etiology of the patient's angina.
(c) D. Nuclear stress testing is the most appropriate initial diagnostic study in the evaluation of a patient with signs
and symptoms consistent with stable angina.
Ref: (28)

Copyright 2010. Physician Assistant Education Association

10
6. Diagnosis/Pulmonology
A 36 year-old African American female comes to the clinic for an insurance physical which requires a chest x-ray.
She denies any respiratory symptoms. Examination of her chest is negative. X-ray results show marked
lymphadenopathy in the right paratracheal region. Angiotensin-converting enzyme (ACE) levels are elevated. Which
of the following is the most likely diagnosis?
A. Sarcoidosis
B. Tuberculosis
C. Pulmonary fibrosis
D. Lymphoma
Explanations
(c) A. Sarcoidosis is characterized by paratracheal lymphadenopathy and elevated ACE levels. It is more common
in African American patients and may be asymptomatic.
(u) B. See A for explanation.
(u) C. See A for explanation.
(u) D. See A for explanation.
Ref: (28)



7. Scientific Concepts/Orthopedics/Rheumatology
A 33 year-old male presents to your office with a complaint of right knee injury associated with pain and swelling. He
states he was running after his loose dog and suddenly stopped, hyperextended his knee, heard a pop and noticed
immediate swelling. On physical examination, the Lachman test and anterior drawer test demonstrates joint laxity.
Which of the following ligaments is most likely injured?
A. Medial collateral
B. Lateral collateral
C. Posterior cruciate
D. Anterior cruciate
Explanations
(u) A. Medial collateral ligament injuries often occur with rotational injuries or direct impact to the lateral knee.
Tenderness medially with laxity with valgus (medial) stress is noted.
(u) B. Lateral collateral ligament injury causes pain mostly on the lateral aspect of the knee and patients can
experience knee buckling with normal gait. Tenderness laterally with laxity with varus (lateral) stress is noted.
(u) C. Posterior cruciate ligament injuries occur with an outside directed force, often a posterior directed force such
as a knee striking a dashboard. The patients often do not hear a pop. A posterior drawer test or posterior sag
test can be useful in the diagnosis.
(c) D. Anterior cruciate ligament injuries occur with sudden deceleration injuries. Patients often hear a pop and the
diagnosis is aided by assessing the anterior drawer test and Lachman test. The immediate swelling as well as
laxity with anterior drawer test and Lachman test should raise suspicion of anterior cruciate ligamental injury.
Ref: (26)



8. Diagnosis/Urology/Renal
A 22 year-old woman comes to the office because her urine is cola-colored and she has not urinated since yesterday
morning. Her past medical history is significant for pharyngitis two weeks ago. Her mother and grandmother have
type 2 diabetes. Her blood pressure is 146/92mmHG. On physical examination, she has edema of her face and
hands. Which of the following is the most likely diagnosis?
A. Glomerulonephritis
B. Acute tubular necrosis
C. Nephrolithiasis
D. Diabetic nephropathy
Explanations
(c) A. Glomerulonephritis presents with hematuria, cola-colored urine, oliguria, and edema of the face and eyes in
the morning. Urinalysis reveals red blood cells, mild proteinuria and red blood cell casts. Glomerulonephritis

Copyright 2010. Physician Assistant Education Association

11
can occur 1-3 weeks after a strep infection.
(u) B. Acute tubular necrosis is caused by acute kidney injury, such as a nephrotoxin, and is associated with uremic
symptoms which include nausea, vomiting, malaise, and altered mental status. Granular casts are non-
specific and may be seen in acute tubular necrosis.
(u) C. Nephrolithiasis usually presents as a sudden onset of colicky flank pain with associated nausea and vomiting.
Urinalysis often reveals gross or microscopic hematuria.
(u) D. Diabetic nephropathy is the most common cause of end stage renal disease in the United States. Urine
examination reveals albuminuria.
Ref: (28)



9. Diagnostic Studies/ENT/Ophthalmology
A 15 year-old girl is hit in the right eye by a golf ball. There is swelling and ecchymosis of the orbit. She complains of
double vision. The right eye does not move with downward gaze or right lateral gaze. Which of the following
diagnostic tests will provide the most accurate information regarding this injury?
A. Facial x-ray
B. Schiotz tonometry
C. Fluorescein angiography
D. Slit lamp biomicroscopy
Explanations
(c) A. X-ray films may show a blow-out fracture of the orbital floor. Such fractures may lead to oculomotor nerve
entrapment or may lead to swelling that impinges on the nerve, causing decreased eye movements.
(u) B. Schiotz tonometry is used to evaluate intraocular pressures; elevated pressure indicates glaucoma.
(u) C. Fluorescein angiography is used to evaluate retinopathies.
(u) D. Slit lamp biomicroscopy is used to evaluate lid contour, lesions, lashes, conjunctiva, sclera, cornea, anterior
chamber, and lens.
Ref: (29)



10. Clinical Intervention/Hematology
A 68 year-old male with a history of atrial fibrillation treated with warfarin (Coumadin) presents to the emergency
department after vomiting large amounts of bright red blood. INR is 3. Which of the following is most appropriate to
rapidly lower the patient's INR?
A. Discontinue warfarin
B. Administer fresh frozen plasma
C. Administer protamine sulfate
D. Administer heparin sulphate
Explanations
(u) A. This will not immediately reverse the effects of warfarin.
(c) B. Fresh frozen plasma is the most rapid way to lower the patient's INR.
(u) C. Protamine sulfate is used to neutralize heparin sulphate not warfarin.
(h) D. Heparin administration would increase the bleeding and be harmful.
Ref: (28)



11. Clinical Therapeutics/Cardiology
A 48 year-old male with diabetes mellitus presents for routine physical examination. Of note his blood pressure each
of his last two follow-up visits was 150/90 mmHg. Today the patient's BP is 148/88 mmHg. The patient denies
complaints of chest pain, change in vision, or headache. Which of the following is the most appropriate management
for this patient?
A. Atenolol (Tenormin)
B. Nifedipine (Procardia)

Copyright 2010. Physician Assistant Education Association

12
C. Hydralazine (Apresoline)
D. Lisinopril (Zestril)
Explanations
(u) A. See D for explanation.
(u) B. See D for explanation.
(u) C. See D for explanation.
(c) D. ACE inhibitors are the first line treatment of choice in a patient with hypertension and diabetes.
Ref: (28)



12. History & Physical/Dermatology
A 72 year-old farmer comes to the office for evaluation of a pearly ulcerated papule on his right nostril. The papule
has been bleeding off and on for the past couple weeks. Which of the following findings would be most concerning on
the physical examination of this patient?
A. Cherry angioma
B. Telangiectasia
C. Spider angioma
D. Pyogenic granuloma
Explanations
(u) A. A cherry angioma is a common, violaceous vascular lesion usually located on the trunk.
(c) B. Telangiectatic vessels are often visible skin findings with basal cell carcinoma.
(u) C. A spider angioma is a benign red papule with radiating telangiectasia usually located on the face, forearms
and hands.
(u) D. A pyogenic granuloma is a solitary vascular nodule that develops rapidly following minor trauma.
Ref: (10)



13. Clinical Intervention/Gastrointestinal/Nutritional
A 24 year-old female presents with complaints of abdominal pain, bloating, and chronic diarrhea. She says she felt
worse last month while on vacation in Italy. Despite eating well she lost weight. What would be the most appropriate
treatment for this patient?
A. Gluten-free diet
B. Small bowel resection
C. Ciprofloxacin (Cipro) therapy
D. Pancreatic enzymes
Explanations
(c) A. Celiac disease is a diffuse disease of the small bowel that is caused by immunologic malfunction. This
disease is active only in the presence of gluten, a constituent of wheat. Avoidance of gluten-containing foods
is the treatment of choice.
(u) B. Although celiac disease affects the small bowel (particularly the jejunum), resection is too aggressive when
dietary manipulation is very effective in the management of this disorder.
(u) C. Celiac disease is immune-mediated and is not due to infection so antibiotics are not needed.
(u) D. Celiac disease affects absorption in the small bowel and does not involve a dysfunction of pancreatic
enzymes.
Ref: (28)

Copyright 2010. Physician Assistant Education Association

13
14. History & Physical/Endocrinology
Hypoglycemia in an elderly patient with diabetes is most likely to manifest as which of the following symptoms?
A. Tachycardia
B. Diaphoresis
C. Intense hunger
D. Mental confusion
Explanations
(u) A. Autonomic responses such as tachycardia, diaphoresis, and hunger are often blunted in the elderly diabetic.
(u) B. See A for explanation.
(u) C. See A for explanation.
(c) D. Manifestations of hypoglycemia in the elderly are mainly from impaired central nervous system function.
Manifestations include mental confusion, bizarre behavior, and ultimately coma.
Ref: (1)



15. Clinical Therapeutics/Pulmonology
An 85 year-old nursing home resident presents with abrupt onset of cough, sore throat, headache, myalgias, and
malaise. On examination the patient's temperature is 102 degrees F; the rest of the exam is unremarkable. Nasal
smear is positive for Influenza B. Which of the following is the treatment of choice in this patient?
A. Amantadine (Symmetrel)
B. Oseltamivir (Tamiflu)
C. Acyclovir (Zovirax)
D. Nevirapine (Viramune)
Explanations
(u) A. Amantadine is only used to treat influenza A.
(c) B. Oseltamivir is used to treat both influenza A and B.
(u) C. Acyclovir is used to treat viral infections due to certain herpes viruses.
(u) D. Nevirapine is used to treat infection due to HIV.
Ref: (28)



16. Health Maintenance/Cardiology
A 36 year-old female presents for a routine physical. She has no current complaints and her only medication is oral
contraceptives. The patient is preparing for a trip to Australia and is worried about the long flight as her mom has a
history of deep vein thrombosis after a long trip several years ago. Physical examination reveals BP 110/60 mmHg,
HR 66 bpm, regular. Heart is regular rate and rhythm without murmur, lungs are clear to auscultation bilaterally and
extremities are without edema. Which of the following is the most appropriate recommendation for your patient?
A. Discontinue oral contraceptives
B. Recommend walking frequently during the flight
C. Begin daily aspirin therapy
D. Increase fluid intake 2-3 days prior to the flight
Explanations
(u) A. See B for explanation.
(c) B. The risk of deep vein thrombosis after air travel increases with flight duration. Preventive measures for
patients include using support hose and performing in-flight exercises and walking.
(u) C. See B for explanation.
(u) D. See B for explanation.
Ref: (28)

Copyright 2010. Physician Assistant Education Association

14
17. Diagnosis/Dermatology
A 43 year-old woman presents with episodes of facial flushing with increased skin temperature followed by the
development of tiny papules and pustules primarily on her cheeks and chin. These symptoms worsen when she eats
spicy foods. On examination, telangiectasia are noted along the nasal folds and tiny pustules on both cheeks. Which
of the following is the most likely diagnosis?
A. Atopic dermatitis
B. Rosacea
C. Lupus
D. Acne vulgaris
Explanations
(u) A. Atopic dermatitis is characterized by chronic dry skin, and pruritic inflammatory lesions of the flexor surfaces of
the body. Repeated rubbing or scratching of the skin causes excoriations and lichenification. Atopic dermatitis
is often associated with a personal or family history of allergic rhinitis or asthma.
(c) B. Rosacea is chronic, episodic flushing of the face followed by the development of tiny papules and pustules.
Eating spicy foods or drinking hot liquids or alcohol can cause exacerbation of the symptoms.
(u) C. The rash associated with lupus is commonly found on the cheeks and not on the chin. There are not typically
pustules or telangiectasia noted.
(u) D. Acne vulgaris does not present with facial flushing and would not be triggered by the consumption of spicy
foods.
Ref: (10)


18. Clinical Intervention/Infectious Diseases
A 25 year-old male presents to the emergency department for evaluation of a wound on his hand. He cut his hand
while tearing down a chicken coop. On examination of his right hand you note a dirty 3cm jagged laceration. The
patient is unaware of his tetanus immunization status. Besides cleaning and debriding the wound, what is the
recommended clinical intervention in this patient?
A. Administer tetanus-diptheria toxoid (Td) and tetanus immune globulin (TIG)
B. Administer diphtheria, tetanus, and pertussis vaccine (DTP)
C. Administer tetanus toxoid vaccine (TT) and tetanus immune globulin (TIG)
D. No tetanus immunization or tetanus immune globulin is necessary in this patient
Explanations
(c) A. Td or Tdap with TIG is recommended as tetanus prophylaxis in a patient with a contaminated wound and
unknown tetanus vaccination status.
(u) B. See A for explanation.
(u) C. See A for explanation.
(u) D. See A for explanation.
Ref: (7)


19. Diagnostic Studies/Obstetrics/Gynecology
A 25 year-old female presents for a routine gynecological examination. You palpate a 2 cm breast mass in her right
breast. Her menstrual period was last week. She has no family history of breast cancer. What is the modality of
choice to further evaluate her breast mass?
A. Magnetic resonance imaging (MRI)
B. Excisional biopsy
C. Ultrasound
D. Mammography
Explanations
(u) A. MRI's of the breast are done in patients with prior breast cancer or who have BRCA 1 or 2 positivity.
(u) B. Excisional biopsy is not indicated without differentiation of lesion by ultrasound.
(c) C. In a 25 year-old an ultrasound is the best choice because of the density of the breast tissue in young
women.
(u) D. Mammographies are not recommended in women prior to the age of 35 without family history of breast
cancer or BRCA positive.
Ref: (4)

Copyright 2010. Physician Assistant Education Association

15
20. Diagnosis/Orthopedics/Rheumatology
A 60 year-old right hand dominant male presents to your office complaining of right shoulder pain and progressively
worsening arm weakness. His symptoms are aggravated when working above the shoulder level. On physical
examination, there is no evidence of muscle atrophy. He has strong (5/5) adduction of his right shoulder but
considerable weakness (1/5) with abduction when raising his arm above his head. Passive range of motion is intact.
Which of the following is the most likely diagnosis?
A. Rotator cuff tear
B. Subacromial bursitis
C. Adhesive capsulitis
D. Supraspinatus tendonitis
Explanations
(c) A. Rotator cuff tears are associated with full passive range of motion but have pain and weakness during active
abduction.
(u) B. Subacromial bursitis causes pain with raising hand above head and has mild degree of weakness.
(u) C. Adhesive capsulitis (frozen shoulder) causes significant reduction in both active and passive range of motion.
(u) D. Of the four rotator cuff muscles, supraspinatus is most likely to strain causing tendonitis. The symptoms
cause limited range of motion due to pain but minimal weakness.
Ref: (26)



21. Health Maintenance/Gastrointestinal/Nutritional
A newborn weighs 8 pounds at birth. Following established growth indices, what would you expect the infant to weigh
at 1 year of age?
A. 16 pounds
B. 24 pounds
C. 32 pounds
D. 40 pounds
Explanations
(u) A. See B for explanation.
(c) B. Most infants should be expected to triple their birth weight by 1 year of age.
(u) C. See B for explanation.
(u) D. See B for explanation
Ref: (13)



22. Diagnostic Studies/Cardiology
A 3 month-old female presents with her mom for physical examination. The patient's mom denies any complaints. On
examination you note a well-developed, well-nourished infant in no apparent distress. There is no cyanosis noted.
Heart examination reveals a normal S1 with a physiologically split S2. There is a grade III/VI high-pitched, harsh,
pansystolic murmur heard best at the 3rd and 4th left intercostal spaces with radiation across the precordium. Which
of the following is the initial diagnostic study of choice in this patient?
A. CT angiogram
B. Electrocardiogram
C. Echocardiogram
D. Cardiac catheterization
Explanations
(u) A. This patient has signs and symptoms consistent with a ventricular septal defect (VSD). CT angiogram and
electrocardiogram are not indicated in establishing the diagnosis of a VSD.
(u) B. See A for explanation.
(c) C. Echocardiogram is the initial diagnostic study of choice in the diagnosis of a VSD.
(u) D. Cardiac catheterization may be necessary to accurately measure pulmonary pressures or if a VSD can not
be well localized on echocardiogram, but it is not the initial diagnostic study of choice.
Ref: (5)

Copyright 2010. Physician Assistant Education Association

16

23. Scientific Concepts/Pulmonology
A 62 year-old male smoker presents to the clinic with the complaint of a chronic cough, hemoptysis, and weight loss.
Chest CT shows a mass obstructing the bronchus with hilar and mediastinal lymph node abnormalities.
Bronchoscopy with biopsy is performed. On reviewing pathology results you explain to the patient that his type of lung
cancer is prone to early hematogenous spread, is rarely amenable to surgical resection and has a very aggressive
course. What type of lung cancer is most likely in this patient?
A. Squamous cell
B. Small cell
C. Large cell
D. Adenocarcinoma
Explanations
(u) A. Squamous cell carcinoma, large cell carcinoma, and adenocarcinoma spread more slowly and have the
possibility of cure in early stages following resection and chemotherapy.
(c) B. Small cell lung cancer is very aggressive with a median survival (untreated) of 6-18 weeks.
(u) C. See A for explanation.
(u) D. See A for explanation.
Ref: (28)




24. History & Physical/ENT/Ophthalmology
Otitis externa can be differentiated from uncomplicated otitis media by which of the following physical examination
findings?
A. Erythematous tympanic membrane
B. Mastoid tenderness
C. Normal pneumatic otoscopy
D. Posterior auricular adenopathy
Explanations
(u) A. In otitis externa, the erythema is because the lateral surface of the tympanic membrane is ear canal skin,
and therefore is often erythematous.
(u) B. Mastoid tenderness is not directly associated with otitis media or otitis externa.
(c) C. In contrast to acute otitis media, the tympanic membrane moves normally with pneumatic otoscopy in a
patient with otitis externa.
(u) D. Both otitis media and otitis externa may have posterior auricular adenopathy on examination.
Ref: (28)



25. Clinical Intervention/Cardiology
A 20 year-old male presents with complaint of brief episodes of rapid heart beat with a sudden onset and offset that
have increased in frequency. He admits to associated shortness of breath and lightheadedness. He denies syncope.
Electrocardiogram reveals a delta wave prominent in lead II. Which of the following is the most appropriate long-term
management in this patient?
A. Implantable cardio defibrillator
B. Radiofrequency ablation
C. Verapamil (Calan)
D. Metoprolol (Lopressor)
Explanations
(u) A. Implantable cardio defibrillators are indicated in the treatment of ventricular arrhythmias, not Wolf-Parkinson-
White (WPW) syndrome.
(c) B. Radiofrequency ablation is the procedure of choice for long-term management in patients with accessory
pathways (WPW) and recurrent symptoms.
(u) C. Calcium channel blockers and beta-blockers are not the best options for the long-term management of WPW.
They may decrease the refractoriness of the accessory pathway or increase the refractoriness of the AV node

Copyright 2010. Physician Assistant Education Association

17
in patients with atrial fibrillation or atrial flutter who have an antegrade conducting bypass tract. This may lead
to faster ventricular rates.
(u) D. See C for explanation.
Ref: (28)


26. Diagnosis/Obstetrics/Gynecology
A 26 year-old woman requests screening after her boyfriend was treated for a sexually transmitted infection recently.
On examination you find a painless vulvar ulcer. Which of the following is the most likely diagnosis?
A. Herpes
B. Syphilis
C. Chancroid
D. Granuloma inguinale
Explanations
(u) A. The classic presentation of herpes is a painful vesicle.
(c) B. The primary lesion of syphilis presents as a painless ulcer or chancre. Secondary syphilis presents with a
skin rash lymphadenopathy and mucocutaneous lesions.
(u) C. Chancroid presents with a painful genital ulcer and tender suppurative inguinal adenopathy.
(u) D. Granuloma inguinale presents with raised, red lesions that bleed easily.
Ref: (4)

27. Clinical Therapeutics/Orthopedics/Rheumatology
A 57 year-old male recently on a high protein diet presents with an exquisitely tender, erythematous, warm right great
toe. Which of the following is the treatment of choice for this patient?
A. Corticosteroids
B. Colchicine
C. Allopurinol
D. Non-steroidal antiinflammatory agents
Explanations
(u) A. Corticosteroids are effective in acute gout attacks but are reserved for people with non-steroidal anti-
inflammatory agent allergies or contraindications.
(u) B. Colchicine is not recommended in the treatment of acute gouty arthritis.
(u) C. Allopurinol is useful in reducing uric acid levels but is not the treatment of choice in acute gouty arthritis.
(c) D. NSAID medications are the treatment of choice for acute gouty attacks.
Ref: (28)



28. Diagnosis/Endocrinology
A 55 year-old female presents to the clinic with lethargy, fatigue, constipation, and menorrhagia. Physical
examination reveals an enlarged thyroid, dry skin, and a heart rate of 50 bpm. Laboratory results show a decrease in
free T4, and an elevation in TSH. Which of the following is the most likely diagnosis?
A. Secondary hypothyroidism
B. Primary hypothyroidism
C. Primary hyperthyroidism
D. Secondary hyperthyroidism
Explanations
(u) A. See B for explanation.
(c) B. This is a classic presentation of symptoms of primary hypothyroidism. Symptoms include weight gain,
fatigue, lethargy, depression, weakness, constipation, menorrhagia; and patients often present with a
palpable, enlarged thyroid.
(u) C. See B for explanation.
(u) D. See B for explanation.
Ref: (28)

Copyright 2010. Physician Assistant Education Association

18
29. Diagnostic Studies/Gastrointestinal/Nutritional
Which of the following diagnostic studies is indicated in the evaluation of an upper gastrointestinal bleed?
A. Esophageal manometry
B. Bleeding scan
C. Upper endoscopy
D. Barium swallow
Explanations
(u) A. Esophageal manometry is not indicated in the evaluation of upper gastrointestinal bleeding.
(u) B. Bleeding scans are most useful to evaluate occult GI bleeding. Radioactive isotope tracing is useful to
concentrate bleeding sites at a single location. It is most useful to find bleeding that is occurring in the large or
small bowel rather than in the upper GI sites.
(c) C. Endoscopy is the evaluation modality of choice in patients with upper GI bleeding. The advantage of this
technique is that it can be used for both diagnostic and therapeutic purposes.
(u) D. Barium swallow is not indicated in the evaluation of upper gastrointestinal bleeding.
Ref: (28)



30. Clinical Intervention/Pulmonology
A 45 year-old male presents to the clinic complaining of morning sluggishness, daytime fatigue, headaches. He
admits to drinking two cocktails each evening. His bed partner reports his loud cyclical snoring, breath cessation and
thrashing movements of his extremities during sleep. BMI is 40. Heart examination reveals regular rate and rhythm
without S3, S4, or murmur and lungs are clear to auscultation. Polysomnography shows apneic episodes lasting as
long as 60 seconds. Which of the following clinical interventions would most likely provide for the acute cessation of
apneic episodes?
A. Avoidance of alcohol
B. Weight loss
C. Nasal continuous positive airway pressure
D. Supplemental oxygen
Explanations
(u) A. Avoidance of alcohol is also a necessary step in managing sleep apnea however does not provide
immediate relief of apneic episodes.
(u) B. Weight loss is also a necessary step in managing sleep apnea however does not provide immediate relief of
apneic episodes.
(c) C. Nasal CPAP is curative in many patients with sleep apnea.
(u) D. Oxygen lessens the severity of nocturnal desaturation but may lengthen the apnea episodes.
Ref: (28)



31. Diagnosis/Psychiatry/Behavioral Medicine
A 45 year-old female presents with her daughter who is concerned because her mom will not leave the house. The
patient describes feeling anxious in crowds and places where there are no easy means of escape. Which of the
following is the most likely diagnosis in this patient?
A. Agoraphobia
B. Generalized anxiety disorder
C. Obsessive-compulsive disorder
D. Panic attack
Explanations
(c) A. Though all of the anxiety disorders can have comorbid agoraphobia the case describes only those symptoms
inherent to this primary diagnosis. Agoraphobia can occur with generalized anxiety disorder and a panic
attack can occur as a result of agoraphobia.
(u) B. See A for explanation.
(u) C. See A for explanation.

Copyright 2010. Physician Assistant Education Association

19
(u) D. See A for explanation.
Ref: (33)



32. Scientific Concepts/Urology/Renal
Which of the following is a cause of acute kidney failure due to prerenal azotemia?
A. Excessive diuresis
B. Urinary tract obstruction
C. Radiologic contract media
D. Aminoglycosides
Explanations
(c) A. Prerenal azotemia is due to renal hypoperfusion which can occur with intravascular volume depletion such
as excessive diuresis, hemorrhage, and gastrointestinal losses.
(u) B. Postrenal azotemia is due to obstruction of urinary flow from both of the kidneys.
(u) C. Radiologic contrast media can be directly nephrotoxic causing acute tubular necrosis, which is an intrinsic
renal disease.
(u) D. Exogenous nephrotoxins, such as aminoglycosides, cause acute tubular necrosis, which is an intrinsic renal
disease.
Ref: (28)



33. History & Physical/Pulmonology
A 19 year-old male presents to the ED complaining of a sudden onset of dyspnea and left sided chest pain. He
denies fever, chills, cough or sore throat. General survey shows that he is 6 feet 2 inches tall and weighs 135 lbs.
Vital signs are BP 86/60 mmHg, HR 130 bpm, RR 28, temp. 98.6 degrees F. Which of the following would you likely
find on examination of his thorax?
A. Left-sided hyperresonance
B. Increased tactile fremitus of left base
C. Scattered rales throughout
D. Increased anterior/posterior diameter
Explanations
(c) A. Hyperresonance is present with accumulation of air in the pleural space.
(u) B. Increased tactile fremitus is present with lung consolidation.
(u) C. Mycoplasma pneumonia, common in young adults, presents with scattered rales audible throughout.
(u) D. Increased A/P diameter is common in older patients with history of COPD.
Ref: (28)



34. Diagnosis/Cardiology
A 60 year-old female recently discharged after an 8 day hospital stay for pneumonia presents complaining of pain
and redness in her right arm. The patient thinks this was the area where her IV was placed. The patient denies fever
or chills. Examination of the area reveals localized induration, erythema and tenderness. There is no edema or
streaking noted. Which of the following is the most likely diagnosis?
A. Acute thromboembolism
B. Thrombophlebitis
C. Cellulitis
D. Lymphangitis
Explanations
(u) A. Acute thromboembolism is usually associated with edema of the extremity and warm temperature.
Thromboembolism is unusual after thrombophlebitis.
(c) B. This patient's signs and symptoms are consistent with thrombophlebitis. Short-term venous catheterization of
a superficial arm vein is commonly the cause and thrombophlebitis characterized by dull pain, induration,

Copyright 2010. Physician Assistant Education Association

20
redness and tenderness along the course of the vein.
(u) C. Cellulitis is usually associated with fever, increased warmth over the affected area and associated edema.
(u) D. Lymphangitis is associated with fever, malaise, chills, and streaking.
Ref: (28)

35. Health Maintenance/Orthopedics/Rheumatology
A 57 year-old male with history of hypertension presents with acute gouty arthritis. Which of the following anti-
hypertensive classes should be avoided in this patient?
A. Thiazide diuretics
B. Beta-blockers
C. ACE inhibitors
D. Calcium channel blockers
Explanations
(c) A. Thiazide diuretics are common causes of increased urate levels causing gout attacks.
(u) B. See A for explanation.
(u) C. See A for explanation.
(u) D. See A for explanation.
Ref: (28)


36. Diagnosis/Endocrinology
A 72 year-old patient with type 2 diabetes treated with a sulfonylurea is brought into the clinic by her daughter, who
provides the patient's history. The patient has had a recent upper respiratory infection and excessive diuresis for the
past 2 days. She has had decreased oral intake. At the clinic, she becomes increasingly stuporous and lethargic.
Which of the following is the most likely diagnosis in this patient?
A. Diabetic ketoacidosis
B. Hyperglycemic hyperosmolar state
C. Hypoglycemic coma secondary to her refusal to eat
D. Lactic acidosis secondary to an infectious process
Explanations
(u) A. Diabetic ketoacidosis is usually preceded by a day or more of polyuria and polydipsia in association with
nausea and vomiting in someone receiving insulin.
(c) B. Hyperglycemic hyperosmolar state is most common in an elderly patient with mild diabetes. Lethargy and
confusion develop as osmolality rises to 300 mosm/kg or higher.
(u) C. Although hypoglycemia induced by oral glucose lowering agents is less common, it tends to occur in
elderly patients with impaired renal function and is generally associated with longer acting sulfonylureas.
(u) D. Lactic acidosis secondary to an infectious process is typically associated with a severe infectious process,
which is not definite in this patient.
Ref: (28)


37. History & Physical/Gastrointestinal/Nutritional
A 68 year-old male presents with jaundice, weight loss, and boring abdominal pain which radiates to the back. The
gallbladder is palpable on physical examination. This finding is most consistent with which of the following?

A. Pancreatic tumor
B. Hemorrhagic pancreatitis
C. Cholecystitis
D. Cholelithiasis
Explanations
(c) A. A large palpable gallbladder resulting from pressure from a tumor in the pancreatic head is known as
Courvoisier's sign.
(u) B. Ecchymosis of the flank associated with hemorrhagic pancreatitis is known as Grey Turner's sign

Copyright 2010. Physician Assistant Education Association

21
(u) C. A sharp increase in tenderness with a sudden stop in inspiratory effort indicative of acute cholecystitis is
known as Murphy's sign.
(u) D. Pain in the right lower quadrant during left-sided pressure on the abdomen suggesting acute appendicitis is
known as Rovsing's sign.
Ref: (11)


38. Scientific Concepts/Dermatology
A 66 year-old woman with type 2 diabetes comes to the office because she has had a painful red patch that has been
spreading on her right lower leg over the past 3 days. Her temperature is 100.4 degrees F. Physical examination of
her right lower leg reveals a 3 cm tender, warm, erythematous, and edematous plaque. Which of the following
pathogens is most likely causing this patient's symptoms?
A. Staphylococcus aureus
B. Haemophilus influenzae
C. Proteus mirabilis
D. Escherchia coli
Explanations
(c) A. S. aureus and group A beta-hemolytic streptococci are the most common causes of cellulitis.
(u) B. Haemophilus influenzae can cause facial cellulitis in young children.
(u) C. Proteus can be one of the pathogens that cause infectious gangrene.
(u) D. Escherchia coli can cause crepitant cellulitis. It would not be the most common cause of cellulitis in this
patient.
Ref: (10)


39. History & Physical/Cardiology
A patient with a history of chronic venous insufficiency presents for routine follow-up. Which of the following findings
is most likely on physical examination?
A. Cold lower extremities
B. Diminished pulses
C. Lower extremity edema
D. Palpable cord
Explanations
(u) A. Cold lower extremities are more commonly seen in peripheral arterial, not venous, disease.
(u) B. Diminished pulses are seen in peripheral arterial disease.
(c) C. Patients with chronic venous insufficiency will commonly have lower extremity edema.
(u) D. A palpable cord is more common in superficial thrombophlebitis.
Ref: (3)



40. Clinical Therapeutics/Obstetrics/Gynecology
A 16 year-old nulliparous acutely ill female presents with bilateral lower abdominal pain. She has a temperature of
100.4 degrees F and on examination has a tender, enlarged left adnexa. Cervical culture is positive for Chlamydia.
Ultrasound reveals a complex tubular structure in the left adnexal area. What is the recommended treatment?
A. Outpatient treatment with IM ceftriaxone and oral doxycycline
B. Oral doxycycline
C. IM procaine penicillin
D. Hospitalization with parenteral doxycycline and cefoxitin
Explanations
(u) A. See D for explanation.
(u) B. See D for explanation.

Copyright 2010. Physician Assistant Education Association

22
(u) C. See D for explanation.
(c) D. This patient has pelvic inflammatory disease and most likely a tubo-ovarian abscess. It is recommended that
the patient be hospitalized and treated with high-dose IV antibiotic therapy. For patients with tubo-ovarian
abscesses, surgical drainage is often necessary.
Ref: (4)



41. Diagnostic Studies/Pulmonology
A 2 year-old presents with sudden onset of cough and stridor. On examination the child is afebrile and appears non-
toxic with a respiratory rate of 42 breaths per minute. What is the next step in the evaluation of this patient?
A. Lateral soft tissue x-ray of the neck
B. Indirect laryngoscopy
C. Finger sweep
D. Chest x-ray
Explanations
(u) A. See D for explanation.
(u) B. See D for explanation.
(u) C. See D for explanation.
(c) D. Chest x-ray should be done first when foreign body aspiration is suspected.
Ref: (13)




42. Scientific Concepts/ENT/Ophthalmology
A 3 year-old patient is brought in with a 10 day history of clear nasal drainage and cough which has now developed
into otalgia and fever exceeding 101 degrees F for the last 5 days. Mom denies other chronic medical problems. The
patient has had similar complaints three times in her life. Which of the following is the most likely causative organism?
A. Mycoplasma pneumoniae
B. Pneumocystis jiroveci
C. Pseudomonas aeruginosa
D. Streptococcus pneumoniae
Explanations
(u) A. Mycoplasma pneumoniae is a common cause of bronchitis and pneumonia.
(u) B. Pneumocystis jiroveci is associated with immunocompromised respiratory illness.
(u) C. Pseudomonas aeruginosa would most commonly cause otitis externa.
(c) D. Acute otitis media is a bacterial infection of the mucous lined air-containing spaces of the temporal bone most
commonly caused by Streptococcus pneumoniae, Haemophilus influenzae, and Streptococcus pyogenes.
Ref: (28)


43. Clinical Therapeutics/Neurology
A 7 year-old is evaluated for episodes of unresponsiveness in which she stares, blinking into space. Episodes last
only 10-20 seconds but may happen several times daily. There are no abnormal movements, sensory loss or
headaches reported. Which of the following is considered first-line therapy for this disorder?
A. Carbamazepine (Tegretol)
B. Ethosuximide (Zarontin)
C. Gabepentin (Neurontin)
D. Topiramate (Topamax)
Explanations
(u) A. See B for explanation.
(c) B. Absence seizures are usually treated successfully with Ethosuximide. The other agents listed are not
indicated and may actually worsen this type of seizure.

Copyright 2010. Physician Assistant Education Association

23
(u) C. See B for explanation.
(u) D. See B for explanation.
Ref: (13)



44. Health Maintenance/Gastrointestinal/Nutritional
Which of the following treatments is used to reduce the hepatic complications of hemochromatosis?
A. Penicillamine
B. Paracentesis
C. Albumin
D. Phlebotomy
Explanations
(u) A. Penicillamine is a chelating agent that may be used in the treatment of Wilson's disease in which there is a
deficiency of the copper-binding protein ceruloplasmin resulting in impaired copper excretion into the bile.
(u) B. Paracentesis may be performed in patients with tense ascites which can symptomatically improve the
patient but does not alter long-term prognosis.
(u) C. Patients with long-standing liver failure have decreased albumin levels but this is not a treatment normally
performed for hemochromatosis.
(c) D. Hemochromatosis is an autosomal recessive inherited disorder that causes cirrhosis, diabetes, and bronze
pigmentation to the skin due to the abnormal accumulation of iron in tissues. Intensive phlebotomy is the
treatment of choice until the iron overload is corrected.
Ref: (7)



45. Diagnostic Studies/Cardiology
A 29 year-old female with history of IV drug abuse presents with ongoing fevers for three weeks. She complains of
fatigue, worsening dyspnea on exertion and arthralgias. Physical examination reveals a BP of 130/60 mmHg, HR 90
bpm, regular, RR 18, unlabored. Petechiae are noted beneath her fingernails. Fundoscopic examination reveals
exudative lesions in the retina. Heart examination shows regular rate and rhythm, there is a grade II-III/VI systolic
murmur noted, with no S3 or S4. Lungs are clear to auscultation bilaterally, and the extremities are without edema.
Which of the following is the diagnostic study of choice in this patient?
A. Electrocardiogram
B. CT angiogram of the chest
C. Cardiac catheterization
D. Transesophageal echocardiogram
Explanations
(u) A. See D for explanation.
(u) B. See D for explanation.
(u) C. See D for explanation.
(c) D. This patient's signs and symptoms are consistent with infective endocarditis. The diagnostic study of choice
would be a transesophageal echocardiogram.
Ref: (28)



46. Clinical Therapeutics/Endocrinology
An 18 year-old female with diabetes presents to the emergency department with altered level of consciousness, deep
breathing and fruity odor to her breath. Which of the following medications is indicated for this patient?
A. NPH insulin
B. Metformin (Glucophage)
C. Regular insulin
D. Glyburide (Diabeta)

Copyright 2010. Physician Assistant Education Association

24
Explanations
(h) A. NPH insulin has a slower onset of action than regular insulin, and should not be administered intravenously.
Onset is 2-4 hours and in a patient with diabetic ketoacidosis the onset needs to be rapid.
(h) B. Metformin and glyburide are oral hypoglycemics that have no role in the emergency treatment of diabetic
ketoacidosis.
(c) C. Regular insulin has a rapid onset of action when given intravenously and is the initial choice in patients with
diabetic ketoacidosis. Following the initial dose, a continuous infusion often promotes a steady, slow fall of
glucose levels to normal, which can then be stabilized by decreasing the insulin.
(h) D. See B for explanation.
Ref: (28)



47. Diagnosis/Pulmonology
A 69 year-old male with a history of chronic lymphocytic leukemia presents to the clinic complaining of cough,
dyspnea and production of copious amounts of foul smelling sputum. Physical examination reveals crackles at the
lung bases. Chest x-ray shows dilated and thickened bronchi that appear as ring-like markings. Which of the following
is the most likely diagnosis?
A. Bronchiectasis
B. Tuberculosis
C. Adenocarcinoma
D. Pulmonary fibrosis
Explanations
(c) A. This patient has signs and symptoms consistent with bronchiectasis including CXR findings of dilated and
thickened bronchi that may appear as tram-tracks or as ring-like markings.
(u) B. TB would present with CXR findings in the apical or posterior segments of the upper lobes.
(u) C. Radiographic findings of adenocarcinoma include enlarged nodule or mass; persistent opacity, atelectasis or
pleural effusion. The sputum would not likely be foul smelling.
(u) D. Pulmonary fibrosis does not present with dilated bronchi or ring-like markings on CXR.
Ref: (28)

48. History & Physical/Orthopedics/Rheumatology
Bony and cartilaginous enlargement of distal interphalangeal joints is commonly seen in which of the following
medical conditions?
A. Rheumatoid arthritis
B. Osteoarthritis
C. Psoriatic arthritis
D. Gouty arthritis
Explanations
(u) A. Findings in Rheumatoid arthritis include skin nodules, pannus formations and symmetric joint involvement
sparing the distal interphalangeal joints.
(c) B. Heberden's nodes are commonly seen in primary osteoarthritis.
(u) C. Psoriatic arthritis manifests commonly as sacroiliitis. Asymmetrical arthritis occurs in psoriatic arthritis and
causes a sausage like appearance to the fingers. It can also mimic the presentation of rheumatoid arthritis.
(u) D. Gout commonly affects the great toe, midfoot, ankle, and knee. Tophi may be present which are subcutaneal
deposits of monosodium urate crystals.
Ref: (26)



49. Diagnosis/Psychiatry/Behavioral Medicine
A 9 year-old male is brought in by his mother who reports the patient has exhibited an extremely negative attitude for
the past year. He seems angry much of the time and frequently loses his temper. Arguing over even trivial details is
common place and he seems to take delight in annoying his family. His grades and conduct at school remain
excellent. He has few friends, though he has never been seen bullying or destroying others' property. What is the

Copyright 2010. Physician Assistant Education Association

25
most likely diagnosis?
A. Attention deficit disorder
B. Conduct disorder
C. Antisocial personality disorder
D. Oppositional defiant disorder
Explanations
(u) A. See D for explanation.
(u) B. See D for explanation.
(u) C. See D for explanation.
(c) D. Oppositional defiant disorder (ODD) best fits this scenario and is differentiated from conduct disorder by the
lack of bullying and the lack of destruction of property. Many children with ODD do drift into conduct disorders
over time. His good grades and conduct at school lessen the probability of untreated ADD. Personality
disorders (i.e. antisocial personality disorder) can not be diagnosed at this early an age.
Ref: (33)



50. Health Maintenance/ENT/Ophthalmology
A patient presents complaining of gradual hearing loss over the past 3 months. He admits to use of Q-tips and
otherwise does not wear ear plugs or place other foreign objects in his ear. On examination external auditory canals
are obstructed with cerumen. After removal of cerumen, hearing is equal on both sides. Appropriate counseling of this
patient includes which of the following?
A. Advise him to discontinue use of cotton swabs
B. Encourage jet irrigator (i.e. WaterPik) to clean ears
C. Instruct in ear irrigation with cold water
D. Refer to dermatologist
Explanations
(c) A. In most people, the ear canal is self-cleansing. In most cases, cerumen impaction is self-induced through ill-
advised attempts at cleaning the ear.
(u) B. Use of jet irrigators designed for cleaning teeth (i.e. waterPik) for wax removal should be avoided since they
may result in tympanic membrane perforations.
(u) C. Irrigation is performed with water at body temperature to avoid a vestibular caloric response.
(u) D. See A for explanation.
Ref: (28)


51. Scientific Concepts/Obstetrics/Gynecology
Dysmenorrhea would most likely occur in which of the following patients?
A. A young teenager who just started having her menses
B. A woman on birth control pills
C. A marathon runner with one menses per year
D. A 35 year-old woman with regular cycles
Explanations
(u) A. Young teenagers who have just started their menses are unlikely to have dysmenorrhea, because they
usually are anovulatory for up to one year.
(u) B. Women on birth control pills do not have dysmenorrhea because they do not ovulate on oral contraceptives
and OCPs are used to treat dysmenorrhea.
(u) C. Marathon runners are often amenorrheic or have oligoamenorrhea. They do not have dysmenorrhea because
dysmenorrhea is a function of ovulatory cycles.
(c) D. Women with regular menstrual cycles are most likely to have dysmenorrhea. Dysmenorrhea is caused by a
excess of prostaglandin F 2 alpha. Prostaglandin production increases under the influence of progesterone,
reaching a peak at, or soon after, the start of menstruation.
Ref: (4)


Copyright 2010. Physician Assistant Education Association

26
52. Clinical Therapeutics/Cardiology
A 49 year-old female presents complaining of several episodes of chest pain recently. Initial ECG in the emergency
department shows no acute changes. Two hours later, while the patient was having pain, repeat electrocardiogram
revealed ST segment elevation in leads II, III, and AVF. Cardiac catheterization shows no significant obstruction of
the coronary arteries. Which of the following is the treatment of choice in this patient?
A. Nifedipine (Procardia)
B. Metoprolol (Lopressor)
C. Lisinopril (Zestril)
D. Carvedilol (Coreg)
Explanations
(c) A. This patient is most likely having coronary artery spasm. This can be treated prophylactically with calcium
channel blockers such as nifedipine.
(h) B. Beta-blockers may exacerbate the symptoms of coronary vasospasm.
(u) C. ACE inhibitors are not effective in the treatment or prevention of coronary vasospasm.
(u) D. Carvedilol is not effective in the treatment or prevention of coronary vasospasm.
Ref: (28)



53. Diagnostic Studies/Gastrointestinal/Nutritional
A 62 year-old male presents with complaints of dyspepsia, early satiety, and dysphagia. What diagnostic study would
be indicated in his initial workup?
A. PET CT scan
B. Abdominal ultrasound
C. Gastric emptying studies
D. Upper endoscopy
Explanations
(u) A. PET CT scan is used primarily to follow the course of an already diagnosed cancer.
(u) B. Abdominal ultrasonography is performed only when pancreatic or biliary disease is suspected.
(u) C. Gastric emptying studies are valuable only in patients with recurrent vomiting.
(c) D. Upper endoscopy is indicated for patients over the age of 55 presenting with new-onset symptoms of
dyspepsia in order to evaluate for gastric cancer or other serious organic disease. Upper endoscopy is the
diagnostic study of choice to diagnose gastroduodenal ulcers, erosive esophagitis, and upper GI malignancy.
Ref: (28)



54. History & Physical/Infectious Diseases
A 6 year-old male, a recent immigrant from Latin America, is brought to the emergency department with difficulty
swallowing and breathing. On physical examination you note a gray-tan pseudomembrane in the back of the pharynx.
There is also tender cervical lymphadenopathy. What physical examination finding is most specific for diagnosis in
this patient?
A. Difficulty swallowing
B. Respiratory compromise
C. Pseudo-membrane in pharynx
D. Cervical adenopathy
Explanations
(u) A. See C for explanation.
(u) B. See C for explanation.
(c) C. Diphtheria most commonly presents with sore throat, adherent tonsillar, pharyngeal, or nasal pseudo-
membrane and low grade fever. The pseudo-membrane is most specific for diphtheria.
(u) D. See C for explanation.
Ref: (7)


Copyright 2010. Physician Assistant Education Association

27
55. Clinical Intervention/Pulmonology
A 40 year-old male nonsmoker in good health undergoes a routine chest x-ray for an insurance physical. Results
show an isolated, well-defined, coin lesion 1 cm in size. Which of the following is the next step in the evaluation of this
problem?
A. Review old radiographs
B. Order chest CT
C. Schedule lung biopsy
D. Prepare for surgical lung resection
Explanations
(c) A. The first and most important step in the radiographic evaluation is to review old radiographs to estimate
doubling time, an important marker for malignancy.
(u) B. See A for explanation.
(u) C. See A for explanation.
(u) D. See A for explanation.
Ref: (28)




56. Diagnosis/ENT/Ophthalmology
Lesions of the optic chiasm will produce which of the following conditions?
A. Amaurosis fugax
B. Bitemporal hemianopia
C. Unilateral blindness
D. Homonymous hemianopia
Explanations
(u) A. See B for explanation.
(c) B. A lesion of the optic chiasm will produce bitemporal hemianopia.
(u) C. See B for explanation.
(u) D. See B for explanation.
Ref: (3)




57. History & Physical/Cardiology
A 75 year-old female with a history of long-standing hypertension presents with shortness of breath. On examination
you note a diastolic murmur at the left upper sternal border. Which of the following maneuvers would accentuate this
murmur?
A. Sitting up and leaning forward
B. Lying on left side
C. Performing Valsalva maneuver
D. Standing upright
Explanations
(c) A. This patient has history findings consistent with aortic insufficiency which is characterized by a diastolic
murmur that is accentuated when the patient sits up and leans forward.
(u) B. The left lateral decubitus position accentuates the murmur of mitral stenosis.
(u) C. Valsalva and standing maneuvers help to differentiate the murmurs associated with aortic stenosis and
hypertrophic cardiomyopathy.
(u) D. See C for explanation.
Ref: (3)




Copyright 2010. Physician Assistant Education Association

28
58. Diagnosis/Obstetrics/Gynecology
A 25 year-old female, G2 P1001, presents to your office at 11-weeks gestation with vaginal bleeding, mild lower
abdominal cramping, and bilateral lower pelvic discomfort. On examination, blood is noted at the dilated cervical os.
No tissue is protruding from the cervical os. The uterus by palpation is 8-9 weeks gestation. No other abnormalities
are found. Which of the following is the most likely diagnosis?
A. Threatened abortion
B. Inevitable abortion
C. Incomplete abortion
D. Complete abortion
Explanations
(u) A. Threatened abortion is characterized by bleeding in the first trimester without loss of fluid or tissue.
(c) B. Inevitable abortion is the gross rupture of membranes in the presence of cervical dilation.
(u) C. Incomplete abortion is when the cervical os is open and allows passage of blood. The products of conception
may remain in utero or may partially extrude through the open os.
(u) D. Complete abortion refers to a documented pregnancy that spontaneously passes all of the products of
conception.
Ref: (4)



59. Diagnostic Studies/Psychiatry/Behavioral Medicine
What laboratory test should be closely monitored in patients on long-term lithium treatment for bipolar disorder?
A. ALT
B. Calcium
C. Lipase
D. TSH
Explanations
(u) A. See D for explanation.
(u) B. See D for explanation.
(u) C. See D for explanation.
(c) D. Patients with bipolar disorder and those on long term lithium therapy are prone to hypothyroidism severe
enough to require treatment. Liver and pancreatic complications are not a common concern. Electrolyte/renal
issues can arise with poor fluid intake and severe vomiting and diarrhea, calcium does not require routine
monitoring.
Ref: (33)



60. Clinical Therapeutics/Dermatology
A 16 year-old female comes to the office because her acne has become more severe and she would like to discuss
treatment. She was originally prescribed topical clindamycin gel 1 year ago to treat open and closed comedones on
her face. Despite regular use of the medication, she now has several comedones and papulopustules across her
nose, cheeks, chin and forehead. Which of the following would be an appropriate additional treatment for moderate
acne?
A. Intralesional steroids
B. Oral doxycycline
C. Benzoyl peroxide gel
D. Metronidazole cream
Explanations
(u) A. Intralesional steroids can be used as an adjunct treatment for large cystic lesions.
(c) B. For moderate acne, oral antibiotics are added to the topical medications treatment regimen.
(u) C. For mild acne, benzoyl peroxide gel can be used alone or in combination with topical antibiotics and/or
topical retinoids.
(u) D. Metronidazole cream is a treatment for rosacea.
Ref: (10)

Copyright 2010. Physician Assistant Education Association

29

61. Diagnosis/Cardiology
A 50 year-old male with history of alcohol abuse presents with complaint of worsening dyspnea. Physical examination
reveals bibasilar rales, elevated jugular venous pressure, an S3 and lower extremity edema. Chest x-ray reveals
pulmonary congestion and cardiomegaly. Electrocardiogram shows frequent ventricular ectopy. Echocardiogram
shows left ventricular dilatation and an ejection fraction of 30%. Which of the following is the most likely diagnosis in
this patient?
A. Hypertrophic cardiomyopathy
B. Dilated cardiomyopathy
C. Restrictive cardiomyopathy
D. Tako-Tsubo cardiomyopathy
Explanations
(u) A. Hypertrophic cardiomyopathy is characterized by a hyperdynamic left ventricle with asymmetric left
ventricular hypertrophy.
(c) B. Dilated cardiomyopathy is often caused by chronic alcohol use. It is characterized by signs and symptoms of
left-sided heart failure, a dilated left ventricle and decreased ejection fraction.
(u) C. Restrictive cardiomyopathy is characterized more commonly by right-sided heart failure than by left-sided
heart failure. There is rapid early filling with diastolic dysfunction. Patients with restrictive cardiomyopathy will
have a small thickened left ventricle and a normal or near normal ejection fraction on echocardiogram.
(u) D. Tako-Tsubo cardiomyopathy (broken heart syndrome) is characterized by signs and symptoms of acute
coronary syndrome, ST segment elevation on ECG and left ventricular apical dyskinesia.
Ref: (28)



62. Scientific Concepts/Infectious Diseases
A 25 year-old female with a history of diarrhea presents to the emergency department with Guillian-Barre syndrome.
What is the most likely causative pathogen in this patient?
A. Shigella dysenteriae
B. Yersinia enterocolitica
C. Entamoeba histolytica
D. Campylobacter jejuni
Explanations
(u) A. See D for explanation.
(u) B. See D for explanation.
(u) C. See D for explanation.
(c) D. Campylobacter jejuni is a Gram-negative bacteria that has been linked to the subsequent development of
Guillain-Barr syndrome (GBS). GBS usually develops two to three weeks after the initial illness.
Ref: (7)



63. Clinical Intervention/Gastrointestinal/Nutritional
A 32 year-old female presents with an 8-week history of a burning sensation deep in the chest, which starts about 30
minutes after she eats and worsens when she lays down at night. She has had some relief with an antacid and
denies dysphagia. Which of the following interventions would be beneficial for this patient?
A. Raising the foot of the bed
B. Taking Cimetidine (Tagamet) 30 minutes after a meal
C. Eating 3 large meals per day
D. Avoiding mint
Explanations
(u) A. The head of the bed should be raised 6 inches to allow gravity to decrease reflux.
(u) B. H2 blockers have a 30-minute onset of action and so are best taken 30 minutes before a meal known to
cause reflux.
(u) C. Patients usually do better with smaller, more frequent meals, along with the recommendation to avoid eating

Copyright 2010. Physician Assistant Education Association

30
3 hours before bedtime.
(c) D. Both peppermint and chocolate are substances known to aggravate reflux.
Ref: (28)


64. Diagnostic Studies/Neurology
A 45 year-old female presents complaining of the worst headache of her life. Which of the following is the most
appropriate initial diagnostic study?
A. Magnetic resonance imaging (MRI)
B. Computed tomography (CT)
C. Electroencephalogram (EEG)
D. Lumbar puncture (LP)
Explanations
(u) A. An MRI is not as sensitive in detecting an acute cerebral bleed as a CT scan.
(c) B. A CT scan is more sensitive in detecting cerebral hemorrhage in the first 24 to 48 hours.
(u) C. Electroencephalogram (EEG) is used in the evaluation of seizures and will not assess the presence of
cerebral hemorrhage.
(u) D. A lumbar puncture is indicated in evaluation of suspected subarachnoid hemorrhage only if the CT scan
does not establish the diagnosis.
Ref: (7)



65. History & Physical/ENT/Ophthalmology
Which physical examination finding distinguishes allergic rhinitis from other rhinitis etiologies?
A. Clear rhinorrhea
B. Erythematous pharynx
C. Nasal flaring
D. Pale nasal turbinates
Explanations
(u) A. See D for explanation.
(u) B. See D for explanation.
(u) C. See D for explanation.
(c) D. On physical examination, the mucosa of the turbinates is usually pale or violaceous with allergic rhinitis
because of venous engorgement in contrast to the erythema of viral rhinitis.
Ref: (28)



66. Diagnosis/Pulmonology
A 56 year-old female with a 35 pack year smoking history presents to the clinic with shortness of breath and cough.
On examination, she is thin with no recent weight loss. She appears uncomfortable, breath sounds are diminished
without adventitious sounds. Pulmonary function tests show a marked increase in total lung capacity (TLC) and a
decreased FEV1. What is the most likely diagnosis for this patient?
A. Persistent asthma
B. Chronic obstructive pulmonary disease
C. Idiopathic fibrosing interstitial pneumonia
D. Sarcoidosis
Explanations
(u) A. Lung function in asthma is evaluated by FEV1/FVC ratio with reduction noted with airflow obstruction.
(c) B. Lung volume measurements in COPD reveal a marked increase in residual volume indicative of air trapping.
(u) C. Pulmonary function testing in idiopathic fibrosing interstitial pneumonia shows a loss of lung volume with
normal to increased airflow rates in interstitial lung disease.
(u) D. Restrictive changes with decreased lung volumes and diffusing capacity are common in sarcoidosis.
Ref: (28)

Copyright 2010. Physician Assistant Education Association

31
67. Health Maintenance/Hematology
Which of the following Rh genotypes in a mother and father would represent a risk for hemolytic disease of the
newborn?
A. Mother Rh-positive, father Rh-negative
B. Mother Rh-positive, father Rh-positive
C. Mother Rh-negative, father Rh-positive
D. Mother Rh-negative, father Rh-negative
Explanations
(u) A. See C for explanation.
(u) B. See C for explanation.
(c) C. If an Rh-negative woman carries an Rh-positive fetus, she may develop antibodies against Rh when fetal
blood cells enter her circulation.
(u) D. See C for explanation.
Ref: (28)



68. Clinical Therapeutics/Cardiology
A 76 year-old active female with history of hypertension and hypothyroidism presents with complaints of palpitations
and dyspnea on exertion. On examination vital signs are BP 120/80 mmHg, HR 76 bpm, irregular, RR 16. Heart
examination reveals an irregularly, irregular rhythm without murmur. Lungs are clear to auscultation and extremities
are without edema. Which of the following is the most important medication to initiate for chronic therapy in this
patient?
A. Warfarin (Coumadin)
B. Verapamil (Calan)
C. Amiodarone (Cordarone)
D. Digoxin (Lanoxin)
Explanations
(c) A. Anticoagulation is necessary in all patients with atrial fibrillation to prevent thromboembolic events unless
there is contraindication.
(u) B. This patient currently has a controlled ventricular rates and does not require chronic calcium channel
blockers or digoxin at this time.
(u) C. Antiarrhythmic therapy may be indicated in some patients with atrial fibrillation, but anticoagulation is
indicated in all patients unless there is contraindication.
(u) D. See B for explanation.
Ref: (28)



69. Diagnosis/Endocrinology
A 32 year-old male presents with a one-month history of weakness, anorexia, and weight loss. On physical
examination, he is hypotensive and his skin has diffuse hyperpigmentation. Plasma cortisol levels drawn at 8 am are
low. Which of the following is the most likely diagnosis?
A. Pituitary insufficiency
B. Addison disease
C. Cushing syndrome
D. Primary aldosteronism
Explanations
(u) A. Pituitary insufficiency presents with hypopigmentation, not hyperpigmentation.
(c) B. This is the classic presentation of Addison's disease (chronic adrenocortical insufficiency).
(u) C. Hypercortisolism (Cushing syndrome) would present with central obesity, hypertension, and elevated cortisol
levels.
(u) D. Weakness is a symptom of primary aldosteronism, but the patient would be hypertensive and aldosterone
levels would be elevated.
Ref: (28)

Copyright 2010. Physician Assistant Education Association

32
70. Diagnostic Studies/Orthopedics/Rheumatology
Antinuclear Antigen (ANA) is most commonly associated with and monitors progress of which of the following
disorders?
A. Systemic Lupus Erythematosis (SLE)
B. Rheumatoid arthritis (RA)
C. Osteoarthritis
D. Ankylosing spondylitis
Explanations
(c) A. Antinuclear Antigen is the most helpful screening tool for SLE.
(u) B. Although present in rheumatoid arthritis, ANA is not specific. A high rheumatoid factor (RF) is more telling
for RA.
(u) C. Osteoarthritis is not significantly associated with ANA.
(u) D. Ankylosing spondylitis is a seronegative disorder and does not reflect RF or ANA levels.
Ref: (24)



71. History & Physical/Gastrointestinal/Nutritional
A 72 year-old woman presents to your clinic complaining of constipation. Which of the following presentations would
be most concerning?
A. Fewer than one stools per week
B. Weight loss
C. Recurrent hemorrhoids
D. Scybala
Explanations
(u) A. See B for explanation
(c) B. Alarm symptoms are concerning for colorectal malignancy and include weight loss, anemia, hematochezia, or
positive fecal occult blood test. These symptoms are particularly worrisome in a patient with a family history of
colorectal cancer.
(u) C. See B for explanation.
(u) D. See B for explanation.
Ref: (28)



72. Diagnosis/Cardiology
A 58-year old male presents for a six week follow-up after an acute anterior wall myocardial infarction. He denies
chest pain and shortness of breath. Electrocardiogram shows persistent ST segment elevation in the anterior leads.
Echocardiogram reveals a sharply delineated area of scar that bulges paradoxically during systole. Which of the
following is the most likely diagnosis in this patient?
A. Left ventricular aneurysm
B. Postinfarction ischemia
C. Ischemic cardiomyopathy
D. Constrictive pericarditis
Explanations
(c) A. Left ventricular (LV) aneurysm develops in about 10-20 percent of patients following acute myocardial
infarctions, especially anterior wall myocardial infarctions. LV aneurysm is identified by ST segment elevation
that is present beyond 4-8 weeks after the acute infarct and a scar that bulges paradoxically during systole
on echocardiogram.
(u) B. Postinfarction ischemia is recurrent ischemia that is more common in patients with non-ST segment
elevation myocardial infarctions and is characterized by postinfarction angina. This patient denies any chest
pain.
(u) C. Ischemic cardiomyopathy would be characterized by decreased ejection fraction on echocardiogram and
wall motion abnormalities. Ischemic cardiomyopathy is not associated with ST segment elevation or bulge of
scar on echocardiogram.

Copyright 2010. Physician Assistant Education Association

33
(u) D. Constrictive pericarditis is characterized by signs and symptoms of right-sided heart failure with increased
jugular venous pressures and a septal bounce on echocardiogram.
Ref: (28)



73. Clinical Intervention/Obstetrics/Gynecology
Which of the following is recommended to reduce the risk for perinatal transmission of HIV in a patient with a viral
load of >1000 copies/mL?
A. Vaginal delivery with female condom
B. Episiotomy to shorten second stage of labor
C. Use of forceps or vacuum extractor to shorten second stage of labor
D. Cesarean section prior to onset of labor and rupture of membranes
Explanations
(u) A. See D for explanation.
(u) B. See D for explanation.
(u) C. See D for explanation.
(c) D. Cesarean section performed prior to the onset of labor and rupture of membranes significantly reduces the
risk of perinatal HIV transmission. Planned cesarean section delivery at 38 weeks of gestation to prevent
perinatal transmission of HIV is recommended in women with a viral load of >1000 copies/mL.
Ref: (4)



74. Health Maintenance/Urology/Renal
A 48 year-old male with a recent episode of nephrolithiasis comes to the office to discuss prevention of further urinary
stone disease. Which of the following dietary recommendations can reduce recurrence of nephrolithiasis?
A. Increase fluid intake
B. Decrease dietary fiber
C. Increase salt intake
D. Decrease calcium intake
Explanations
(c) A. To prevent stone recurrence, the recommendations are to double previous fluid intake.
(u) B. Dietary fiber, such as bran, can significantly decrease the risk of stone formation.
(u) C. Excess sodium ingestion is a cause of calcium stone disease.
(u) D. Dietary recommendations including maintaining a moderate calcium intake. Dietary calcium avoidance can
actually increase stone recurrence.
Ref: (28)



75. Clinical Therapeutics/Pulmonology
A 70 year-old male is admitted to the ICU with fever, leukocytosis and purulent sputum. Sputum culture shows
methicillin-resistant gram-positive cocci in clusters. Which of the following medications should be ordered for this
patient?
A. Vancomycin (Vancocin)
B. Clindamycin (Cleocin)
C. Azithromycin (Zithromax)
D. Astreonam (Azactam)
Explanations
(c) A. Vancomycin is the drug of choice for methicillin-resistant S. aureus.
(u) B. Clindamycin is used if the patient is at risk for Legionella infection.
(u) C. Azithromycin is more commonly used for H. influenza coverage.
(u) D. Astreonam covers gram positive organisms but not methicillin-resistant S. aureus.
Ref: (28)

Copyright 2010. Physician Assistant Education Association

34
76. History & Physical/Cardiology
A 75 year-old female with history of coronary artery disease and dyslipidemia presents for routine follow-up. Physical
examination reveals loss of hair on the lower extremities bilaterally with thinning of the skin. Femoral pulses are +2/4
bilaterally, pedal pulses are diminished bilaterally. Ankle brachial index is reduced. Which of the following signs or
symptoms is this patient most likely to have?
A. Lower extremity edema
B. Calf pain with walking
C. Numbness of the lower extremities
D. Itching of the lower extremities
Explanations
(u) A. This patient has signs and symptoms consistent with arterial insufficiency. Lower extremity edema is seen in
patients with venous insufficiency.
(c) B. This patient has signs and symptoms consistent with arterial insufficiency and would most likely complain of
intermittent claudication.
(u) C. Numbness of the lower extremities would be seen with acute arterial occlusion.
(u) D. Itching of the lower extremities may be seen in chronic venous insufficiency because of secondary skin
changes, but is not common in arterial insufficiency.
Ref: (28)



77. Diagnostic Studies/Neurology
A 37 year-old right-handed male presents with daily headaches for the past 2 months. They are worse in the morning
and awaken him from sleep. In the past week, he has noticed a tendency to drop things. Neurologic examination
reveals upper extremity strength of 4/5 on the right and 5/5 on the left. Which of the following is the most appropriate
next step in the evaluation of this patient?
A. Lumbar puncture (LP)
B. Electroencephalogram (EEG)
C. Magnetic resonance imaging (MRI)
D. Electromyelogram (EMG)
Explanations
(h) A. Lumbar puncture results would give information on the cerebrospinal fluid. In the presence of focal neurologic
findings, a lumbar puncture would be contraindicated prior to an MRI.
(u) B. An EEG is used in the evaluation of seizure disorders.
(c) C. MRI is the most appropriate diagnostic study in the evaluation of a suspected intracranial neoplasm.
(u) D. EMG is not indicated in the evaluation of a suspected intracranial neoplasm.
Ref: (1)



78. History & Physical/Orthopedics/Rheumatology
Which of the following histories is most consistent with rheumatoid arthritis (RA)?
A. Acute onset, morning stiffness, monoarticular joint swelling and tenderness
B. Insidious onset, morning stiffness, symmetrical joint swelling and tenderness
C. Insidious onset, morning stiffness, monoarticular joint swelling and tenderness
D. Acute onset, morning stiffness, symmetrical joint swelling and tenderness
Explanations
(u) A. Rheumatoid arthritis has an insidious onset, morning stiffness is typically longer than 30 minutes, and
although early RA may have monoarticular presentation, it is typically symmetrical.
(c) B. Rheumatoid arthritis has an insidious onset, morning stiffness is typically longer than 30 minutes, and is
typically symmetrical in presentation.
(u) C. Rheumatoid arthritis has an insidious onset, morning stiffness is typically longer than 30 minutes, and
although early RA may have monoarticular presentation, it is typically symmetrical.
(u) D. Rheumatoid arthritis has an insidious onset, morning stiffness is typically longer than 30 minutes, and
although early RA may have monoarticular presentation, it is typically symmetrical.
Ref: (28)

Copyright 2010. Physician Assistant Education Association

35
79. Clinical Therapeutics/Hematology
Low molecular weight heparin (LMWH) dosage is based on which of the following?
A. Weight
B. International normalized ratio (INR)
C. Prothrombin time (PT)
D. Partial thromboplastin time (PTT)
Explanations
(c) A. LMWH is based on a patient's weight in kilograms.
(u) B. INR is used to monitor anticoagulant medications.
(u) C. PT is used to monitor warfarin efficacy.
(u) D. PTT is used to monitor heparin use but not to determine the dosage.
Ref: (15)




80. Clinical Intervention/ENT/Ophthalmology
A 23 month-old male presents to the office with his father who reports the patient has had an acute onset of severe
pharyngitis, fever of 103.5 degrees F and what sounds like harsh, high-pitched breath sounds. His dad states the
child has started drooling and seems to be worsening. The child is not presently crying but has muffled voice sounds.
The child has not been immunized due to religious reasons. What is the most appropriate next step?
A. Administer ceftriaxone (Rocephin)
B. Emergent transfer
C. Administer racemic epinephrine
D. Obtain intravenous access
Explanations
(h) A. See B for explanation.
(c) B. Epiglottitis requires endotracheal intubation to maintain the airway but should be performed only in the
operating room or emergency room with a competent physician prepared to place an endotracheal tube or
less often to perform a tracheostomy.
(u) C. See B for explanation.
(h) D. See B for explanation.
Ref: (5)




81. History & Physical/Obstetrics/Gynecology
On physical examination of a pregnant patient, which can be considered a normal finding?
A. Increased second heart sound split with inspiration
B. Diastolic murmur
C. Facial edema
D. Hyperreflexia
Explanations
(c) A. Increased second heart sound split with inspiration is common in pregnancy due to the increased blood flow
across the aortic and pulmonic valves.
(u) B. Diastolic murmurs in pregnancy should be considered pathological and evaluated further.
(u) C. Facial edema in uncommon in pregnancy and if it occurs, the medical provider should consider preeclampsia.
(u) D. Hyperreflexia occurs with preeclampsia and does not occur in a normal pregnancy.
Ref: (4)





Copyright 2010. Physician Assistant Education Association

36
82. Scientific Concepts/Gastrointestinal/Nutritional
Which of the following is the most common cause of ascites?
A. Nephrotic syndrome
B. Portal hypertension
C. Bacterial peritonitis
D. Uterine malignancy
Explanations
(u) A. See B for explanation.
(c) B. Over 80% of patients with ascites have portal hypertension secondary to chronic liver disease. Infection,
malignancy, and nephrotic syndrome are common causes of non-portal hypertensive ascites.
(u) C. See B for explanation.
(u) D. See B for explanation.
Ref: (28)



83. Diagnosis/Cardiology
A 55 year-old male with history of hypertension and diabetes mellitus presents to the emergency department. The
patient's wife states that the patient developed progressive irritability and confusion today after complaining of a
headache. Physical examination reveals a BP of 230/130 mmHg and papilledema. Which of the following is the most
accurate diagnosis in this patient?
A. Resistant hypertension
B. Hypertensive urgency
C. Hypertensive emergency
D. Malignant hypertension
Explanations
(u) A. Resistant hypertension is the failure to reach blood pressure control in patients who are compliant with a 3
drug regimen including a diuretic.
(u) B. Hypertensive urgency is a systolic BP >220 or a diastolic BP >125 in a patient who is asymptomatic or who
has disk edema, progressive target organ complications. Hypertensive urgency must be treated within a few
hours of presentation.
(u) C. Hypertensive emergency is similar to hypertensive urgency, however the BP is significantly elevated and must
be lowered within an hour.
(c) D. Malignant hypertension is significantly elevated BP with progressive retinopathy, including papilledema,
encephalopathy, and headache.
Ref: (28)




84. Clinical Therapeutics/Urology/Renal
A 19 year-old man comes to the office because he has had a fever, frequent urination, urgency, dysuria and scrotal
pain for the past 2 days. On physical examination, he has a temperature of 101 degrees F, scrotal swelling, and
purulent urethral discharge is visible on penile examination. Gram stain of the discharge shows gram negative
intracellular diplococci. Which of the following is the most appropriate antibiotic therapy?
A. Ampicillin (Principen)
B. Ciprofloxacin (Cipro)
C. Ceftriaxone (Rocephin)
D. Nitrofurantoin (Macrobid)
Explanations
(u) A. Ampicillin can be prescribed in conjunction with gentamicin to treat acute pyelonephritis.
(u) B. Ciprofloxacin can be prescribed to treat acute cystitis or chronic bacterial prostatitis.
(c) C. Ceftriaxone is an effective treatment for acute epididymitis caused by Neisseria gonorrhoeae.
(u) D. Nitrofurantoin can be prescribed to treat acute cystitis.
Ref: (28)

Copyright 2010. Physician Assistant Education Association

37
85. Diagnosis/Pulmonology
A 74 year-old male with a history of coronary artery disease and atrial fibrillation presents to the clinic for follow-up of
his shortness of breath. Patient's medications include amiodarone (Cordarone) and metoprolol (Lopressor). His chest
x-ray reveals patchy ground-glass infiltrates. Which of the following is the most likely diagnosis?
A. COPD
B. Tuberculosis
C. Bronchiectasis
D. Pulmonary fibrosis
Explanations
(u) A. COPD appears as hyperinflation with flattening of the diaphragm on chest radiograph.
(u) B. Tuberculosis presents with pulmonary infiltrates on chest radiograph most often apical; cavitations may be
seen with progressive primary tuberculosis.
(u) C. Chest x-ray in bronchiectasis shows dilated and thickened bronchi that appear as ring-like markings.
(c) D. Pulmonary fibrosis presents with ground-glass infiltrates on CXR and is often associated with certain
medication use.
Ref: (28)



86. Health Maintenance/Psychiatry/Behavioral Medicine
What patient demographic is at highest risk for the development of anorexia nervosa?
A. College aged swimmer
B. High school football player
C. Married, 50 year old, heterosexual female
D. Single, 20 year old, homosexual male
Explanations
(u) A. See D for explanation.
(u) B. See D for explanation.
(u) C. See D for explanation.
(c) D. Homosexual males are at increased risk for anorexia nervosa due to the strong community desire for slimness.
This is not the case in the lesbian community. Wrestlers are at the greatest risk group among athletes along
with ballet dancers. Though anorexia nervosa is more common in the female population, its onset is far more
typical in the teens to early 20's.
Ref: (33)



87. Diagnostic Studies/Dermatology
A 58 year-old female comes to the office because she noticed a white spot on her tongue. She has a 42 pack/year
smoking history. On examination, she has an 8 mm white thickened, keratinized lesion on the ventral tongue. The
lesion cannot be removed by rubbing the mucosal surface. Which of the following studies is most appropriate to
confirm the suspected diagnosis?
A. Incisional biopsy
B. Tzanck smear
C. Gram stain
D. Potassium hydroxide wet prep
Explanations
(c) A. Leukoplakia is characterized as a white plaque that cannot be removed by wiping the mucosal surface.
Tobacco use is strongly associated with an increased incidence of leukoplakia and oral cancer. An incisional
biopsy of the lesion is required for histologic evaluation of the tissue.
(u) B. Herpes simplex is a viral infection caused by the herpes simplex virus (HSV) group. Vesicles may be
visualized on the soft palate, floor of mouth, and tongue. A Tzanck smear is taken from a vesicular lesion
and stained for identification of herpes simplex.
(u) C. A direct gram stain of a scraping/swab from oral cavity is performed on lesions due to suspected bacterial or
yeast infections.

Copyright 2010. Physician Assistant Education Association

38
(u) D. Oral candidiasis is characterized by creamy curd-like patches that can be easily rubbed off the mucosal
surface. The diagnosis can be confirmed by wet preparation using potassium hydroxide which reveals
pseudohyphae and yeast forms.
Ref: (28)



88. Scientific Concepts/Cardiology
A patient undergoes biopsy for suspected myocarditis. Which of the following is the most likely etiologic agent?
A. West Nile virus
B. Rhinovirus
C. Coxsackie B virus
D. Cytomegalovirus
Explanations
(u) A. See C for explanation.
(u) B. See C for explanation.
(c) C. Coxsackie B virus, Hepatitis C, adenovirus, and HIV are the predominant agents in clinically significant
acute viral myocarditis in the US.
(u) D. See C for explanation.
Ref: (7)



89. Diagnostic Studies/Orthopedics/Rheumatology
A 26 year-old male was lifting a heavy object two weeks ago when he felt a sudden onset of low back pain. He
describes pain in the low mid back at the belt line aggravated with movement. Radicular symptoms are noted in the
left buttock down the leg to the dorsal aspect of the foot. He denies any urine or bowel complaints His examination
demonstrates an inability to stand on his toes and a positive straight leg raise. Which of the following is most
appropriate diagnostic study in this patient?
A. Computed tomography (CT)
B. Magneti resonance imaging (MRI)
C. Discography
D. Electromyelogram
Explanations
(u) A. See B for explanation.
(c) B. MRI is the diagnostic study of choice in a patient with suspected disc herniation.
(u) C. See B for explanation.
(u) D. See B for explanation.
Ref: (1)



90. Health Maintenance/Neurology
Household contacts of a patient with bacterial meningitis are best treated with which of the following?
A. Amoxicillin (Amoxil)
B. Ciprofloxin (Cipro)
C. Tetracycline (Sumycin)
D. Vancomycin (Vancocin)
Explanations
(u) A. See B for explanation.
(c) B. Rifampin, Cipro, Levaquin, Zithromax and Rocephin are the drugs of choice.
(u) C. See B for explanation.
(u) D. See B for explanation.
Ref: (7)

Copyright 2010. Physician Assistant Education Association

39
91. Diagnosis/Gastrointestinal/Nutritional
A 45 year-old male nonsmoker presents with a 3-month history of difficulty swallowing. He describes the gradual
onset of dysphagia for solids and liquids; he now complains of substantial discomfort after eating. What is his most
likely diagnosis?
A. Achalasia
B. Schatzki ring
C. Esophageal carcinoma
D. Zenkers diverticulum
Explanations
(c) A. Achalasia is by far the most common cause of dysmotility. This is particularly true in a non-smoker (less
likely to be esophageal carcinoma or a lung cancer causing a paraneoplastic syndrome) and someone who
has not been to an area endemic for Chagas disease.
(u) B. See A for explanation.
(u) C. See A for explanation.
(u) D. See A for explanation.
Ref: (28)



92. Scientific Concepts/Pulmonology
An 18 month-old male presents with his parents who report symptoms of a barking cough and intermittent stridor that
has worsened over the past 12 hours. They note improvement in symptoms when he was taken outdoors to the cool
night air. Which of the following is the most likely organism causing this patient's symptoms?
A. Rubeola virus
B. Adenovirus
C. Influenza virus
D. Para-influenza virus
Explanations
(u) A. See D for explanation.
(u) B. See D for explanation.
(u) C. See D for explanation.
(c) D. Croup is most often caused by parainfluenza virus.
Ref: (13)



93. Clinical Intervention/Endocrinology
A 75 year-old male with chronic hypertension and history of coronary artery disease is assessed in the office for
routine evaluation. His hypertension is well-controlled with metoprolol (Lopressor) and hydrochlorothiazide (Diuril).
Electrocardiogram reveals U waves. Which of the following is the most likely cause for U waves in this patient?
A. Beta blocker therapy
B. Hypocalcemia
C. Hypokalemia
D. Medication noncompliance
Explanations
(u) A. Although beta blockers slow conduction through the AV node, they are not responsible for causing U waves
in this patient.
(u) B. Patients maintained on thiazide diuretics actually have a decrease in calciuria and are therefore, not at risk
for the development of hypocalcemia. Additionally, calcium disturbances lengthen the QRS complex and may
affect the QT interval but, they are not associated with the formation of U waves.
(c) C. U waves are associated with hypokalemia and also with the ingestion of certain medications such as
quinidine, procainamide, and disopyramide.
(u) D. See C for explanation.
Ref: (28)

Copyright 2010. Physician Assistant Education Association

40
94. Health Maintenance/Obstetrics/Gynecology
When is the recommendation for the next Pap smear in a 36 year-old patient with a history of 3 consecutive negative
annual Pap smears and no history of cervical dysplasia?
A. Yearly Pap smears
B. Pap smear in 4 years
C. Pap smear in 3 years
D. Patient does not need any further Pap smears
Explanations
(u) A. See C for explanation.
(u) B. See C for explanation.
(c) C. Women who have had 3 consecutive negative annual Pap smears results may be screened every 2 or 3 years
if they are 30 or older with no history of CIN 2 or 3, immunosuppression, HIV infection, or diethystilbestrol
(DES) exposure in utero.
(u) D. See C for explanation.
Ref: (4)



95. History & Physical/Psychiatry/Behavioral Medicine
What behavior would be most typical for a patient diagnosed with schizoid personality disorder?
A. Chooses solitary activities
B. Odd thinking and speech
C. Reckless disregard for safety of others
D. Uses physical appearance to draw attention
Explanations
(c) A. Persons with schizoid personalities are very withdrawn and do not seek or enjoy relationships and are
indifferent to praise or criticism. They generally appear cold and unfeeling to others.
(u) B. Schizotypal personalities have this along with magical thinking and ideas of reference. They may become
withdrawn due to the lack of acceptance.
(u) C. Reckless disregard for safety of others is typical of antisocial personality disorders.
(u) D. Using physical appearance to draw attention is more typical of histrionic disorder.
Ref: (33)



96. Health Maintenance/Cardiology
A 65 year-old male with coronary artery disease, hypertension, and diabetes mellitus is admitted with dyspnea and
lower extremity edema. The chest x-ray reveals small bilateral pleural effusions. Echocardiogram shows an ejection
fraction of 30% with no valvular heart disease. The patient is treated in the hospital with furosemide (Lasix) and
lisinopril (Zestril). What education should be given to this patient upon discharge to help prevent readmission?
A. Elevate the head of bed at home
B. Avoid physical activity
C. Monitor daily weights
D. Restrict fluid intake
Explanations
(u) A. Elevation of the head of the bed may help the patient if they have symptoms of dyspnea but it will not help
prevent readmission to the hospital with a CHF exacerbation.
(u) B. In stable patients increasing physical activity or regular exercise can diminish symptoms.
(c) C. Strategies to prevent rehospitalization can include monitoring daily weights, case management and patient
education regarding self-adjustment of diuretics.
(u) D. Fluid restriction is not helpful in the readmission for CHF.
Ref: (28)



Copyright 2010. Physician Assistant Education Association

41
97. Diagnosis/ENT/Ophthalmology
A 67 year-old African American male presents for a new patient evaluation. History reveals an aphasic CVA which
limits his history. Funduscopic examination reveals an abnormal vessel light reflex described as a silver or copper-
wire appearance. Where the vessels intersect, there appears to be some nicking. He has no carotid bruits, and his
cardiac exam is normal. What is the most likely cause of his ocular findings?
A. Cytomegalovirus retinitis
B. Diabetic retinopathy
C. Hypertensive retinopathy
D. Sickle cell retinopathy
Explanations
(u) A. See C for explanation.
(u) B. See C for explanation.
(c) C. Chronic hypertension accelerates the development of atherosclerosis. The retinal arterioles become more
tortuous and narrow and develop abnormal light reflexes (silver-wiring and copper-wiring). There is increased
venous compression at the retinal arteriovenous crossings (arterio-venous nicking), an important factor
predisposing to branch retinal vein occlusion.
(u) D. See C for explanation.
Ref: (28)


98. Diagnostic Studies/Pulmonology
A 14 year-old male presents to the ED experiencing a severe asthma attack. His respiratory effort is shallow and he
is using accessory muscles to breathe. Auscultation of his chest reveals no audible wheezing. Vital signs include BP
90/60 mmHg, P 160 bpm, RR 52. An arterial blood gas (ABG) is ordered. Normal ABG values at your institution are
pH 7.35-7.45, CO2 35-45, pO2 80-95. Which of the following ABG findings suggests the poorest prognosis?
A. pH =7.27 pCO2 =46 pO2 =56
B. pH =7.60, pCO2 =18 pO2 =80
C. pH =7.44, pCO2 =38 pO2 =90
D. pH =7.52, pCO2 =28, pO2 =80
Explanations
(c) A. pH =7.27 pCO2 =46 pO2 =56 is associated with the poorest prognosis in this patient.
(u) B. See A for explanation.
(u) C. See A for explanation.
(u) D. See A for explanation.
Ref: (28)


99. Clinical Intervention/Orthopedics/Rheumatology
A 22 year-old male presents to the emergency department complaining of right hand pain after punching a brick wall.
His pain is noted at the ulnar aspect of his hand and worsens with touch and movement. On examination, you notice
obvious swelling and tenderness over the dorsum of the right hand proximal to the metacarpal phalangeal (MCP) joint
of the fifth digit. Skin is intact. X-ray reveals a fracture of the proximal fifth metacarpal with good alignment. Which of
the following is the treatment of choice?
A. Immediate orthopedic consult for surgical fixation
B. Long finger splint extending beyond the MCP
C. Ulnar gutter splint to immobilize fracture site
D. Referral for casting
Explanations
(u) A. Fixation is warranted for displaced fractures, angulated fractures beyond 40 degrees, or extensor lag.
(u) B. A long finger splint would not immobilize the joint above and below and may not offer enough
immobilization.
(c) C. An ulnar gutter splint is the most appropriate care in the emergency room.
(u) D. Although you will eventually refer the patient for casting, the choice treatment initiated in the emergency
room should be immobilization.
Ref: (26)

Copyright 2010. Physician Assistant Education Association

42
100. Clinical Therapeutics/Neurology
A 22 year-old female presents with onset of right eye vision loss 24 hours ago which has partially improved. She has
experienced various widespread paresthesias, regional weakness and fatigue over the past six months. Episodes
vary in location, severity and duration but invariably lead to recovery. Her exam is completely normal but MRI reveals
numerous areas of periventricular gadolinium uptake. For this condition, what treatment has shown the greatest
benefit with long term use?
A. Interferon beta 1a (Avonex)
B. Ethosuximide (Zarontin)
C. Methylprednisolone (Solumedrol)
D. Pyridostigmine (Mestinon)
Explanations
(c) A. The interferon agents have been the longest used and best studied medications that provide the best long
term benefits in multiple sclerosis. Steroids do play a role in acute exacerbations but not as long term agents.
(u) B. See A for explanation.
(u) C. See A for explanation.
(u) D. See A for explanation.
Ref: (7)



101. Diagnostic Studies/Hematology
An African American male is placed on hydroxychlorquine (Plaquenil) for travel to Africa. Within six weeks he
presents complaining of fatigue. CBC shows anemia. Which of the following is the diagnostic study of choice in this
patient?
A. Peripheral smear
B. G6PD assay
C. Total iron binding capacity
D. Hemoglobin electrophoresis
Explanations
(u) A. Peripheral smear may reveal "bite" cells suggestive of G6PD, but is not diagnostic.
(c) B. G6PD assay is the diagnostic study of choice for G6PD deficiency.
(u) C. TIBC would be indicated in the evaluation of iron deficiency anemia, and is not indicated in the evaluation of
G6PD deficiency.
(u) D. Hemoglobin electrophoresis is indicated in the evaluation of hemolytic anemia and is not indicated in the
evaluation of G6PD deficiency.
Ref: (28)



102. History & Physical/Cardiology
A 60 year-old male nonsmoker with history of coronary artery disease presents with complaint of worsening dyspnea
on exertion for three weeks. He admits to orthopnea and lower extremity edema, but denies chest pain, palpitations,
and syncope. The patient's last echocardiogram revealed an ejection fraction of 30%. Which of the following would
you most likely find on physical examination?
A. Pericardial friction rub
B. Third heart sound
C. Accentuated first heart sound
D. Mid-systolic click
Explanations
(u) A. A pericardial friction rub is a sign of pericarditis, not heart failure.
(c) B. An S3 on physical examination is consistent with heart failure.
(u) C. An accentuated first heart sound is noted in tachycardia, short PR interval rhythms, increased cardiac
output states and mitral stenosis.
(u) D. A mid-systolic click is noted in patients with mitral valve prolapse.
Ref: (3 )

Copyright 2010. Physician Assistant Education Association

43
103. Diagnostic Studies/Psychiatry/Behavioral Medicine
A 25 year-old female presents with signs and symptoms of depression. She does not have any other known medical
problems. What diagnostic study is indicated in the initial evaluation of this patient?
A. Thyroid stimulating hormone (TSH)
B. Prolactin
C. Growth hormone (GH)
D. Cortisol
Explanations
(c) A. Patients who are presenting with symptoms of depression should be evaluated with a TSH because 10
percent of patients evaluated for depression have previously undetected thyroid dysfunction.
(u) B. See A for explanation.
(u) C. See A for explanation.
(u) D. See A for explanation.
Ref: (33)



104. Clinical Therapeutics/Orthopedics/Rheumatology
A 44 year-old female hairdresser presents to the clinic with a two-month history of pain and numbness of her left
hand that awakens her from sleep. Your exam reveals electric tingling sensations with percussion over the volar
surface of the wrist just proximal to the palmar crease and paresthesias noted within 15 seconds of passively flexing
the wrist. Which of the following would be the best initial treatment in this patient?
A. Corticosteroid injection
B. Surgical intervention
C. Nocturnal splinting
D. Gabapentin (Neurontin)
Explanations
(u) A. Corticosteroid use is recommended after initial treatment is unsuccessful.
(u) B. Surgical intervention is reserved for patients not responsive to initial/conservative treatment.
(c) C. Splinting the affected wrist is the initial treatment choice and maintains a neutral position of the wrist.
(u) D. Gabapentin is not indicated in the management of carpal tunnel syndrome.
Ref: (28)



105. History & Physical/Urology/Renal
A 34 year-old man comes to the urgent care with fever of 102 degrees F, chills, and perineal pain for the past 2 days.
In addition, he has difficulty voiding, dysuria, frequency, urgency, and nocturia. Which of the following physical
examination findings are consistent with this history?
A. Mild suprapubic tenderness
B. Scrotal swelling and tenderness
C. Prostate tender, warm, swollen
D. Costovertebral angle tenderness
Explanations
(u) A. Acute cystitis symptoms include dysuria, urgency and frequency. On physical exam, the patient is usually
afebrile and may have suprapubic tenderness.
(u) B. Acute epididymitis is characterized by symptoms of cystitis or urethritis (urethral discharge, pain at the tip of
the penis) with pain in the scrotum that may radiate to the flank.
(c) C. The diagnosis of acute prostatitis is supported by fever, irritative voiding symptoms, perineal pain and
exquisite tenderness of the prostate on rectal exam.
(u) D. Acute pyelonephritis is characterized by fever, flank pain and irritative voiding symptoms.
Ref: (28)



Copyright 2010. Physician Assistant Education Association

44
106. Clinical Therapeutics/Gastrointestinal/Nutritional
A patient was treated for community acquired pneumonia with amoxicillin-clavulanate (Augmentin). On day 7 of
therapy he develops fulminate diarrhea. The diarrhea is described as greenish and foul-smelling. He admits to
associated abdominal cramps. Which of the following is the treatment of choice for this patient?
A. Metronidazole (Flagyl)
B. Diphenoxylate/atropine (Lomotil)
C. Clindamycin (Cleocin)
D. Ciprofloxacin (Cipro)
Explanations
(c) A. Patients with C. difficile colitis should be treated with Flagyl for 10-14 days following cessation of the
diarrhea-inducing antibiotics.
(u) B. See A for explanation.
(u) C. See A for explanation.
(u) D. See A for explanation.
Ref: (28)



107. Diagnosis/Obstetrics/Gynecology
A 48 year-old G3P3003 female presents to the office complaining of severe secondary dysmenorrhea and
menorrhagia over the last 6 months. On physical examination her uterus feels enlarged and irregular. Which of the
following is the most likely diagnosis?
A. Endometritis
B. Endometriosis
C. Uterine leiomyomata
D. Endometrial hyperplasia
Explanations
(u) A. Endometritis is an infection of the endometrium. It occurs most commonly postpartum.
(u) B. Endometriosis most commonly presents with dysmenorrhea, dyschezia, and dyspareunia. Usually the uterus
is not enlarged.
(c) C. Uterine leiomyomata (fibroid) typically presents with severe dysmenorrhea and menorrhagia. An enlarged,
irregular uterus is noted on examination.
(u) D. Endometrial hyperplasia can cause menorrhagia but is not usually a cause of dysmenorrhea.
Ref: (4)




108. Scientific Concepts/ENT/Ophthalmology
What is the most common anatomical location of anterior bleeding from the nose?
A. Kiesselbach plexus
B. Maxillary sinus ostia
C. Nasal pyramid
D. Superior turbinate
Explanations
(c) A. Bleeding is most common in the anterior septum where a confluence of veins creates a superficial venous
plexus known as Kiesselbach plexus.
(u) B. See A for explanation.
(u) C. See A for explanation.
(u) D. See A for explanation.
Ref: (28)




Copyright 2010. Physician Assistant Education Association

45
109. Health Maintenance/Dermatology
A 9 year-old boy comes to the office with his mother to discuss treatment for chronic dry skin, and pruritic
inflammatory lesions of the flexor surfaces of the neck, hands and wrists. Past medical history is significant for
allergic rhinitis in the spring and fall. On physical examination the skin lesions are excoriated, and lichenified with
crusted patches. Which of the following prevention strategies can help minimize the symptoms in this patient?
A. Avoid rubbing or scratching
B. Limit sun exposure
C. Take hot baths
D. Consume hot liquids
Explanations
(c) A. Rubbing or scratching plaques can exacerbate the pruritis and lichenification in atopic dermatitis.
(u) B. UVA-UVB phototherapy is an effective treatment for atopic dermatitis.
(u) C. Hot baths can exacerbate the pruritis in atopic dermatitis. Tepid baths followed by use of unscented
emollient creams can help prevent itching and scratching.
(u) D. In rosacea, the facial flushing and increased skin temperature often occurs in response to the stimulation of
hot liquids in the mouth.
Ref: (10)


110. Clinical Intervention/Pulmonology
A 60 year-old female with a 30 pack year smoking history complains of new onset shortness of breath. On physical
examination, dullness is noted on percussion with dimished breath sounds over her left base. Chest x-ray shows a
new left pleural effusion. Which of the following is the next step in the management of this patient?
A. Repeat chest x-ray in two months
B. Perform diagnostic thoracentesis
C. Order MRI of the chest
D. Treat with antibiotic
Explanations
(u) A. See B for explanation.
(c) B. Diagnostic thoracentesis should be performed whenever there is a new pleural effusion and no clinically
apparent cause.
(u) C. See B for explanation.
(u) D. See B for explanation.
Ref: (28)



111. Clinical Intervention/Neurology
A 50 year-old female has a history of severe, stabbing pains, lasting only seconds, over the cheek and chin areas.
She also experiences intense pain in these areas with chewing, washing her face and even with smiling. Examination
reveals no sensory or motor deficits. Head computer tomography (CT) and erythrocyte sedimentation rate (ESR) are
normal. She has been unresponsive to maximum doses of carbamazepine. What is the next best option for this
patient?
A. Combination simvastatin (Zocor) and sertraline (Zoloft)
B. Bilateral deep brain stimulation
C. Stereotactic (Gamma knife) radiosurgery
D. High dose corticosteroid therapy
Explanations
(u) A. See C for explanation.
(u) B. See C for explanation.
(c) C. Trigeminal neuralgia is generally responsive to Carbamazepine however, for those who fail to respond to
medicinal therapy surgery is the next valid option. Though there are various procedures, Stereotactic
(Gamma knife) radiosurgery has provided the best results.
(u) D. See C for explanation.
Ref: (7)

Copyright 2010. Physician Assistant Education Association

46
112. Diagnosis/Orthopedics/Rheumatology
A 41 year-old male with a history of intravenous drug abuse presents to your office with acute, nontraumatic right
knee pain, chills, and sweats starting 2 days ago. On physical examination, his temperature is 102.9 degrees F. The
right knee is erythematous, edematous and tender to palpation and range of motion. Plain knee x-ray reveals soft
tissue swelling. Which of the following is most likely the diagnosis?
A. Gouty arthritis
B. Septic arthritis
C. Rheumatoid arthritis
D. Psoriatic arthritis
Explanations
(u) A. See B for explanation.
(c) B. This patient's signs and symptoms are most consistent with septic arthritis. IV drug abuse places this patient
at even greater risk.
(u) C. See B for explanation.
(u) D. See B for explanation.
Ref: (28)



113. Clinical Therapeutics/Cardiology
A 78 year-old male with history of coronary artery disease s/p coronary artery bypass grafting, hypertension, and
dyslipidemia presents for routine physical examination. He feels well except for occasional brief episodes of
substernal chest pain with exertion that are relieved with rest. He denies associated dyspnea, nausea or
diaphoresis. Physical examination reveals a BP of 110/70 mmHg, HR 56 bpm, regular, RR 14, unlabored. Lungs are
clear to auscultation, heart is bradycardic, but regular with no S3, S4 or murmur. Electrocardiogram done in the
office shows no acute ST-T wave changes. Which therapy is indicated for the acute management of this patient's
symptoms?
A. Sublingual nitroglycerine
B. Metoprolol (Lopressor)
C. Verapamil (Calan)
D. Lisinopril (Zestril)
Explanations
(c) A. Sublingual nitroglycerine is the drug of choice for the acute management of chronic stable angina.
(u) B. Beta-blockers are preventative and not the first choice for the acute management of chronic stable angina.
Beta-blockers may worsen this patient's bradycardia.
(u) C. Calcium channel blockers are the third-line antiischemic agent and may also reduce the patient's heart
rate.
(u) D. ACE inhibitors will not provide acute relief of anginal symptoms.
Ref: (28)


114. Diagnosis/Psychiatry/Behavioral Medicine
A patient is evaluated for extreme fears of abandonment and an inability to care for himself throughout his adult life.
He avoids disagreements and has difficulty initiating projects or acting on his own thoughts and ideas. Advice and
reassurance are sought for even minor daily details. There are no reported suicidal tendencies or signs of self harm.
What is the most likely personality disorder?
A. Narcissistic
B. Borderline
C. Dependent
D. Obsessive-compulsive
Explanations
(u) A. See C for explanation.
(u) B. See C for explanation.
(c) C. These patients are constantly seeking external support and will do even unpleasant things for others to gain
approval and nurturing. Narcissitic patients are egotistic and would not seek the opinion of others. Patients with
borderline disorder can share some of these traits as they make aggressive efforts to avoid abandonment but

Copyright 2010. Physician Assistant Education Association

47
the suicidal tendencies, impulsivity and self mutilation differentiate it. Obsessive-compulsive patients do not
rely on external support and obey very strict personal rules of perfection and efficiency and choose not to rely
on others.
(u) D. See C for explanation.
Ref: (33)


115. History & Physical/Hematology
A 52 year-old female is diagnosed with chronic myeloid leukemia (CML). Which of the following would you expect to
find on physical examination of this patient?
A. Hepatomegaly
B. Splenomegaly
C. Posterior cervical lymphadenopathy
D. Inguinal lymphadenopathy.
Explanations
(u) A. See B for explanation.
(c) B. CML patients present with an enlarged spleen.
(u) C. CML patients typically present with enlarged spleen and fatigue, but there are not enlarged lymph nodes.
(u) D. Patients with CML do not present with inguinal lymphadenopathy.
Ref: (28)


116. Health Maintenance/Endocrinology
Screening for hypothyroidism is indicated in which of the following patients??
A. Newborn
B. Young female with anxiety
C. Patient with new onset of atrial fibrillation
D. Patient with heat intolerance
Explanations
(c) A. The American Academy of Pediatrics recommends screening of hypothyroid disease between 2-4 days of
birth.
(u) B. Hypothyroidism is not a cause of anxiety in young females. Anxiety is more common with hyperthyroidism.
(u) C. Atrial fibrillation is more likely to occur with thyrotoxicoxis.
(u) D. Hypothyroidism presents most commonly with cold intolerance.
Ref: (13)


117. Clinical Intervention/ENT/Ophthalmology
A construction worker presents with severe eye pain after splashing concrete in his eye. Examination is complicated
by the patient's difficulty opening his eye. What is the most appropriate therapy - remembering that cement is an
alkali?
A. Irrigation with saline
B. Instill corticosteroids drops
C. Neutralize with acidic solution
D. Prescribe local anesthetic drops
Explanations
(c) A. Chemical burns are treated by copious irrigation of the eyes with saline solution, plain water, or buffering
solution if available as soon as possible after exposure. Neutralization of an acid with an alkali or vice versa
generates heat and may cause further damage. Alkali injuries are more serious and require prolonged
irrigation, since alkalis are not precipitated by the proteins of the eye as are acids.
(u) B. See A for explanation.
(u) C. See A for explanation.
(u) D. See A for explanation.
Ref: (28)

Copyright 2010. Physician Assistant Education Association

48
118. Diagnostic Studies/Pulmonology
A 57 year-old man is being evaluated for shortness of breath. The following spirometric data are obtained:

VC 4.90 L (predicted), 5.15 L (observed) 105% predicted
FRC 3.99 L (predicted), 4.37 L (observed) 110% predicted
RV 2.47 L (predicted), 3.17 L (observed) 128% predicted
FEV1 3.50 L (predicted), 2.35 L (observed) 67% predicted

These findings are consistent with which of the following?
A. No demonstratable abnormality
B. Restrictive lung disease
C. Obstructive lung disease
D. A ventilation/perfusion mismatch
Explanations
(u) A. See C for explanation.
(u) B. Restrictive lung disease would show decreased total lung capacity, vital capacity, and normal to increased
FEV1.
(c) C. Spirometry findings in obstructive lung disease typically show normal or increased total lung capacity,
decreased vital capacity, prolonged FEV1, and increased residual volume.
(u) D. A ventilation/perfusion scan would be abnormal with a pulmonary embolism.
Ref: (1)


119. History & Physical/Obstetrics/Gynecology
Your patient has just delivered her baby vaginally without difficulty. The patient has a laceration of the vaginal
mucosa including the perineal body. You repair it without difficulty. On the chart you document this as what type of
tear?
A. First degree
B. Second degree
C. Third degree
D. Fourth degree
Explanations
(u) A. A first degree tear involves the vaginal mucosa or perineal skin, but does not involve the underlying tissue.
(c) B. A second degree tear involves the underlying subcutaneous tissues, but not the rectal sphincter or rectal
mucosa.
(u) C. A third degree tear extends through the rectal sphincter, but not into the rectal mucosa.
(u) D. A fourth degree tear extends into the rectal mucosa.
Ref: (4)



120. Scientific Concepts/Cardiology
A 36 year-old male presents for follow-up of his hypertension. The patient is currently on three anti-hypertensive
medications without improvement of his blood pressure. On examination his BP is 170/86mmHg and his HR is 60bpm
and regular. His heart examination reveals a regular rate and rhythm without S3, S4 or murmur and his lungs are
clear to auscultation bilaterally. Abdominal examination reveals a bruit over his left upper abdomen. Which of the
following is the most likely underlying etiology for this patient's hypertension?
A. Pheochromocytoma
B. Renal artery stenosis
C. Cushing syndrome
D. Coarctation of the aorta
Explanations
(u) A. Pheochromocytoma is an uncommon cause of hypertension characterized by paroxysms of headache,
sweating and palpitations. There are no bruits associated with pheochromocytoma.
(c) B. Renal artery stenosis is characterized by hypertension that is resistant to three or more medications and renal

Copyright 2010. Physician Assistant Education Association

49
artery bruits on examination.
(u) C. Cushing syndrome is characterized by "moon" facies, a buffalo hump, a protuberant abdomen, weakness and
headache. There are no renal artery bruits associated with Cushing syndrome.
(u) D. Coarctation of the aorta is associated with hypertension in the upper extremities and normal or low blood
pressure in the lower extremities. There are often weak femoral pulses and a late systolic ejection murmur or
associated aortic insufficiency murmur.
Ref: (28)



121. Health Maintenance/Orthopedics/Rheumatology
A 56 year-old white post-menopausal female had a recent surveillance DEXA bone densitometry. The T-score is -2.7
for her right hip. In counseling this patient, in addition to medications, which of the following would you recommend?
A. Weight bearing exercises
B. Water aerobics
C. Limit sun exposure
D. Daily red wine consumption
Explanations
(c) A. Weight bearing exercises are an appropriate adjunct to medication in a patient with osteoporosis. Water
aerobics are non-weight bearing. Sun exposure adn vitamin D supplementation are useful in preventing and
treating osteomalacia.
(u) B. See A for explanation.
(u) C. See A for explanation.
(u) D. See A for explanation.
Ref: (28)



122. History & Physical/Neurology
What term is given to an ocular examination finding where small, irregular pupils are seen that react with near vision
but not to light?
A. Amaurosis fugax
B. Argyll-Robertson
C. Anisocoria
D. Optic neuritis
Explanations
(u) A. See B for explanation.
(c) B. This finding may be seen in diabetes or in cases of neurosyphilis whereas amaurosis fugax is vision loss due
to a central lesion, anisocoria has unequal sized pupils, unilateral loss of vision due to multiple sclerosis.
(u) C. See B for explanation.
(u) D. See B for explanation.
Ref: (1)



123. Clinical Therapeutics/Urology/Renal
A 63 year-old man comes to the office to discuss treatment for erectile dysfunction. He is interested in learning more
about the medication sildenafil (Viagra). He has a history of coronary artery disease, asthma and benign prostatic
hyperplasia. Which of the following medications is contraindicated (category X) with Viagra due to the potential drug
interaction?
A. Enalapril (Vasotec)
B. Albuterol
C. Finasteride (Proscar)
D. Nitroglycerin

Copyright 2010. Physician Assistant Education Association

50
Explanations
(u) A. The additive effect of sildenafil on enalapril can potentiate the anti-hypertensive effect. This is a category C
interaction; monitor therapy as dosage adjustments may be required.
(u) B. There is no interaction between albuterol and sildenafil.
(u) C. There is no interaction between finasteride and sildenafil.
(c) D. The additive effect of sildenafil on nitrates can amplify cardiac preload reduction and hypotension. Sildenafil
is contraindicated in patients taking nitrates.
Ref: (15)



124. Diagnostic Studies/Orthopedics/Rheumatology
A 62 year-old female presents to the emergency room with significant back pain without radiation after lifting a box
weighing approximately 15 pounds. She denies any previous trauma or injuries. Past history includes hysterectomy at
age 42 and a 49 pack year smoking history. Her current weight is 107 pounds. Lumbo-sacral spine film indicates a
spinal compression fracture at level L4. Which of the following tests would you perform to further assess the patient's
findings?
A. Computed tomography (CT) of the spine
B. Magnetic resonance imaging (MRI) of the spine
C. DEXA scan
D. Technetium-99m bone scan
Explanations
(u) A. CT of the spine should be reserved for fractures that remain symptomatic or progress after treatment.
(u) B. MRI is a good tool in evaluation of union versus non union fractures and should be reserved for fractures that
remain symptomatic or progress after treatment.
(c) C. Low patient weight, smoking, and early estrogen deficiency are all risk factors for osteoporosis. Spontaneous
fractures occurring from lifting with above risk factors should be evaluated for bone density.
(u) D. Technetium-99m bone scan is useful in the evaluation of active bone formation (or lack of) and is reserved
for fractures that remain symptomatic or progress after treatment.
Ref: (26)



125. Diagnosis/Gastrointestinal/Nutritional
A 31 year-old female complains of diarrhea. She admits to associated weight loss and steatorrhea, but denies
melena or hematochezia. Which of the following is the most likely diagnosis?
A. Celiac disease
B. Lactose intolerance
C. Short bowel syndrome
D. Irritable bowel syndrome
Explanations
(c) A. Greater than 10 grams of fecal fat in 24 hours is most consistent with a malabsorption syndrome such as
celiac sprue.
(u) B. See A for explanation.
(u) C. See A for explanation.
(u) D. See A for explanation.
Ref: (28)



126. Clinical Therapeutics/ENT/Ophthalmology
A 9 year-old patient presents with conjunctivitis after swimming at the local pool. On examination, there is visible lid
edema with redness of the palpebral conjunctiva, copious watery discharge, and scanty exudate. The sanitation
system of the public pool is through the use of a salt water system; therefore, the possibility of a chemical induced
conjunctivitis is almost non-existent. Which of the following should be instituted to prevent the sequalae of the

Copyright 2010. Physician Assistant Education Association

51
condition?
A. Ketorolac tromethamine (Acular)
B. Dexamethasone opthalmic
C. Naphazoline HCL (Naphcon A)
D. Sulfacetamide opthalmic
Explanations
(u) A. See D for explanation.
(u) B. See D for explanation.
(u) C. See D for explanation.
(c) D. One of the most common causes of viral conjunctivitis is adenovirus type 3. Contaminated swimming pools
can be source of infection. Topical sulfonamides prevent secondary bacterial infection.
Ref: (28)



127. Clinical Intervention/Pulmonology
A 36 year-old male who is hospitalized because of severe injuries from a motor vehicle accident develops rapid onset
of profound dyspnea. Initial chest x-ray shows a normal heart size with diffuse bilateral infiltrates. Follow-up chest x-
ray shows confluent bilateral infiltrates that spare the costophrenic angles. Which of the following is the best clinical
intervention for this patient?
A. Provide supplemental oxygen
B. Insert chest tube
C. Tracheal intubation
D. Administer corticosteroids
Explanations
(u) A. Marked hypoxemia is refractory to treatment with supplemental oxygen in ARDS.
(u) B. Chest tube insertion is not indicated in a patient with ARDS.
(c) C. Treatment of hypoxemia in acute respiratory distress syndrome (ARDS) usually requires tracheal
intubation.
(u) D. Routine use of corticosteroids is not recommended in the management of ARDS.
Ref: (28)



128. Clinical Therapeutics/Cardiology
A 16 year-old male presents with complaint of syncope after basketball practice today. Physical examination reveals
a systolic murmur along the left sternal border that increases with Valsalva maneuver. An electrocardiogram reveals
left ventricular hypertrophy. Echocardiogram shows asymmetric left ventricular hypertrophy with a hypercontractile
left ventricle. Which of the following is the initial medication of choice in this patient?
A. Metoprolol (Lopressor)
B. Losartan (Cozaar)
C. Lisinopril (Zestril)
D. Hydrochlorothiazide (Diuril)
Explanations
(c) A. Beta-blockers are the initial drug of choice in a symptomatic patient with hypertrophic cardiomyopathy.
(u) B. See A for explanation.
(u) C. See A for explanation.
(u) D. See A for explanation.
Ref: (28)

Copyright 2010. Physician Assistant Education Association

52
129. Diagnostic Studies/Dermatology
A 37 year-old right-handed landscaper comes to the office for evaluation of recurrent itching and stinging of the skin
on his right hand. Physical examination reveals confluent papules, vesicles, erosions and crusts on the dorsum of his
right hand. Which of the following is the most appropriate initial diagnostic study in this patient?
A. Skin scraping and microscopy
B. Patch testing
C. RAST testing
D. Skin biopsy
Explanations
(u) A. Skin scraping and microscopy are used to identify the organism, ova or feces in the laboratory diagnosis of
scabies.
(c) B. In patch testing, substances are applied to the skin in shallow cups and left in place for 24-48 hours.
Sensitivity to an allergen is confirmed by a papular vesicular reaction that occurs within 48-72 hours of the
laboratory testing.
(u) C. RAST testing is more specific and would not be done as the initial diagnostic study in this patient.
(u) D. A skin biopsy is indicated for all skin lesions that are suspected to be neoplasms.
Ref: (10)



130. Scientific Concepts/Obstetrics/Gynecology
What phase of the female menstrual cycle occurs at the time of elevated estrogen and LH/FSH surge?
A. Follicular phase
B. Proliferative phase
C. Ovulation
D. Secretory phase
Explanations
(u) A. The follicular phase begins with the onset of menses (day 1 of the menstrual cycle) and ends on the day of
the LH surge.
(u) B. Progesterone causes differentiation of the endometrial components and converts proliferative endometrium
into a secretory endometrium.
(c) C. Ovulation occurs within 30-36 hours of the LH surge and at the time of elevated estrogen.
(u) D. Secretory phase occurs when estrogen is elevated.
Ref: (4)



131. Scientific Concepts/Hematology
A 27 year-old male nonsmoker is diagnosed with upper extremity venous thrombosis. What is the most likely factor
abnormality found in this patient?
A. Factor VIII
B. Factor V
C. Factor I
D. Factor X
Explanations
(u) A. See B for explanation.
(c) B. In young adults with otherwise unexplained thrombosis, factor V Leiden is the most common associated
abnormality.
(u) C. See B for explanation.
(u) D. See B for explanation.
Ref: (1)




Copyright 2010. Physician Assistant Education Association

53
132. Diagnosis/Orthopedics/Rheumatology
A 34 year-old female construction worker presents with episodic blanching of her fingers when exposed to cold
weather. The physical examination of her extremities and digits is normal at this time. Which of the following is the
most likely diagnosis?
A. Sjogren syndrome
B. Scleroderma
C. Raynaud phenomenon
D. Systemic lupus erythematosus
Explanations
(u) A. Sjogren syndrome is a secondary cause of Raynauds, but it is not the disorder causing the symptoms.
(u) B. Scleroderma is a secondary cause of Raynauds, but it is not the disorder causing the symptoms.
(c) C. Raynauds phenomenon is caused by vascular spasm when exposed to cold or stressful situations.
(u) D. Systemic Lupus Erythematosus is a secondary cause of Raynauds, but it is not the disorder causing the
symptoms.
Ref: (28)



133. History & Physical/ENT/Ophthalmology
You are evaluating a 67 year-old Asian male in the emergency room for acute onset right eye pain. He states he was
at the evening premier of a newly released movie when the pain started. He had acute, profound visual loss in the
affected eye. The pain was intense enough for him to leave the theatre before the movie's conclusion, and present to
your location. On examination, the eye appears injected (red) and the cornea appears hazy. His pupils are 6 mm on
the affected side and 3 mm on the unaffected side. They respond to light on the unaffected side but not on the
affected side. On palpation, the globe feels tense. What history question is most relevant to support the diagnosis?
A. Contact lens use
B. Past sexual contacts
C. Recent URI symptoms
D. Visualizing halos around street lights
Explanations
(u) A. The major risk from contact lens wear is bacterial, amebic, or fungal corneal infection, potentially a blinding
condition. In this condition the eye may appear red, however the cornea would be clear, and the globe would
not be tense.
(u) B. Past sexual contacts would be related to pupillary abnormalities associated with neurosyphilis.
(u) C. URI symptoms would be considered when associated with conjunctivitis. Pupil size is normal and is the
pupillary light response. Intraoccular pressure is normal.
(c) D. Primary acute angle-closure glaucoma occurs only with closure of a preexisting narrow anterior chamber angle
found in older age groups, hyperopes, inuites, and Asians. Angle closure may be precipitated by pupilary
dilation and thus can occur from sitting in a darkened room, at times of stress or, rarely, from pharmacologic
mydriasis. The symptoms given are classic for acute angle-closure glaucoma (older age group, Asian, rapid
onset of severe pain and profound visual loss with halos around light, red eye, steamy cornea, dilated pupil,
hard eye to palpation).
Ref: (28)



134. Clinical Intervention/Cardiology
An 80 year-old female presents with syncope and recent fatigue and lightheadedness over the past month. She
denies chest pain or dyspnea. Physical examination reveals BP 130/70 mmHg, HR 40 bpm, regular, and RR 16.
Electrocardiogram reveals two p waves before each QRS complex. Which of the following is the treatment of choice
for this patient?
A. Cardio defibrillator insertion
B. Atropine as needed
C. Permanent dual chamber pacemaker insertion
D. Ritalin therapy daily

Copyright 2010. Physician Assistant Education Association

54
Explanations
(u) A. Cardio defibrillators treat ventricular tachycardia and are not indicated in the management of second degree
AV block.
(u) B. Atropine can be used in the acute management of second degree AV block Mobitz type II, but it should not
be used as long-term therapy.
(c) C. This patient has findings consistent with symptomatic second degree AV block Mobitz type II for which
permanent pacing is the treatment of choice.
(u) D. Ritalin therapy is not indicated in the management of second degree heart block.
Ref: (7)


135. Diagnosis/Neurology
A patient describes a history of intermittent and uncontrollable twitching of his right hand that spreads to involve the
entire arm after a few minutes. Afterward, the arm is extremely weak. There are no other areas of involvement,
sensory deficit or altered consciousness. What is the most likely seizure diagnosis?
A. Absence
B. Complex-partial
C. Simple-partial
D. Myoclonic
Explanations
(u) A. See C for explanation.
(u) B. See C for explanation.
(c) C. The lack of altered consciousness and focal motor symptoms are strongly indicative of this type of seizure.
(u) D. See C for explanation.
Ref: (7)



136. History & Physical/Urology/Renal
A 15 year-old man comes to the office with acute onset of nausea, severe pain and swelling within the right testis. He
has no fever or irritative voiding symptoms. Which of the following scrotal physical examination findings would you
expect in this patient?
A. Nodular mass within the testis
B. High-lying testis
C. Transilluminated fluid mass
D. Palpation of a soft bag of worms
Explanations
(u) A. Testicular cancer is characterized by a painless nodular enlargement of the testis typically discovered by the
patient.
(c) B. Testicular torsion occurs in the 10-20 year age group, and is characterized by acute onset of severe pain and
swelling of the testis. Physical examination will reveal a tender high-riding testis.
(u) C. A hydrocele is a collection of fluid within the scrotum which can be transilluminated on exam.
(u) D. A varicocele refers to varicose veins of the spermatic cord. It feels like a soft bag of worms in the scrotum
separate from the testis.
Ref: (3)



137. Clinical Therapeutics/Gastrointestinal/Nutritional
A 56 year-old diabetic male with a three-month history of epigastric pain after eating says he gets full really fast now.
His test for Helicobacter pylori is negative. Which of the following is the recommended medication to treat this
patient?
A. Calcium carbonate (Tums)
B. Ranitidine (Zantac)

Copyright 2010. Physician Assistant Education Association

55
C. Omeprazole (Prilosec)
D. Metoclopramide (Reglan)
Explanations
(u) A. Calcium carbonate has no effect on gastric emptying.
(u) B. H2 blockers have no effect on gastric emptying.
(u) C. Prilosec has no effect on gastric emptying.
(c) D. Metoclopramide accelerates gastric emptying and is the treatment of choice for this patient.
Ref: (28)




138. Clinical Intervention/Psychiatry/Behavioral Medicine
Which of the following is the most appropriate management of acute psychosis in a patient with schizophrenia?
A. Amitriptyline (Elavil)
B. Fluphenazine (Prolixin)
C. Lithium (Eskalith)
D. Sertraline (Zoloft)
Explanations
(u) A. See B for explanation.
(c) B. There are numerous options used in the management of acute psychosis including benzodiazepines and the
typical antipsychotics such as fluphenazine, haloperidol, and olanzapine. Antidepressants and lithium have no
benefit in the treatment of acute psychosis.
(u) C. See B for explanation.
(u) D. See B for explanation.
Ref: (33)



139. Health Maintenance/Pulmonology
An asymptomatic 60 year-old female with a 30 pack year history of smoking presents to the clinic requesting a chest
x-ray to check for lung cancer. Which of the following do you recommend?
A. A low-dose helical computed tomography (CT)
B. Carcinoembryonic antigen (CEA)
C. Serial chest radiographs to identify early stage malignancy
D. No testing and referral for smoking cessation
Explanations
(u) A. A low-dose helical computed tomography is very sensitive but is expensive, has increase false-positive tests
and increases over-diagnosis. Mortality benefit remains to be proved.
(u) B. CEA lacks clinical validation as a screening for lung cancer.
(u) C. No major advisory organization recommends serial radiograph screening for lung cancer.
(c) D. No major advisory organization recommends screening for lung cancer.
Ref: (28)



140. Diagnostic Studies/Obstetrics/Gynecology
Colposcopic examination of the cervix of a 38 year-old woman with a high-grade lesion on Papanicolaou (Pap) smear
yields a positive endocervical canal curettage (ECC) as its only abnormality. Which of the following is the most
appropriate next step in this patient?
A. Repeat the Pap smear in 3 months
B. Perform a conization of the cervix
C. Repeat the colposcopic examination in 3 months
D. No follow-up is required

Copyright 2010. Physician Assistant Education Association

56
Explanations
(u) A. See B for explanation.
(c) B. A conization of the cervix is recommended in this case because there is a substantial discrepancy between
the screening Pap test and the histologic data from biopsy and ECC. Additionally the ECC is positive for
disease in this case.
(u) C. See B for explanation.
(h) D. See B for explanation.
Ref: (4)




141. History & Physical/Cardiology
Which of the following would be expected on physical examination of a newborn diagnosed with Tetrology of Fallot?
A. Palpable right ventricular lift
B. Pulse discrepancy between arms and legs
C. Mid-diastolic murmur with opening snap
D. Polymorphous exanthema
Explanations
(c) A. Tetralogy of Fallot is commonly associated with a palpable right ventricular lift.
(u) B. Coarctation of the aorta is associated with a pulse discrepancy between the upper and lower extremities.
(u) C. A mid-diastolic murmur with an opening snap is heard in a patient with mitral stenosis, not Tetralogy of Fallot.
(u) D. Polymorphous exanthema is seen in patients with Kawasaki disease.
Ref: (13)



142. Clinical Therapeutics/Endocrinology
Which of the following is the rationale for the utilization of beta blocker therapy in a patient with hyperthyroidism?
A. Reduce goiter size
B. Alleviate symptoms
C. Regulate TSH
D. Normalize free T4
Explanations
(u) A. See B for explanation.
(c) B. Propranolol is used for symptomatic relief until the hyperthyroidism is resolved. It relieves tachycardia, tremor,
diaphoresis and anxiety.
(u) C. See B for explanation.
(u) D. See B for explanation.
Ref: (28)



143. Scientific Concepts/Orthopedics/Rheumatology
Which of the following types of hip fracture has the highest risk for avascular necrosis and nonunion?
A. Femoral neck
B. Intertrochanetric
C. Subtrochanteric
D. Greater trochanteric
Explanations
(c) A. Fractures involving the femoral neck typically disrupt the blood supply to the femoral head and may cause
avascular necrosis and nonunion.
(u) B. Intertrochanteric fractures usually do not cause avascular necrosis and nonunion due to the capsule and blood

Copyright 2010. Physician Assistant Education Association

57
supply to the femoral head remaining intact.
(u) C. Subtrochanteric fractures usually do not cause avascular necrosis and nonunion due to the capsule and blood
supply to the femoral head remaining intact.
(u) D. Greater trochanteric fractures usually do not cause avascular necrosis and nonunion due to the capsule and
blood supply to the femoral head remaining intact.
Ref: (28)



144. Health Maintenance/ENT/Ophthalmology
You are counseling a newly diagnosed type 2 diabetic about the need for referral to ophthalmology for a dilated
funduscopic exam. Which of the following best describes the rationale for referral?
A. He can wait until next year when he goes to get his refraction
B. He does not need to see an ophthalmologist if his Hemaglobin A1C is <6.0
C. Retinopathy is present in 20% of patient with type 2 Diabetes Mellitus at time of diagnosis
D. Your non-dilated exam can substitute for this referral
Explanations
(u) A. In type 2 diabetes, retinopathy is present in up to 20% of patients at diagnosis and may be the presenting
feature. Eye examination for vision usually does not require a dilated eye exam, and refraction is not calculated
with ophthalmoscope or direct visualization but by refractometer which does not examine the retina where
retinopathy occurs.
(u) B. See C for explanation.
(c) C. In type 2 diabetes, retinopathy is present in up to 20% of patients at diagnosis and may be the presenting
feature.
(u) D. See C for the explanation.
Ref: (28)



145. Clinical Therapeutics/Psychiatry/Behavioral Medicine
A patient is evaluated for an overblown sense of self-importance and sense of entitlement. He is described as being
arrogant, envious, exploitative and lacking in empathy. He is prone to mood swings and though he has a productive
career in business he has few meaningful personal relationships. Which of the following is the most appropriate
management for this patient?
A. Alprazolam (Xanax)
B. Clozaril (Clozapine)
C. Lithium (Lithobid)
D. Olanzapine (Zyprexa)
Explanations
(u) A. See C for explanation.
(u) B. See C for explanation.
(c) C. The diagnosis of narcissistic personality disorder is indicated in this scenario. His success in his career helps to
lessen the probability of schizophrenia and/or delusions of grandeur. Antipsychotics are therefore not indicated
and anxiolytics would not be helpful. Lithium can be used with this diagnosis if mood swings are prominent
along with antidepressants to address the frequent co-morbid depression.
(u) D. See C for explanation.
Ref: (33)



146. Diagnosis/Urology/Renal
A 54 year-old man comes to the urgent care because he was awoken suddenly from his sleep this morning with
severe left flank pain radiating to his left testicle with associated nausea and vomiting. He is afebrile and vital signs
are normal. He is constantly moving to find a comfortable position. On physical examination, left flank tenderness is
noted with no direct testicular tenderness. Urinalysis reveals a pH of 5.5 and microscopic hematuria, but is otherwise

Copyright 2010. Physician Assistant Education Association

58
unremarkable. Which of the following is the most likely diagnosis?
A. Acute epididymitis
B. Appendicitis
C. Diverticulitis
D. Nephrolithiasis
Explanations
(u) A. Acute epididymitis is characterized by symptoms of cystitis or urethritis (urethral discharge, pain at the tip of
the penis) with pain in the scrotum that may radiate to the flank.
(u) B. Appendicitis is associated with fever, abdominal pain and peritoneal signs.
(u) C. Diverticulitis presents with left lower quadrant pain that does not radiate to the testicle. The patient would not
have hematuria.
(c) D. Nephrolithiasis usually presents as a sudden onset of colicky flank pain with associated nausea and
vomiting. Urinalysis often reveals gross or microscopic hematuria.
Ref: (28)



147. Diagnostic Studies/Infectious Diseases
An 18 year-old female presents to the office with fever, fatigue and sore throat. Physical examination reveals an
erythematous pharynx, cervical lymphadenopathy and splenomegaly. CBC reveals an increased white blood cell count
with atypical lymphocytes, normal hemoglobin and hematocrit and normal platelet count. What additional laboratory test
will help you make the diagnosis in this patient?
A. Heterophile test
B. Lymph node biopsy
C. Serum antibody screening
D. Serum transaminase
Explanations
(c) A. The heterophile test is used for the diagnosis of infectious mononucleosis in children and adults.
(u) B. Lymph node biopsy would be done if lymphoma was suspected in this patient.
(u) C. Serum antibody screening could be done if one suspected Rubella or toxoplasmosis as the diagnosis in this case.
(u) D. Serum transaminases often are elevated in a patient with infectious mono, however, they are not diagnostic for the
disease.
Ref: (7)



148 Clinical Therapeutics/Cardiology
A 67 year-old male with history of mitral valve stenosis undergoes a mechanical valve replacement. Which of the
following is the appropriate duration of anticoagulation therapy if the patient has no other risk factors for
thromboembolic events or significant bleeding risks?
A. One month
B. Three months
C. Six months
D. Lifelong
Explanations
(u) A. See D for explanation.
(u) B. See D for explanation.
(u) C. See D for explanation.
(c) D. Patients with mechanical valves require lifelong anticoagulation to prevent thrombosis.
Ref: (28)

Copyright 2010. Physician Assistant Education Association

59
149. History & Physical/Gastrointestinal/Nutritional
What nail finding is most consistent with cirrhotic liver disease?
A. Clubbing
B. Pitting
C. Terry's nails
D. Beau's lines
Explanations
(u) A. Clubbing is a bulbous swelling of soft tissue at the nail base most commonly seen in congestive heart
disease, interstitial lung disease, lung cancer, irritable bowel syndrome, and malignancies.
(u) B. Pitting appears as punctuate depressions of the nail caused by defective layering of the superficial nail plate
by the proximal nail matrix. Pitting is usually associated with psoriasis.
(c) C. Terry's nails occur when the nail plate turns white with a ground glass appearance, a distal band of reddish
brown, and obliteration of the lunula. Terry's nails are seen in liver disease, usually cirrhosis.
(u) D. Beau's lines are transverse depressions of the nails and are usually bilateral and result from temporary
disruption of proximal nail growth from systemic illness.
Ref: (3)



150. History & Physical/Pulmonology
A 30 year-old male presents with sudden onset of chills, fever, chest pain and a cough productive of greenish-brown
sputum. On examination his temperature is 102 degrees F. He appears acutely ill and his respirations are shallow.
Chest x-ray demonstrates left lower lobe consolidation. Which of the following findings would most likely be present
on examination of his left lower lung?
A. Hyperresonance
B. Vesicular breath sounds
C. Increased tactile fremitus
D. Wheezing
Explanations
(u) A. Hyperresonance is an abnormal sound as a result of air trapping as in COPD.
(u) B. Vesicular breath sounds are the description of normal lung sounds.
(c) C. Increased tactile fremitus occurs in the presence of fluid or a lung consolidation such as lobar pneumonia.
(u) D. Wheezing is a musical noise sounding like a squeak caused by high velocity air flow through a narrow or
obstructed airway.
Ref: (3)



151. Clinical Intervention/Neurology
A 26 year-old female reports progressive distal to proximal spread of extremity weakness over the last 36 hours
without fever, headache or syncope. Examination reveals symmetrical, paresis of the hands and feet with loss of the
brachioradialis and Achillis reflexes. Biceps and knee reflexes are present but diminished. Sensory exam is normal.
What are the most likely findings on cerebral spinal fluid (CSF) analysis?
A. Decreased glucose, increased WBC count and decreased protein
B. Increased glucose, normal WBC count and normal protein
C. Normal glucose, decreased WBC count and elevated protein
D. Normal glucose, normal WBC count and elevated protein
Explanations
(u) A. See D for explanation.
(u) B. See D for explanation.
(u) C. See D for explanation.
(c) D. Guillain-Barre syndrome is typified by progressive symmetrical, distal to proximal spread of weakness and
areflexia without fever or sensory deficits. CSF analysis shows elevated protein due to axonal demyelination
but no glucose disturbances and no significant or sustained pleocytosis.
Ref: (7)

Copyright 2010. Physician Assistant Education Association

60
152. History & Physical/Obstetrics/Gynecology
Which of the following physical examination findings is present in a 12 week singleton pregnancy?
A. Chadwick's sign
B. Uterus palpable at the level of the umbilicus
C. Blood pressure lower than non-pregnant state
D. Hyperreflexia
Explanations
(c) A. Chadwick's sign is a bluish discoloration of the vagina early in pregnancy; it usually appears by 12 weeks of
gestation.
(u) B. The uterus is palpable at the level of the umbilicus at 20 weeks in a singleton pregnancy.
(u) C. Blood pressure lower than non-pregnant state occurs in the second trimester.
(u) D. Hyperreflexia is an uncommon finding and may occur with preeclampsia.
Ref: (4)



153. Clinical Intervention/ENT/Ophthalmology
A blacksmith presents to your clinic after feeling like something went in his eye while he was grinding on a piece of
metal. You stain the eye with Fluorescein and can visualize uptake with what appears like a deep abrasion. You are
unable to visualize any foreign body with your indirect ophthalmoscope. Although he makes an attempt, he complains
of some visual loss in that eye. His last Tetanus booster was 4 years ago. What is the most appropriate next step in
the management of this patient?
A. Administer tetanus
B. Prescribe Tetracaine ophthalmic
C. Provide reassurance only
D. Refer to ophthalmologist
Explanations
(u) A. Tetanus booster is only required every 5 years.
(u) B. See D for explanation.
(u) C. See D for explanation.
(c) D. Intraoccular foreign body requires emergency treatment by an ophthalmologist. Patients giving a history of
something hitting the eye, particularly while hammering on metal or using grinding equipment, must be
assessed for the possibility of intraoccular foreign body especially when no corneal foreign body is seen, a
corneal or scleral wound is apparent, or there is marked visual loss or media opacity.
Ref: (28)



154. History & Physical/Orthopedics/Rheumatology
Which of the following histories best describes spinal stenosis?
A. Gradual onset of back and thigh pain exacerbated by walking and alleviated by sitting
B. Acute onset of low back pain with radiation of pain to right foot, aggravated by sitting
C. Aching in bilateral buttocks with associated pain felt in shoulder or neck
D. Thigh pain aggravated by walking and absence of pedal pulses
Explanations
(c) A. Spinal stenosis typically presents as back pain and thigh pain aggravated by ambulation and relieved by
sitting.
(u) B. Spinal stenosis is gradual in onset and is improved by sitting.
(u) C. Spinal stenosis, unlike polymyalgia rheumatic, does not have associated shoulder or neck pain.
(u) D. True claudication will have thigh pain similar to spinal stenosis but can be differentiated by absence of pulses.
Spinal stenosis preserves distal pulses.
Ref: (28)



Copyright 2010. Physician Assistant Education Association

61
155. Clinical Intervention/Dermatology
A 36 year-old female comes to the office because a mole on her left calf has changed. On physical examination of
the left posterior lower leg, there is a 12 mm, asymmetrical, variegated blue-black macule with raised pink plaque in
the upper half of the lesion. Which of the following is the most appropriate clinical management of this lesion?
A. Cryosurgery
B. Topical retinoids
C. Excisional surgery
D. Topical chemotherapy
Explanations
(u) A. Physical destruction of suspected melanoma should never be used because histologic verification of the
diagnosis cannot be performed.
(u) B. Chronic use of topical retinoids may be effective for superficial solar keratoses.
(c) C. Surgical excision of suspected melanoma is necessary for histologic diagnosis and treatment of the lesion.
(u) D. Topical chemotherapy, such as 5-Fluorouracil, is an effective treatment for squamous cell carcinoma.
Ref: (10)



156. Diagnosis/Cardiology
A 60 year-old female with history of radiation therapy for the treatment of cancer presents with progressive dyspnea
and fatigue. On examination the patient has lower extremity edema, significant ascites, and an elevated jugular
venous pressure that does not fall with inspiration. Heart examination reveals a pericardial knock. Echocardiogram
shows rapid early filling and reduced mitral inflow velocities with inspiration. Which of the following is the most likely
diagnosis in this patient?
A. Pulmonary hypertension
B. Atrial myxoma
C. Constrictive pericarditis
D. Tako-Tsubo cardiomyopathy
Explanations
(u) A. Pulmonary hypertension is usually associated with chest pain, dyspnea, fatigue and syncope. Examination
would reveal a narrow splitting of S2 with a loud pulmonic component. Echocardiogram would show increased
pulmonary artery pressures, right ventricular enlargement and possibly paradoxical motion of the
intraventricular septum.
(u) B. Atrial myxoma is characterized by fever, weight loss, malaise, embolization, and a diastolic sound or murmur
(tumor plop) on examination. Atrial myxoma would be seen on echocardiogram.
(c) C. Constrictive pericarditis is associated with TB, radiation therapy, cardiac surgery, or following viral pericarditis.
There is evidence of right-sided heart failure, a positive Kussmaul sign, and a septal bounce and reduced
mitral inflow velocities with inspiration on echocardiogram.
(u) D. Tako-Tsubo cardiomyopathy (broken heart syndrome) is commonly seen in postmenopausal women who
experience signs and symptoms of acute coronary syndrome. Echocardiogram shows left ventricular apical
dyskinesia.
Ref: (28)




157. Clinical Therapeutics/Pulmonology
A 4 month-old infant is brought to the clinic by his mother with complaints of a cough for the past 3 weeks. Initially,
symptoms included running nose, sneezing and an irritating cough. Over the past week the cough has changed to
persistent staccato, paroxysmal forceful coughs ending with a loud inspiration. WBC is 20,0000/mcl with 72%
lymphocytes. Which of the following is the drug of choice for managing this patient?
A. Azithromycin (Zithromax)
B. Ceftriaxone (Rocephin)
C. Ampicillin (Unasyn)
D. Gentamicin (Garamycin)

Copyright 2010. Physician Assistant Education Association

62
Explanations
(c) A. Azithromycin terminates respiratory tract carriage of Bordetella pertussis.
(u) B. Ceftriaxone does not eradicate Bordetella pertussis.
(a) C. Ampicillin may be used for macrolide-intolerant patients however it is not the drug of choice.
(u) D. Gentamicin does not eradicate Bordetella pertussis.
Ref: (13)



158. Diagnostic Studies/Urology/Renal
A 28 year-old female comes to the office with fever, flank pain, and dysuria for the past two days. Which of the
following urinalysis results are suggestive of acute pyelonephritis?
A. Hyaline casts
B. Red cell casts
C. White cell casts
D. Granular casts
Explanations
(u) A. Hyaline casts may be found in concentrated urine and are not indicative of renal disease.
(u) B. Red cell casts are indicative of glomerulonephritis.
(c) C. White cell casts are indicative of renal infection or inflammation as seen in pyelonephritis or interstitial
nephritis.
(u) D. Granular casts are non-specific and may be seen in acute tubular necrosis.
Ref: (28)



159. Clinical Therapeutics/Gastrointestinal/Nutritional
What is the initial treatment of choice for inflammatory bowel disease?
A. Ondansetron (Zofran)
B. Sulfasalazine (Azulfidine)
C. Metronidazole (Flagyl)
D. Azothioprine (Imuran)
Explanations
(u) A. Zofran is an antiemetic and not indicated in the treatment of inflammatory bowel disease.
(c) B. Sulfasalazine and other 5-aminosalicylic acid drugs are the cornerstone of therapy in mild to moderate
inflammatory bowel disease as they have both anti-inflammatory and antibacterial properties.
(u) C. Metronidazole may have a role in Crohn's disease after ileal resection but has no role in the treatment of mild
to moderate disease.
(u) D. Azothioprine is used in severe, glucocorticoid-dependent inflammatory bowel disease.
Ref: (7)



160. Health Maintenance/Infectious Diseases
A patient is traveling to Africa on a church mission trip. The area he is traveling to has a high incidence of
chloroquine-resistant malaria. What drug is indicated for prophylaxis in this patient?
A. Atovaquone (Malarone)
B. Hydroxychloroquine sulfate (Plaquenil)
C. Quinine sulfate (Qualaquin)
D. Lumefantrine (Benflumetol)
Explanations
(c) A. Malarone is used for prophylaxis for malaria in areas with chloroquine- or mefloquine-resistant malaria.
(u) B. Plaquenil is used for primary prophylaxis only in areas with chloroquine-sensitive malaria.

Copyright 2010. Physician Assistant Education Association

63
(u) C. Quinine is an antimalarial drug used for treatment not prophylaxis.
(u) D. Lumefantrine is an antimalarial drug used for treatment not prophylaxis.
Ref: (7)



161. History & Physical/Neurology
What spinal nerve root is most likely affected in a patient with weak wrist extension, thumb and index finger
paresthesias and diminished triceps reflex?
A. Cervical 4
B. Cervical 5
C. Cervical 6
D. Cervical 7
Explanations
(u) A. See C for explanation.
(u) B. See C for explanation.
(c) C. In contrast, cervical 5 would be associated with deltoid and biceps weakness and diminished biceps reflex
while cervical 7 would result in triceps weakness and paresthesias in the middle finger and diminished
brachioradialis reflex.
(u) D. See C for explanation.
Ref: (1)



162. Diagnosis/Orthopedics/Rheumatology
A 13 year-old female presents to the office with right knee and thigh pain and the inability to bear weight since waking
yesterday morning. The mother states the child had a fever of 100.9 degrees F this morning and continues to be non-
weight bearing. Examination reveals a warm, erythematous, swollen knee. Which of the following tests would be most
beneficial in the diagnosis and treatment of this patient?
A. Plain x-ray of the femur
B. Antistreptolysin O titer
C. Magnetic resonance imaging (MRI) of the femur
D. Culture of joint aspirate
Explanations
(u) A. Plain x-rays may aid in diagnostic suspicion, usually after a week or two, but do not aid in treatment choice.
(u) B. ASO titer is not indicated in a patient with osteomyelitis.
(u) C. MRI will detect the early inflammation of osteomyelitis but does not aid in the treatment choice.
(c) D. A culture of the joint fluid will confirm the diagnosis and offer information regarding infectious agent.
Ref: (13)



163. Scientific Concepts/Cardiology
A 59 year-old male with history of hypertension and dyslipidemia presents with complaint of substernal chest pain for
two hours. The pain woke him from sleep, does not radiate, and is associated with nausea and diaphoresis.
Electrocardiogram reveals ST segment elevation in leads II, III, and AVF. Which of the following walls of the ventricle
is most likely at risk?
A. Anterior
B. Inferior
C. Lateral
D. Posterior
Explanations
(u) A. See B for explanation.
(c) B. Inferior wall myocardial infarction is characterized by ST segment elevation in leads II, III and AVF.

Copyright 2010. Physician Assistant Education Association

64
(u) C. See B for explanation.
(u) D. See B for explanation.
Ref: (7)



164. Health Maintenance/Pulmonology
A recent Haitian immigrant presents to the clinic for an employment physical examination before starting work at a
local hospital. The patient has a history of receiving bacilli Calmette-Guerin (BCG) vaccination. Screening for
tuberculosis for this employee should include which of the following tests?
A. Sputum induction
B. PPD skin test
C. Chest x-ray
D. No screening needed
Explanations
(u) A. Sputum induction should not be used as a screening test for tuberculosis.
(u) B. False-positive tuberculin skin test reactions can occur in persons previously vaccinated against M.
tuberculosis with BCG. PPD should be avoided as a screening test in these patients.
(c) C. Chest x-ray is the test of choice in patients where the PPD test is not indicated or in high-risk individuals.
(u) D. This patient has recently emigrated from a possible endemic region and should be screened for tuberculosis.
Ref: (28)


165. Clinical Therapeutics/ENT/Ophthalmology
A 33 year-old presents with sinusitis unresponsive to three various antibiotics over the past four months. Sinus
puncture is performed with culture positive for Aspergillus fumigatus. What is the most appropriate treatment for this
patient?
A. Amphotericin (Amphotericin B)
B. Amoxicillin/Clavulanate (Augmentin)
C. Ofloxacin (Floxin)
D. Cefuroximine (Ceftin)
Explanations
(c) A. Aspergillus fumigatus is a fungal organism therefore this patient would be most appropriately treated with
Amphotericin B which is an anti-fungal medication.
(u) B. See A for explanation.
(u) C. See A for explanation.
(u) D. See A for explanation.
Ref: (1)



166. History & Physical/Psychiatry/Behavioral Medicine
A patient with schizophrenia states that newspapers and the television are constantly talking about him though they
never mention his name. What term best fits this description?
A. Cosmic identity
B. Ideas of reference
C. Retrospective falsification
D. Thought broadcasting
Explanations
(u) A. See B for explanation.
(c) B. Ideas of reference are part of the abnormal thought content that is hallmark in schizophrenic patients. Cosmic
identity refers to a patient's belief that they have physically and mentally fused with the universe while thought
broadcasting is the belief that others can read the patient's mind or that their thoughts are broadcast over the
radio or similar media. A distorted memory filtered through a person's present emotional state refers to

Copyright 2010. Physician Assistant Education Association

65
retrospective falsification.
(u) C. See B for explanation.
(u) D. See B for explanation.
Ref: (14)



167. Clinical Intervention/Obstetrics/Gynecology
You are following a patient in labor at term. You evaluate the fetal monitoring tracing and note the presence of
variable, repetitive decelerations in the heart rates. The contractions have a sharp deceleration slope. What is the
recommended treatment for these decelerations?
A. Change maternal position
B. Administer tocolytic therapy
C. Apply vibroacustic stimulation
D. Perform cesarean section
Explanations
(c) A. Variable decelerations occur from umbilical cord compression and oligohydramnios. They are treated by
changing maternal positioning to relieve pressure on the umbilical cord. Additionally, amnioinfusion may be
used to relieve umbilical cord compression in cases of oligohydramnios.
(u) B. See A for explanation.
(u) C. See A for explanation.
(u) D. Delivery of the baby by c-section is not indicated in variable FHR decelerations and may be harmful to the
baby.
Ref: (4)



168. Clinical Therapeutics/Neurology
A 27 year-old female presents with 5-6 headaches monthly for the past year that are described as severe, throbbing
and initially unilateral. They is associated with nausea, photophobia and phonophobia. There have been no
concomitant sensory or motor deficits. Her physical examination and brain MRI are normal. What treatment would be
best to reduce the frequency of the headaches?
A. Sumatriptan (Imitrex)
B. Isometheptene (Midrin)
C. Propranolol (Inderal)
D. Hydrocodone (Vicodin)
Explanations
(u) A. See C for explanation.
(u) B. See C for explanation.
(c) C. Each medication listed can be used in the management of migraine headaches, however only propranolol is
indicated for reducing the frequency of migraine headaches.
(u) D. See C for explanation.
Ref: (1)



169. Diagnosis/Cardiology
A 24 year-old female presents complaining of palpitations described as occasional "skipped" beats. The patient
denies chest pain, lightheadedness, syncope, or dyspnea. On examination you note a midsystolic click without
murmur. Which of the following is the most likely diagnosis in this patient?
A. Mitral valve prolapse
B. Aortic stenosis
C. Atrial septal defect
D. Pulmonic stenosis

Copyright 2010. Physician Assistant Education Association

66
Explanations
(c) A. Patients with mitral valve prolapse will often present with complaint of palpitations. Auscultation would reveal
a mid-systolic click with or without a late systolic murmur.
(u) B. Aortic stenosis presents with a systolic murmur with no click. Patients may complain of chest pain, dyspnea
or syncope.
(u) C. Atrial septal defect is not associated with a midsystolic click.
(u) D. Pulmonic stenosis is not associated with a midsystolic click.
Ref: (28)


170. History & Physical/Orthopedics/Rheumatology
A 14-year old male active in sports, has been complaining of intermittent anterior right knee pain for several months.
He denies any specific injuries. On examination, there is no erythema, swelling, deformities, joint laxity, or crepitus.
Palpation reveals tenderness over the tibial tubercle and bursa of the right knee. This finding is characteristic of which
of the following disorders?
A. Discoid meniscus
B. Osgood-Schlatter disease
C. Chondromalacia patella
D. Osteochondritis dissecans
Explanations
(u) A. Discoid meniscus causes clicking over the lateral meniscus during flexion. This disorder is mostly painless
and occasionally may cause mild aching or effusion.
(c) B. Osgood-Schlatter disease causes pain at the tibial tubercle and it is caused by fragmentation of the tip of the
proximal tibial physis.
(u) C. Chondromalacia patella may demonstrate exaggerated knee valgus and subluxation.
(u) D. Osteochondritis dissecans in older children may have effusions, pain, and locking of the joint caused by a
portion of the joint surface softening and a shearing leading to a loose fragment.
Ref: (26)



171. Clinical Intervention/Urology/Renal
A 28 year-old man comes to the office for his yearly follow up of testicular cancer. He was successfully treated with
an orchiectomy two years ago. Which of the following tumor markers can be used to identify early relapse in testicular
cancer?
A. Carcinoembryonic antigen (CEA)
B. Prostate specific antigen (PSA)
C. CA 125 glycoprotein
D. Alpha fetoprotein (AFP)
Explanations
(u) A. Carcinoembryonic antigen can be elevated in multiple conditions including colon cancer, gastritis, peptic ulcer
disease, diverticulitis, liver disease, chronic obstructive pulmonary disease, and diabetes.
(u) B. Prostate specific antigen is a sensitive marker for prostate cancer.
(u) C. CA 125 glycoprotein is a marker in epithelial ovarian cancer and can be used for post-treatment surveillance.
(c) D. Periodic surveillance of alpha fetoprotein is a sensitive method for detecting early relapse of testicular cancer.
Ref: (28)



172. Clinical Therapeutics/Pulmonology
A 6 year-old boy is brought to the pediatric clinic by his mother for an evaluation of his asthma. He coughs about 3
days out of the week with at least 2-3 nights of coughing. Which of the following would be the most appropriate
treatment for this patient?
A. Mast cell stabilizer
B. Long acting beta agonist

Copyright 2010. Physician Assistant Education Association

67
C. Leukotriene receptor antagonist
D. Low dose inhaled corticosteroid
Explanations
(u) A. A mast cell stabilizer is an alternative treatment but not the preferred treatment.
(u) B. Long acting beta agonist can be used as adjunctive therapy with an anti-inflammatory.
(u) C. Leukotriene receptor antagonists are an alternative treatment but not the preferred treatment.
(c) D. Low dose inhaled corticosteroids are the preferred treatment for mild persistent asthma.
Ref: (13)



173. Diagnosis/Gastrointestinal/Nutritional
A 68 year-old male with a history of alcohol abuse presents with coffee-ground emesis. He denies vomiting prior to
this episode. What is the most likely cause of his bleeding?
A. Erosive gastritis
B. Gastric neoplasm
C. Mallory-Weiss tear
D. Zenker diverticulum
Explanations
(c) A. Drugs and alcohol are the most common causes of upper gastrointestinal bleeding.
(u) B. Gastric neoplasm is also uncommon (less than 1% of upper GI bleeding) but should be ruled out).
(u) C. Mallory-Weiss tears are lacerations of the gastroesophageal junction causing 5-10% of cases of upper GI
bleeding. Many patients report a history of heavy alcohol use or retching.
(u) D. Peptic ulcer disease is a common cause of upper GI bleeding but is becoming less common with the
increasing irradication of H. pylori infections.
Ref: (28)




174. Clinical Therapeutics/Psychiatry/Behavioral Medicine
What class of medications has been found most beneficial in the treatment of alcoholism not associated with a
concomitant psychiatric illness?
A. Antipsychotics
B. Benzodiazepines
C. Melatonin agonists
D. Selective serotonin reuptake inhibitors
Explanations
(u) A. See B for explanation.
(c) B. Though no pharmacologic therapies have substantial supportive data in treating alcoholism not associated with
a mood or anxiety disorder short term use of benzodiazepines has the greatest benefit especially with acute
cessation of alcohol.
(u) C. See B for explanation.
(u) D. See B for explanation.
Ref: (33)



175. Diagnostic Studies/Endocrinology
Which of the following laboratory values would be noted in a patient with primary hyperthyroidism?
A. Increased TSH, increased free T4
B. Increased TSH, decreased free T4
C. Decreased TSH, increased free T4
D. Decreased TSH, decreased free T4

Copyright 2010. Physician Assistant Education Association

68
Explanations
(u) A. See C for explanation.
(u) B. See C for explanation.
(c) C. Serum TSH is depressed and serum T3, T4 and uptake are usually all increased in patients with primary
hyperthyroidism.
(u) D. See C for explanation.
Ref: (28)



176. Diagnosis/Infectious Diseases
A 16 year-old female presents to the office with a rash. She was healthy until one week ago when she developed a
fever, headache, generalized lymphadenopathy and a rash on her trunk. On physical examination you note an
erythematous rash with central clearing. Her left knee is swollen with painful range of motion. She recently returned
from Girl Scout Camp. Which of the following is the most likely diagnosis?
A. Rocky Mountain spotted fever
B. Lyme disease
C. J uvenile rheumatoid arthritis
D. Babesiosis
Explanations
(u) A. RMSF usually presents in the early part of the disease with fever, headache, malaise, myalgia, and nausea
and vomiting. This is often unable to be distinguished from other viral illnesses. The patient then becomes
progressively more ill with vascular infection and injury occuring.
(c) B. This presentation is classic for stage 2 (disseminated infection) Lyme disease. Commonly patients have skin
involvement, headache, mild stiffness of the neck, fever, chills, migratory musculoskeletal pain, arthralgias,
malaise, and fatigue. Lyme disease is the most common vector borne disease in the US and Europe.
(u) C. J uvenile rheumatoid arthritis is a persistent non infectious arthritis lasting more than 6 weeks to 3 months
after other possible etiologies have been ruled out.
(u) D. Babesiosis is a tick borne illness that presents with the most common symptoms being fever, fatigue,
malaise, sweats, shaking chills and myalgias.
Ref: (7)



177. Health Maintenance/Cardiology
A 10 year-old female experiences fever and polyarthralgia. On examination you note a new early diastolic murmur.
Laboratory results are positive for antistreptolysin O. The patient has no known drug allergies. Which of the following
is the recommended prophylaxis for this condition?
A. Doxycycline
B. Erythromycin
C. Benzathine penicillin G
D. Trimethoprim/sulfamethoxazole
Explanations
(u) A. Doxycycline and Bactrim are not indicated for the prophylaxis of recurrent rheumatic fever.
(u) B. Erythromycin is considered second line for prophylaxis of recurrent rheumatic fever in a patient with a penicillin
allergy.
(c) C. Recurrences of rheumatic fever are most common in patients who have had carditis during their initial episode
and in children. The preferred method of prophylaxis is Benzathine penicillin G every four weeks.
(u) D. See A for explanation.
Ref: (28)



178. Clinical Intervention/Orthopedics/Rheumatology
A patient is 12 hours post-closed reduction of a tibial fracture and is in a long leg cast. Despite pain medication the
patient complains of unrelieved pain and cannot move his toes. Which of the following is the most appropriate clinical

Copyright 2010. Physician Assistant Education Association

69
intervention in this patient?
A. Elevate the leg
B. Bivalve the cast
C. Encourage ambulation
D. Place on PCA pump
Explanations
(u) A. This measure may be used while the necessary equipment is obtained to bivalve the cast, but it will not
treat the underlying condition.
(c) B. The cast for this patient is too tight and the pressure needs to be released. Bivalving the cast is the best
option for this patient.
(u) C. The cast for this patient is too tight and the pressure needs to be released. Ambulation is not appropriate in
this patient.
(u) D. Increasing the patient's pain medication will not treat the underlying condition.
Ref: (27)



179. Diagnosis/ENT/Ophthalmology
Which of the following is most appropriate to rule out vertebrobasilar insufficiency as a cause of recurrent vertigo?
A. Carotid Doppler
B. Dix-Hallpike maneuver
C. Epply maneuver
D. Magnetic resonance imaging
Explanations
(u) A. Central vertigo can be caused by cerebral ischemia. Since this stems from the vertebral basilar distribution it
would not be identifiable on carotid ultrasound studies.
(u) B. Dix-Hallpike maneuver is used as a reproducing test for positional vertigo not vertebrobasillar insufficiency.
(u) C. Epply maneuver is used a treatment for benign paroxysmal positioning vertigo.
(c) D. MRI is the most appropriate of the listed screening tools to differentiate vertebrobasilar insufficiency from other
etiologies.
Ref: (1)


180. Scientific Concepts/Pulmonology
A 13 year-old male with known cystic fibrosis presents to the emergency department, accompanied by his parents,
with increased coughing, wheezing and low grade fever. Rales are audible on auscultation of the lungs. Treatment
should target which of the following organisms?
A. Pneumocystis jiroveci
B. Haemophilus influenza
C. Mycoplasma pneumoniae
D. Pseudomonas aeruginosa
Explanations
(u) A. Pneumocystis jiroveci is common in HIV patients, not patients with cystic fibrosis.
(u) B. Staphylococcus aureus and Haemophilus Influenzae often begin in the first few months of life even in
asymptomatic infants with cystic fibrosis, but are less common in older children.
(u) C. Mycoplasma pneumoniae is a less common cause of pneumonia in patients with cystic fibrosis.
(c) D. Pseudomonas aeruginosa is the most predominant pathogen in patients with cystic fibrosis.
Ref: (13)

Copyright 2010. Physician Assistant Education Association

70
181. Clinical Therapeutics/Urology/Renal
A 68 year-old woman comes to the office for evaluation of urinary incontinence. For the past few months, she has
had an intense urgency to urinate, followed by leakage of urine. Which of the following is the most appropriate
intervention for this patient?
A. Intravaginal estrogen cream (Premarin cream)
B. Terazosin (Hytrin)
C. Oxybutynin (Ditropan)
D. Intravaginal miconazole cream (Monistat cream)
Explanations
(u) A. Intravaginal estrogen is indicated for women with vaginal atrophy that is contributing to stress incontinence.
(u) B. Terazosin is indicated for men with urinary incontinence caused by urethral obstruction due to benign
prostatic hyperplasia.
(c) C. Urge incontinence is caused by detrusor overactivity that causes urinary leakage. Antimuscarinics, such as
oxybutynin, are prescribed to relax the pelvic muscles.
(u) D. Duloxetine is effective in reducing the number of stress incontinence episodes in women.
Ref: (28)



182. Health Maintenance/Obstetrics/Gynecology
What is the recommended initial first trimester screening test for fetal aneuploidy?
A. Amniocentesis
B. Pregnancy-associated plasma protein A (PPA), beta-hCG, and ultrasound of nuchal transparency
C. Maternal serum alpha feto protein, beta-hCG, estriol, and inhibin-A
D. Level II ultrasound
Explanations
(u) A. Amniocentesis is a diagnostic test not a screening test.
(c) B. Pregnancy-associated plasma protein A (PPA), beta-hCG, and ultrasound of nuchal transparency are
screening tests done at 10-13 weeks of gestation.
(u) C. Maternal serum alpha feto protein, beta hCG, estriol, and inhibin-A, are included in the quad screen which is a
screening test done in the second trimester from 15-20 weeks of gestation.
(u) D. Level II ultrasound is done in the second trimester.
Ref: (4)



183. Clinical Intervention/Cardiology
A 26 year-old patient is brought to the emergency department after a head on collision. The patient complains of
chest pain, dyspnea and cough. Examination reveals the patient to be tachypneic and tachycardic with a narrow
pulse pressure. J ugular venous distension is noted. Electrocardiogram reveals nonspecific t wave changes and
electrical alternans. Which of the following is the most appropriate management plan for this patient?
A. Serial echocardiograms
B. Pericardiocentesis
C. Cardiac catheterization
D. Pericardiectomy
Explanations
(u) A. Serial echocardiograms would be indicated if a patient had a small pericardial effusion and no intervention
was immediately needed. This patient has signs and symptoms of cardiac tamponade and needs immediate
intervention.
(c) B. Urgent pericardiocentesis is the initial treatment of choice in a patient with cardiac tamponade.
(u) C. There is no indication for cardiac catheterization in the management of cardiac tamponade.
(u) D. A partial pericardiectomy may be needed in patients with recurrent pericardial effusions that occur secondary
to neoplastic disease and uremia, but there is no indication for partial pericardiectomy in the acute
management of cardiac tamponade.
Ref: (28)

Copyright 2010. Physician Assistant Education Association

71
184. History & Physical/Dermatology
A 23 year-old male comes to the office with an intensely itchy rash, especially at night for the past week. His
roommates have similar symptoms but have not been evaluated. On physical examination, the wrists, web spaces of
the hands, and axillae have thin linear markings, tiny vesicles, and excoriations. Which of the following is the most
likely diagnosis?
A. Pediculosis
B. Folliculitis
C. Impetigo
D. Scabies
Explanations
(u) A. Pediculosis is a lice infection of the scalp, trunk or pubic region. Body louse occurs in persons living in
overcrowded residences, and is characterized by intense pruritis with resultant deep excoriation of the neck,
shoulders, and upper back. Nits are identifiable on hair shafts.
(u) B. Folliculitis is an infection of the upper hair follicles characterized by itching, burning and pustules in the hairy
regions of the body.
(u) C. Impetigo is a contagious infection of the skin caused by staphylococci or streptococci characterized by honey-
crusted erythematous erosions of the face or other exposed areas of the body.
(c) D. Scabies is an infestation of the mite Sarcoptes scabiei. It is usually spread by skin-to-skin contact. Scabies is
characterized by intense pruritic burrows, vesicles and excoriations of the finger webs, wrist creases and
axillae.
Ref: (10)



185. Diagnosis/Neurology
A 55 year-old male presents with a three month history of progressive weakness in the extremities without associated
sensory changes or deficits. Examination reveals widespread muscle atrophy, fasciculations and bilateral hyperactive
reflexes with Babinski sign. Which of the following is the most likely diagnosis?
A. Amyotrophic lateral sclerosis
B. Polymyalgia rheumatica
C. Myasthenia gravis
D. Multiple sclerosis
Explanations
(c) A. Though some variability in presentation does occur the characteristic progressive weakness without sensory
changes and upper and lower motor nerve dysfunction is a hallmark of amyotrophic lateral sclerosis (ALS).
(u) B. See A for explanation.
(u) C. See A for explanation.
(u) D. See A for explanation.
Ref: (7)



186. Clinical Therapeutics/Infectious Diseases
A 5 year-old presents with perianal pruritus that is worse at night. Scotch tape is positive for Enterobius vermicularis.
The treatment of choice is
A. mebendazole (Vermox).
B. metronidazole (Flagyl).
C. clindamycin (Cleocin).
D. thiabendazole (Mintezol).
Explanations
(c) A. Treatment of choice for pinworms is mebendazole.
(u) B. Metronidazole is used in the treatment of amebic dysentery and trichomoniasis.
(u) C. Clindamycin is used in the treatment of anaerobic bacterial infections.
(u) D. Thiabendazole is used in the treatment of helminth infections.
Ref: (5)

Copyright 2010. Physician Assistant Education Association

72
187. Clinical Intervention/Psychiatry/Behavioral Medicine
A 17 year-old female is seen who has a history of eating large amounts of food at night 3 to 4 times weekly. These
episodes are always followed by extreme guilt and either induced vomiting or hours of strenuous exercise. She
excels at work and school and maintains many active relationships. Her physical examination reveals a normal BMI
and an otherwise normal exam. What is the best initial intervention for this patient?
A. Begin an anxiolytic
B. Immediate hospitalization
C. Start an antidepressant
D. Test for substance abuse
Explanations
(u) A. See C for explanation.
(u) B. See C for explanation.
(c) C. Uncomplicated bulimia generally does not require hospitalization as the physical manifestations
encountered in bulimia are generally mild if present at all. Some patients with bulimia have concomitant
substance abuse issues but her successful relationships and work activities lessen the probability.
Antidepressants, not anxiolytics, have been found helpful in lessening the binge/purge cycles and
improving overall well-being even outside of comorbid mood disorders.
(u) D. See C for explanation.
Ref: (33)



188. Scientific Concepts/Gastrointestinal/Nutritional
Prolonged use of a proton pump inhibitor can lead to low levels of which of the following nutrients?
A. Folic acid
B. Magnesium
C. Vitamin B6
D. Vitamin B12
Explanations
(u) A. See D for explanation.
(u) B. See D for explanation.
(u) C. See D for explanation.
(c) D. Vitamin B12 requires gastric acid for absorption in the stomach. Prolonged use of a proton pump inhibitor
suppresses gastric acid production.
Ref: (21)



189. Health Maintenance/Cardiology
A 40 year-old G3P3003 female presents complaining of dull aching discomfort of her lower extremities, which is
worse in the evening. The patient currently works as a waitress. Examination reveals dilated, tortuous veins beneath
the skin in the thigh and leg bilaterally. Which of the following is the best initial approach to prevent progression of
disease and complications in this patient?
A. Compression stockings
B. Warfarin (Coumadin) therapy
C. Sclerotherapy
D. Clopidogrel (Plavix)
Explanations
(c) A. Graduated compression stockings can be used in patients with early varicosities to prevent progression of the
disease and when used with leg elevation complications from varicose veins can be avoided.
(u) B. There is no indication for warfarin or clopidogrel therapy in patients with varicose veins.
(u) C. Sclerotherapy is not the best initial choice to prevent disease progression and complications.
(u) D. See B for explanation.
Ref: (28)


Copyright 2010. Physician Assistant Education Association

73
190. History & Physical/Pulmonology
A 69 year-old male presents with complaint of increasing dyspnea over the past 6-8 months. The patient denies
cough, chest pain or smoking history. Physical examination reveals inspiratory crackles at the bases and clubbing of
the nails. Chest x-ray reveals interstitial fibrosis of the lower lungs, thickened pleura and calcified pleural plaques of
the lateral chest wall. Pulmonary function testing shows a restrictive pattern with a decreased diffusing capacity. What
information is most likely noted in this patient's history?
A. Coal mining
B. Silica exposure
C. Textile work
D. Asbestos exposure
Explanations
(u) A. Patients with coal miners pneumoconiosis are typically asymptomatic with unremarkable pulmonary function
tests. CXR shows small opacities in the upper lungs.
(u) B. Patients with a history of silica exposure are also asymptomatic and have unaffected pulmonary function
tests. CXR shows small rounded opacities throughout the lung and calcified hilar lymph nodes.
(u) C. Textile workers present with an asthma-like disorder with chest tightness, cough and dyspnea that is worse
on the first day back to work and improves as the week goes on.
(c) D. Asbestos exposure often presents years later with increasing dyspnea and interstitial fibrosis of the lower
lungs, thickened pleura and calcified pleura plaques. They will have a restrictive pattern on PFT.
Ref: (28)



191. Clinical Therapeutics/ENT/Ophthalmology
A 54 year-old type 2 diabetic male presents for follow up evaluation of previously diagnosed persistent otitis externa.
Early in the disease process, a CT scan was obtained secondary to non-improvement on antibiotics. Results showed
osseous erosion of the floor of the ear canal. He has been on ciprofloxacin 1000mg twice daily for two months since
the CT scan and currently has no further edema, erythema, or exudate from the external auditory canal or
surrounding tissue. Which of the following is an appropriate treatment plan?
A. Continue prophylactic antibiotics for an additional 6 weeks
B. Immediately discontinue antibiotics
C. Obtain gallium scan to ensure reduction of inflammatory process
D. Skin swab culture of healthy tissue
Explanations
(u) A. See C for explanation.
(u) B. See C for explanation.
(c) C. Treatment of malignant external otitis requires prolonged antipseudomonal antibiotic administration often for
several months. To avoid relapse, antibiotics should be continued even in the asymptomatic patient, until
gallium scanning indicates a marked reduction in the inflammatory process.
(u) D. See C for explanation.
Ref: (28)



192. Clinical Intervention/Endocrinology
A person with type 2 diabetes who requires insulin brings in a home monitoring record for review. The current insulin
regimen is 15 units of NPH with 5 units of regular in the morning, and 10 units of NPH with 5 units of regular in the
evening. A trend of elevated blood sugar readings occurring at noon should prompt an insulin increase of which of the
following?
A. Morning NPH dose
B. Morning regular dose
C. Evening NPH dose
D. Evening regular dose
Explanations
(u) A. The effectiveness of the morning NPH dose would be reflected by the 5 PM sugar reading.
(c) B. The elevated noon blood sugar reading indicates a need to increase the morning regular dose.

Copyright 2010. Physician Assistant Education Association

74
(u) C. The morning glucose reading would reflect the effectiveness of the evening NPH.
(u) D. The evening (10 PM) reading would reflect the effect of the evening regular dose.
Ref: (28)



193. Diagnostic Studies/Obstetrics/Gynecology
In which of the following conditions would human chorionic gonadotropin (hCG) level be lower than expected for
gestational age?
A. Choriocarcinoma
B. Hydatidiform mole
C. Ectopic pregnancy
D. Twin gestation
Explanations
(u) A. Choriocarcinoma is a persistent form of gestational trophoblastic neoplasia and will have an elevated hCG
level.
(u) B. Quantitative hCG levels are excessively elevated for the gestational age of the pregnancy.
(c) C. An ectopic pregnancy will have an abnormally low hCG level because the hCG will not double every 48 hours
as a normal pregnancy would.
(u) D. A twin gestation will have a higher hCG level because of the presence of two fetuses.
Ref: (4)



194. Clinical Therapeutics/Urology/Renal
A 26 year-old woman comes to the office for her first prenatal visit at 9 weeks gestation. During evaluation, her
urinalysis reveals asymptomatic bacteriuria. Which of the following antibiotics is the preferred treatment in this
patient?
A. Doxycycline (Vibromycin)
B. Trimethoprim (Monotrim)
C. Nitrofurantoin (Macrobid)
D. Erythromycin (Ery-tab)
Explanations
(h) A. Doxycycline is contraindicated during pregnancy because of dental staining in the exposed child.
(u) B. Trimethoprim is a folic acid antagonist, and should be avoided during organogenesis in the first trimester.
(c) C. Nitrofurantoin is an effective treatment of asymptomatic bacturia in the pregnant woman.
(u) D. Erythromycin is an ineffective treatment for asymptomatic bacteriuria.
Ref: (8)



195. History & Physical/Gastrointestinal/Nutritional
What physical examination finding is most specific for acute cholecystitis?
A. Psoas sign
B. Rovsing's sign
C. Murphy's sign
D. Cullen's sign
Explanations
(u) A. See C for explanation.
(u) B. See C for explanation.
(c) C. A sharp increase in tenderness with a sudden stop in inspiratory effort constitutes a postive Murphy's sign
and is most specific for cholecystitis.
(u) D. See C for explanation.
Ref: (3)

Copyright 2010. Physician Assistant Education Association

75
196. Clinical Therapeutics/Orthopedics/Rheumatology
Which of the following medications is the initial treatment of choice for suppressing the progression of rheumatoid
arthritis (RA)?
A. Naproxen sodium (Naprosyn)
B. Prednisone
C. Methotrexate (Rheumatrex)
D. Gold salts
Explanations
(u) A. Antiinflammatory medication may help with the symptoms but does not aid in suppression of RA.
(u) B. Corticosteroids are useful in the treatment of symptoms but does not aid in the suppression of disease.
(c) C. Methotrexate is the initial treatment choice for RA and aids in suppression of disease.
(u) D. Gold salts can be used for an acute RA flare, but do not suppress of the progression of the disease.
Ref: (28)



197. Diagnostic Studies/Cardiology
A 60 year-old male with history of hypertension presents for routine physical examination. He has no current
complaints. Vital signs are BP of 136/70 mmHg, HR 60 bpm, regular, RR 14, unlabored. Heart shows regular rate
and rhythm with no S3, S4 or murmur, Lungs are clear to auscultation bilaterally, and the abdomen is soft, nontender.
There is a 5cm palpable pulsatile abdominal mass noted. Which of the following is the best initial diagnostic study in
this patient?
A. Magnetic resonance imaging (MRI)
B. Arteriography
C. Ultrasound (US)
D. Plain film
Explanations
(u) A. MRI, arteriography, and abdominal flat plate are not indicated in the initial diagnostic evaluation of a patient
with a suspected abdominal aortic aneurysm.
(u) B. See A for explanation.
(c) C. Abdominal ultrasound is the diagnostic study of choice for the initial diagnosis of an abdominal aortic
aneurysm.
(u) D. See A for explanation.
Ref: (28)



198. Clinical Therapeutics/Pulmonology
What is the mechanism of action of salmeterol (Serevent) in the treatment of asthma?
A. Anti-inflammatory
B. Immunotherapy for specific allergens
C. Relaxing of bronchial smooth muscle
D. Reduction of leukotriene production
Explanations
(u) A. See C for explanation.
(u) B. See C for explanation.
(c) C. The mechanism of action for salmeterol is the relaxation of bronchial smooth muscle.
(u) D. See C for explanation.
Ref: (28)

Copyright 2010. Physician Assistant Education Association

76
199. Diagnosis/ENT/Ophthalmology
A 62 year-old male chronic smoker is examined for slowly progressive non-painful vision loss. Visual acuity showed
20/150 in both eyes. Extra-ocular muscles are intact. Pupils are responsive to direct and consensual stimuli however
appear hazy. During funduscopic exam you are unable to visualize the optic cup, disk or the vessels. What is the
most appropriate diagnosis?
A. Anterior uveitis
B. Narrow angle glaucoma
C. Retinal detachment
D. Senile cataract
Explanations
(u) A. See D for explanation.
(u) B. See D for explanation.
(u) C. See D for explanation.
(c) D. Patients with cataract have gradually progressive blurred vision without pain or redness. Lens opacification
usually develops and can be grossly visible. Cataracts are the leading cause of blindness worldwide. They
usually occur bilaterally. Senile cataracts is by far the most common type with most persons over the age of 60
having some degree of lens opacity. Cigarette smoking increases the risk of cataract formation.
Ref: (28)



200. Clinical Therapeutics/Obstetrics/Gynecology
A 47 year-old perimenopausal female with vasomotor symptoms complains of vulvar itching and copious vaginal
discharge with a rancid odor. Physical examination reveals erythema of the vulva and petechiae on the cervix. The
pH of the vaginal discharge is five. Which of the following is the recommended treatment for this patient?
A. Topical metronidazole (Metrogel)
B. Oral fluconazole (Diflucan)
C. Topical estradiol (Estrace) cream
D. Oral metronidazole (Flagyl)
Explanations
(u) A. See D for explanation.
(u) B. See D for explanation.
(u) C. See D for explanation.
(c) D. This is a classic description of trichomonas vulvovaginitis. This condition must be treated with oral
metronidazole or tinidazole.
Ref: (4)



201. Clinical Intervention/Orthopedics/Rheumatology
A 30 year-old female presents with left wrist pain after slipping on the ice while walking to her car. On examination,
pain is noted on palpation over the anatomical snuff box. X-ray of her wrist shows no identifiable fracture. Which of
the following is the most appropriate treatment in this patient?
A. No treatment necessary
B. ACE wrap application
C. Splint application
D. Immediate orthopedic referral
Explanations
(u) A. See C for explanation.
(u) B. See C for explanation.
(c) C. Tenderness in the snuff box should be treated as a suspected scaphoid fracture. The patient should be
treated as if it is fractured and placed in a splint with a referral to an orthopedic specialist for further
evaluation and repeat imaging.
(u) D. See C for explanation.
Ref: (26)

Copyright 2010. Physician Assistant Education Association

77
202. Scientific Concepts/Psychiatry/Behavioral Medicine
In what region of the brain are cerebrovascular lesions most likely to cause post-event depressive symptoms?
A. Cerebellum
B. Occipital
C. Pons
D. Temporal
Explanations
(u) A. See D for explanation.
(u) B. See D for explanation.
(u) C. See D for explanation.
(c) D. Cerebrovascular lesions in the anterior brain regions are more common than the posterior areas. The temporal
lobe and the diencephalon are especially prone to post event depression.
Ref: (33)



203. Diagnosis/Cardiology
A 70 year-old female with history of hypertension, diabetes, and hypothyroidism presents with complaint of sudden
onset of left lower extremity pain. Examination reveals a cool left lower extremity with a mottled appearance. Dorsalis
pedis and posterior tibialis pulses are absent. Which of the following is the most likely diagnosis?
A. Acute arterial occlusion
B. Thromboangiitis obliterans
C. Deep vein thrombosis
D. Peripheral neuropathy
Explanations
(c) A. Acute arterial occlusion presents with sudden onset of extremity pain, with absent or diminished pulses. The
extremity will be cool to the touch and have a mottled appearance.
(u) B. Thromboangitis obliterans occurs in younger patients and primarily effects the distal extremities, especially
the toes. It is typically secondary to smoking.
(u) C. Deep vein thrombosis presents with lower extremity pain and edema. Pulses would be intact.
(u) D. Patients with peripheral neuropathy would have diminished sensation. They would not have a mottled
appearance and pulses would be intact.
Ref: (28)



204. Diagnostic Studies/Urology/Renal
A 54 year-old woman with history of lupus comes to the office with increasing significant peripheral edema over the
past four days. Laboratory findings include marked proteinuria, hypoalbuminemia and hyperlipidemia. Which of the
following diagnostic studies is the best for determining the cause of the proteinuria?
A. Renal ultrasound
B. Renal biopsy
C. Cystoscopy
D. Computed tomography scan
Explanations
(u) A. Renal ultrasound may identify hydronephrosis from a stone or other source of obstruction.
(c) B. Renal biopsy is performed in adults with new onset of nephrotic syndrome to determine the cause of the
proteinuria and to guide management decisions.
(u) C. Cystoscopy can be used in the evaluation of hematuria to assess for bladder or urethral neoplasm, benign
prostatic hyperplasia, and radiation or chemical cystitis.
(u) D. CT scanning may identify neoplasms of the kidney or ureter as well as benign conditions such as urolithiasis.
Ref: (28)



Copyright 2010. Physician Assistant Education Association

78
205. Clinical Therapeutics/Gastrointestinal/Nutritional
For patients undergoing highly emetogenic chemotherapy regimens, which of the following would be the first-line
antiemetic medication?
A. Ondansetron (Zofran)
B. Lorazepam (Ativan)
C. Dronabinol (Marinol)
D. Prochlorperazine (Compazine)
Explanations
(c) A. A 5HT3 receptor blocker such as ondansetron is the agent of choice, given its high efficacy and low risk of side
effects (headache being the major, and usually easily manageable, one).
(u) B. Benzodiazepines (lorazepam) are associated with significant psychologic side effects including sedation and
amnesia, although its anxiolytic properties make it a good choice for patients suffering from anticipatory
nausea.
(u) C. Cannabinoids (dronabinol) likewise have significant psychogenic side effects.
(u) D. The side effect profile of phenothiazides (prochlorperazine) includes hypotension, restlessness, sedation, and
extrapyramidal symptoms.
Ref: (21)



206. Clinical Intervention/Dermatology
A 26 year-old female comes to the office for evaluation of a painful lump on her right buttock for the past week.
Initially, it was a firm, tender nodule that has increased in size and tenderness in the past two days. On physical
examination of the right buttock, there is a 3-cm fluctuant tender red nodule. Which of the following is the most
appropriate initial intervention?
A. Incision and drainage
B. Mupirocin ointment
C. Systemic antibiotics
D. Moist compresses
Explanations
(c) A. Incision and drainage is the mainstay of therapy for abscesses.
(u) B. Individuals may be chronic carriers of S. aureus. Mupirocin ointment is effective in eliminating nasal carriage
of S. aureus.
(u) C. In healthy individuals, incision and drainage is generally adequate therapy for abscesses. Systemic antibiotics
may speed the healing of the tissue, and is instituted after incision and drainage of the abscess.
(u) D. Application of moist heat can localize or consolidate the abscess when the furuncle is firm and non-fluctuant
aiding in the development and drainage of the abscess.
Ref: (10)



207. Diagnosis/Hematology
A mean corpuscular volume (MCV) of less than 80 cubic microns is a manifestation of which of the following
diagnoses?
A. Iron deficiency
B. Vitamin B12 deficiency
C. Folate deficiency
D. G6PD deficiency
Explanations
(c) A. Iron deficiency is associated with microcytic anemia.
(u) B. Vitamin B12 deficiency is associated with macrocytic anemia.
(u) C. Folate deficiency is associated with macrocytic anemia.
(u) D. G6PD deficiency is not associated with a low MCV.
Ref: (28)

Copyright 2010. Physician Assistant Education Association

79
208. History & Physical/Pulmonology
A 55 year-old female presents to the emergency department with complaints of dyspnea, chest pain and coughing
with hemoptysis. Past medical history includes breast cancer 5 years ago, currently in remission. Vital signs are
Temp. 98.6 degrees F, BP 150/90 mmHg, P 110 bpm, RR 20. Physical examination shows her right leg swollen with
pain on palpation of deep veins. Which of the patient's history or examination findings is most suggestive of a
pulmonary embolus (PE)?
A. Leg swelling and pain with palpation of deep veins
B. Heart rate >100
C. Hemoptysis
D. Past history of cancer
Explanations
(c) A. Leg swelling and pain with palpation of the deep veins are consistent with a DVT and increase the
likelihood of a PE.
(u) B. While a tachycardic rate may be present in a patient with a PE, it is not specific for a PE.
(u) C. While hemoptysis may be present in a PE, it is not a specific finding.
(u) D. History of cancer may place a patient at increased risk for PE, however is not the most suggestive finding
in this patient.
Ref: (28)



209. Diagnosis/Neurology
A 57 year-old male presents with episodic diplopia over the past two months. Symptoms progressed over the last two
days with the onset of bilateral facial weakness made worse with repetitive use. Weakness improves somewhat with
rest. He denies fever, headache or areas of pain. Exam reveals a nasal voice, drooping eyelids and a normal sensory
exam. Which of the following is the most likely diagnosis?
A. Multiple sclerosis
B. Guillain-Barre syndrome
C. Lambert-Eaton syndrome
D. Myasthenia gravis
Explanations
(u) A. See D for explanation.
(u) B. See D for explanation.
(u) C. See D for explanation.
(c) D. Episodic diplopia progressing to proximal weakness worsened by repetition and lacking sensory deficits is
typical of myasthenia gravis. In Lambert-Eaton syndrome muscular responses actually improve with repetition
and the weakness in Guillain-Barre syndrome always begins peripherally.
Ref: (7)



210. Diagnostic Studies/Endocrinology
Which of the following is the best radiologic study to image the pituitary gland?
A. Lateral skull films
B. Ultrasound
C. Magnetic resonance imaging (MRI)
D. Computed tomography (CT) scan
Explanations
(u) A. Lateral skull films will not show the pituitary gland due to location.
(u) B. It is not possible to visualize the pituitary gland by ultrasound due to physical location.
(c) C. The pituitary gland and specifically pituitary tumors are best visualized by an MRI.
(u) D. CT scan is not the preferred test for imaging the pituitary gland.
Ref: (28)


Copyright 2010. Physician Assistant Education Association

80
211. Clinical Therapeutics/ENT/Ophthalmology
An otherwise healthy college student presents with complaints of cough, malaise, fever, yellow-green nasal discharge
and headache for the past 3 weeks. She complains of facial pain over the cheeks and while chewing. On
examination, the nasal turbinates are erythematous and edematous nearly obstructing the nares bilaterally. She has
not been previously evaluated. What is the most appropriate antibiotic in this patient?
A. Amoxicillin/clavulanate (Augmentin)
B. Trimethoprim/sulfamethoxazole (Bactrim)
C. Levofloxacin (Levaquin)
D. Moxifloxacin (Avelox)
Explanations
(u) A. See B for explanation.
(c) B. Acute bacterial rhinosinusitis (sinusitis) is associated with S. pneumoniae, H. Influenzae, and less commonly
S. Aureus or Moraxella catarrhalis. First line therapies for bacterial sinusitis include amoxicillin, Bactrim, or
doxycycline. Second line therapy includes Augmentin, Moxifloxacin, or Telithromycin.
(u) C. See B for explanation.
(u) D. See B for explanation.
Ref: (28)




212. Diagnostic Studies/Cardiology
A 45 year-old male presents to the Emergency Department complaining of sudden onset of tearing chest pain
radiating to his back. On examination the patient is hypertensive and his peripheral pulses are diminished.
Electrocardiogram shows no acute ST-T wave changes. Which of the following is the diagnostic study of choice in
this patient?
A. Computed tomography (CT) scan
B. Transthoracic echocardiogram
C. Magnetic resonance imaging (MRI)
D. Cardiac catheterization
Explanations
(c) A. This patient has signs and symptoms of acute aortic dissection for which CT scan is the diagnostic study of
choice.
(u) B. CT scan is better than transthoracic echocardiogram for the diagnosis of acute aortic dissection.
Transesophageal echocardiogram (TEE) is a good diagnostic modality, however it is not always available in
the acute setting.
(u) C. MRI is good in the diagnosis of a chronic aortic dissection, but the longer imaging time and the difficulty in
monitoring the patient during the test makes it not the first choice in the setting of an acute dissection.
(u) D. Cardiac catheterization is not indicated in the diagnosis of an acute aortic dissection.
Ref: (28)




213. Clinical Therapeutics/Orthopedics/Rheumatology
A 25 year-old female with irritable bowel syndrome presents with complaint of upper and lower extremity discomfort.
The patient has pinpoint muscle tenderness over the shoulder girdle, low back and hips. Which of the following
medications is the most appropriate treatment in this patient?
A. Hydrocodone (Vicodin)
B. Amitriptyline (Elavil)
C. Methotrexate (Rheumatrex)
D. Naproxen sodium (Naprosyn)
Explanations
(u) A. Opioid medication is not effective in the treatment of fibromyalgia.
(c) B. Amitriptyline is the treatment of choice for fibromyalgia and is typically administered at bedtime.
(u) C. Methotrexate is not effective in treating fibromyalgia.

Copyright 2010. Physician Assistant Education Association

81
(u) D. NSAIDs are generally ineffective in treating fibromyalgia.
Ref: (28)



214. Clinical Intervention/Gastrointestinal/Nutritional
Which of the following complications of Crohn's disease most frequently requires surgery?
A. Hemorrhage
B. Intra-abdominal abscess
C. Malabsorption
D. Perforation
Explanations
(u) A. Hemorrhage and perforation are uncommon in Crohn's disease.
(c) B. Surgery is the treatment of choice for an abscess, which is a common complication of Crohn's disease.
(u) C. Malabsorption is often a complication of the surgery itself.
(u) D. See A for explanation.
Ref: (28)



215. Health Maintenance/Pulmonology
What is the most effective prophylaxis against respiratory syncytial virus (RSV) infection in the general pediatric
population?
A. Proper hand-washing techniques
B. A monoclonal antibody
C. H. Influenzae B vaccine
D. Oseltamivir (Tamiflu)
Explanations
(c) A. Proper hand washing and reduction in exposure is most effective in general population to prevent RSV.
(u) B. Prophylaxis with a monoclonal antibody has proven effective in high risk infants but is not indicated in the
general pediatric population.
(u) C. Prophylaxis with H. Influenzae B vaccine has reduced incidence of epiglottis, not RSV.
(u) D. Oseltamivir is not indicated in the prophylaxis of RSV.
Ref: (13)



216. Clinical Therapeutics/Psychiatry/Behavioral Medicine
What are the most effective agents in treating somatoform spectrum pain disorder?
A. Analgesics
B. Antidepressants
C. Antipsychotics
D. Anxiolytics
Explanations
(u) A. See B for explanation.
(c) B. Antidepressants, especially SSRIs and TCAs are the main pharmacologic treatments used in somatoform
spectrum pain disorder as analgesics and anxiolytics provide little to no relief and generally result in
dependency. As there is no associated psychosis, delusions or hallucinations antipsychotics are not helpful.
(u) C. See B for explanation.
(u) D. See B for explanation.
Ref: (33)


217. Clinical Intervention/Urology/Renal

Copyright 2010. Physician Assistant Education Association

82
A 73 year-old male with chronic renal failure due to longstanding hypertension and diabetes is being discharged from
the hospital today. Which dietary recommendation should be made to prevent further complications and progression
of his renal disease?
A. Increase salt intake
B. Restrict dietary protein
C. Add a daily potassium supplement
D. Decrease gluten consumption
Explanations
(u) A. Increased salt intake can lead to volume overload in patients with chronic renal disease.
(c) B. Restricting protein in the diet may slow the progression of chronic kidney disease.
(u) C. Hyperkalemia can develop in patients with chronic renal failure who are taking supplements or have a high
potassium diet.
(u) D. Decreasing gluten consumption has no effect on renal failure.
Ref: (28)



218. Scientific Concepts/Cardiology
A 60 year-old male complains of progressive fatigue and dyspnea. On examination his lungs are clear to auscultation
bilaterally, heart exam reveals regular rate and rhythm without S3, S4 or murmur, and extremities show 1+edema
bilaterally. Chest x-ray reveals cardiomegaly. electrocardiogram shows low voltage, and echocardiogram shows an
ejection fraction of 55% with a small, thickened left ventricle that has rapid early filling with diastolic dysfunction.
Which of the following is the most likely underlying etiology of this patient's cardiomyopathy?
A. Alcoholism
B. Myocarditis
C. Amyloidosis
D. Chronic hypertension
Explanations
(u) A. Chronic alcohol use is commonly associated with a dilated left ventricle with left ventricular dysfunction.
(u) B. Myocarditis is associated with a dilated, not small, left ventricle.
(c) C. Amyloidosis is the most common cause of restrictive cardiomyopathy and is associated with a small
thickened left ventricle that has rapid early filling with diastolic dysfunction.
(u) D. Chronic hypertension is associated with a hypertrophic, hypercontractile left ventricle.
Ref: (28)



219. Diagnosis/Gastrointestinal/Nutritional
A 22 year-old graduate student presents with a six-month history of abdominal pain relieved with defecation which
seems to coincide with her starting PA school. She describes alternating constipation and diarrhea as well as
bloating. She denies any recent weight changes. Her only medication is citalopram (Celexa) What is the most likely
diagnosis for this patient?
A. Crohn's disease
B. Irritable bowel syndrome
C. Thyroid storm
D. Celiac disease
Explanations
(u) A. See B for explanation.
(c) B. The patient's symptoms are consistent with irritable bowel syndrome. More than 50% of patients who seek
medical attention for symptoms of IBS also have a diagnosis of depression, anxiety, or somatization.
(u) C. See B for explanation.
(u) D. See B for explanation.
Ref: (1)
220. History & Physical/Endocrinology

Copyright 2010. Physician Assistant Education Association

83
A 30 year-old female presents with amenorrhea, proximal muscle weakness, weight gain, and increased emotional
lability. Her physical examination consists of central obesity, full face, and protuberant abdomen with thin extremities.
Her blood pressure is 150/96 mmHg. Which of the following is the most likely diagnosis?
A. Addison disease
B. Hypothyroidism
C. Cushing syndrome
D. Pituitary insufficiency
Explanations
(u) A. Addison disease symptoms may include weakness but will show weight loss, anorexia, nausea, anxiety,
hyperpigmentation over the knuckles, elbows, knees and sometimes vitiligo.
(u) B. Hypothyroidism does not present with central obesity, thin extremities or moon face.
(c) C. A classic presentation of cushing syndrome is moon facies, buffalo hump, protuberant abdomen, thin
extremities, weakness, back and head ache, hypertension, acne and superficial skin infections. There are
typically purple striae and easy bruisability.
(u) D. Patients with pituitary insufficiency present with short stature and growth failure. Typically the presentation is
at an earlier age.
Ref: (28)



221. Diagnosis/Dermatology
An 18 year-old young woman comes to the office with a mildly itchy rash on her chest and back for the past two
weeks. She noticed an initial area on her back two weeks ago, and now it has spread across her back and chest. On
physical examination, she has several salmon-colored plaques with fine scale that follow the cleavage lines of the
trunk. Which of the following is the most likely diagnosis?
A. Tinea versicolor
B. Seborrheic dermatitis
C. Psoriasis
D. Pityriasis rosea
Explanations
(u) A. Tinea versicolor is a mild infection characterized by hypopigmented macules on the trunk. This eruption is
often asymptomatic-usually appearing during humid, warmer temperatures. It is most often on the upper trunk
of older teenagers or young adults. Patients may complain of mild pruritis.
(u) B. Seborrheic dermatitis consists of pruritic greasy scales and underlying erythematous patches or plaques on
the scalp, central face, presternal areas, and upper back.
(u) C. Psoriasis is characterized by chronic, recurring, erythematous papules and plaques with thick silvery white
scale on the elbows, knees, and scalp.
(c) D. Pityriasis rosea is an acute eruption of fine scaling fawn-colored papules and plaques that are distributed
along the cleavage lines of the trunk, often referred to as a Christmas tree pattern. A single plaque, called a
herald patch, precedes the secondary eruption by 1-2 weeks.
Ref: (10)



222. Diagnostic Studies/Pulmonology
An O2 saturation of 90% corresponds to what PO2 value?
A. 90 mmHg
B. 80 mmHg
C. 70 mmHg
D. 60 mmHg
Explanations
(u) A. See D for explanation.
(u) B. See D for explanation.
(u) C. See D for explanation.

Copyright 2010. Physician Assistant Education Association

84
(c) D. O2 sat values above 90% correspond with a PO2 >70 mmHg and values less than 94% represent
hypoxemia. Less than 90% O2 sat warrants measurement of arterial blood gasses.
Ref: (28)



223. Clinical Intervention/Cardiology
A 70 year-old male with history of ischemic cardiomyopathy presents with a syncopal episode. He denies complaints
of chest pain, palpitations, or dyspnea. ECG shows no acute ST-T wave changes. Echocardiogram reveals an
ejection fraction of 25% with no valvular abnormalities. Which of the following is the most appropriate management
for this patient?
A. Dual chamber permanent pacemaker
B. Diltiazem (Cardizem)
C. Implantable cardio defibrillator
D. Midodrine (ProAmatine)
Explanations
(u) A. See C for explanation.
(u) B. See C for explanation.
(c) C. This patient has ischemic cardiomyopathy and syncope, which is most likely due to ventricular tachycardia.
Instertion of a cardio defibrillator is the management of choice in this patient.
(u) D. See C for explanation.
Ref: (28)



224. Diagnostic Studies/Gastrointestinal/Nutritional
Which of the following is the diagnostic study of choice in the evaluation of Zollinger-Ellison syndrome?
A. D-xylose absorption test
B. Gastrin level
C. 5-hydroxyindoleacetic acid assay (5-HIAA)
D. Urea breath test
Explanations
(u) A. D-xylose absorption test is used in the evaluation of possible gluten-induced enteropathy.
(c) B. Zollinger-Ellison Syndrome (ZES) is characterized by elevated levels of serum gastrin resulting from gastrin-
secreting tumors usually located in the duodenum or pancreas.
(u) C. 5-HIAA levels are elevated in metastatic carcinoid tumors.
(u) D. The urea breath test is used to evaluate ulcers caused by H. pylori infection.
Ref: (28)



225. Scientific Concepts/Neurology
Which of the following neurotransmitters is decreased in early Parkinson's disease?
A. Dopamine
B. Serotonin
C. Norepinephrine
D. Acetylcholine
Explanations
(c) A. Decreased dopamine levels are typical of Parkinson's disease even early in the course of the disease.
(u) B. See A for explanation.
(u) C. See A for explanation.
(u) D. See A for explanation.
Ref: (7)

Copyright 2010. Physician Assistant Education Association

85

Physician Assistant Education Association
PACKRAT
Comment Form

Please fill out this form and return it to: Physician Assistant Education Association
300 N. Washington St., Suite 710
Alexandria, VA 22314

YES NO

1. I found the PACKRAT a useful study tool.

Comment:





2. I was able to understand the directions in the Directions and Explanations.


Comment:






3. The feedback provided was easy to understand.

Comment:





4. I know where my weaknesses lie and where to focus my studies.

Comment:






5. The content of the examination was comprehensive.

Comment:






Copyright 2010. Physician Assistant Education Association

86
6. The examination questions were clearly worded.

Comment:






7. Suggestions for improvement:

_________________________________________________________________

_________________________________________________________________

_________________________________________________________________

_________________________________________________________________

Das könnte Ihnen auch gefallen